You are on page 1of 63

Art 12 - SECTION 3

Director of Lands V Aquino


FACTS: In this case, majority of the land that was bought by Abra Industrial Corporation (AIC) for the purpose of setting
up a cement factory was put into question because the said land was still part of Central Cordillera Forest Reserve which
had not yet been released as alienable and disposable land pursuant to the Public Law Land. On the other hand, the
respondents contended that upon the application of AIC for registration of the disputed lands, nobody from the petitioners
herein objected thereby, a valid purchased.
ISSUE: Whether the land in question can be alienated from public domain
RULING & IMPORTANT POINT OF THE CASE: The SC ruled that it cannot since the power to exclude or reclassify area
from the forest zone belongs to the President of the Philippines upon the recommendation of the Secretary of Agriculture
and Natural Resources and not to the District Forester or even the Director of Forestry where AIC filed such registration
pursuant to Land Registration Act.
Furthermore, although a forest has been denuded, its classification from being such does not mean that it has ceases to
be a forest land, thus, still part of the public domain.
Republic V CA
FACTS: The subject of this case is the registration of a parcel of land by Jose dela Rosa on his own behalf and of his
children which was allegedly challenged by Benguet Consolidated Inc., Atok Big Wedge Corporation on the grounds of
being successors of James Kelly and Harrison and Reynolds as the locators of such land earlier than the claims of Jose
dela Rosa. The CA affirmed the rights of private purchaser while at the same time reserving the rights of the two private
mining companies. However, the Republic of the Philippines interposed and said that neither of the claimants have the
right to such land since its covered by the Central Cordillera Forest Reserve under Proclamation 217. Also, by reason of
its nature, it is not subject to alienation under 1935 and 1973 Constitution.
ISSUE: Whether the said land belongs to the State, the private companies or private individual?
RULING: It belongs to the two private companies. The Court ruled that the enactment of 1935 and 1973 Constitution
which prohibited the alienation of agricultural lands which are public domain cannot impair the vested rights already
vested to the two private companies prior to it. Furthermore, the perfection of the mining claim converted the property to
mineral land and under the laws then in force removed it from the public domain. By such act, the locators acquired
exclusive rights over the land, against even the government, without need of any further act such as the purchase of the
land or the obtention of a patent over it. As the land had become the private property of the locators, they had the right to
transfer the same, as they did, to Benguet and Atok.
IMPORTANT POINT OF THE CASE: The classification of the land can either be completely agricultural or mineral. In the
case at hand, the land which was originally a forest land (a public domain, thus, owned by the State and subject to the
Regalian doctrine) ceases to be so and become mineral and completely mineral.
Apex Mining V Southeast Mindanao Gold Inc.
FACTS: Due to the pressing concerns in the Diwalwal Gold Rush Area, a rich tract of mineral in Davao,brought about by
unregulated small to medium-scale mining operations causing ecological, health and peace and order problems, the
President, on 25 November 2002, issued Proclamation No. 297, which declared the area as a mineral reservation and as
an environmentally critical area. The area being a mineral reservation, the Executive Department has full control over it
pursuant to Section 5 of Republic Act No. 7942. It can either directly undertake the exploration, development and
utilization of the minerals found therein, or it can enter into agreements with qualified entities. Since the Executive
Department now has control over the exploration, development and utilization of the resources in the disputed area,
SEMs exploration permit, assuming that it is still valid, has been effectively withdrawn. The exercise of such power
through Proclamation No. 297 is in accord with jura regalia, where the State exercises its sovereign power as owner of
lands of the public domain and the mineral deposits found within.
ISSUE: Whether Proclamation No. 297 transgresses the vested rights of Marcopper Mining Corporation (MMC) and
Southeast Mindanao Gold Mining Corporation (SEM) over Diwalwal.
RULING: No. As a mere license or privilege, an exploration permit can be validly amended by the President of the
Republic when national interests suitably necessitate.

IMPORTANT POINT OF THE CASE: Recognizing the importance of the countrys natural resources, not only for national
economic development, but also for its security and national defense, Section 5 of Republic Act No. 7942 empowers the
President, when the national interest so requires, to establish mineral reservations where mining operations shall be
undertaken directly by the State or through a contractor
Director of Lands V IAC (Intermediate Appellate Court)
FACTS: Acme Plywood & Veneer Co. Inc. purchased 5 parcel of land amounting to more than 1,000 hectares from Infiels
of Dumagat tribe
which possession dates back before the Philippineswas discovered by Magellan. Morover, such
land sold to the company was considered as private land pursuant to the provisions of RA 3872 granting absolute
ownership to the tribes of Dumagat.
ISSUE: Whether or not the acquisition of said land from the tribes of Dumagat by Acme Plywoodis valid given the
prohibition of lease not exceeding 1,000 hectares
RULING: It is a valid acquisition since during the time when Acme Plywood purchased such land, the prevailing
Constitution then was the 1935 wherein the provision of not more than 1000 hectares was not provided. Moreover,
subject only to the said prohibitionwere lands of the public domain wherein in this case, the land in question was
considered as a private land.
Ten Forty Realty V Cruz
(Yung Lorenzanananasa syllabus e mali. Si Lorenzana e President ng Petitioner)
FACTS: Petitioner herein purchased from Barbara Galino by virtue of Deed of Absolute Sale parcel of house and lot.
Almost 2 years after being acquired by the petitioner, Barabara also sold the said land to Cruz. Barbara said that she did
not sell her house and lot to the petitioner but merely obtained a load from it.
ISSUE: Whether the purchase of the disputed parcel of house and lot by the petitioner is valid
RULING: No since private corporations are disqualified from obtaining lands of public domain as provided by Sec 3 of
Article XII. In this case, the petitioner failed to show that the land has ceased to be of public domain and was already a
private land.
Chavez V PEA (Public Estates Authority)
FACTS: Pursuant to the reclaimed foreshore and offshore areas in Manila Bay under the Manila-Cavite Coastal Road and
Reclamation Project (MCCRRP) by then President Marcos and the Freedom Island which were all transferred to PEA, it
entered into a Joint Venture Agreement (JVA) to with AMARI, a private corporation. Under the Agreement, PEA will now
transfer parcel of land it reclaimed to AMARI for it to develop, especially the Freedom Islands and an additional 250
hectares of land.
ISSUE: Whether AMARI, a private corporation, can acquire and own under the Amended JVA 367.5 hectares of reclaimed
foreshore and submerged areas in Manila Bay in view of Sections 2 and 3, Article XII of the 1987 Constitution
RULING: No. The mere reclamation of these areas by PEA does not convert these inalienable natural resources of the
State into alienable or disposable lands of the public domain. There must be a law or presidential proclamation officially
classifying these reclaimed lands as alienable or disposable and open to disposition or concession. Moreover, these
reclaimed lands cannot be classified as alienable or disposable if the law has reserved them for some public or quasipublic use.
IMPORTANT POINT OF THE CASE: Until a public land is classified into alienable land, it remains inalienable. Thus, those
reclaimed foreshore and submerged areas in Manila Bay are inalienable until classified as alienable.
Republic V Southside
FACTS: SHAI (Southside Homeowners Association Inc.), a non-stock corporation organized mostly by wives of AFP
military officers, was able to secure a Transfer Certificate of Title (TCT) of the JUSMAG Area which was a public domain
for being reserved for military purposes. The total purchase price as written in the conveying deed was P11,997,660.00 or
P30.00 per square meter.
ISSUE: Whether the JUSMAG area sold by SHAI through TCT No. 15084 is valid?

RULING: No. The JUSMAG area as part of the whole 15,912,684-square meter area under Proclamation 423 which
established a military reservation known as Fort Andres Bonifacio Military Reservation (FBMR) was still considered as
public domain, thus, not subject to alienation.
IMPORTANT POINT OF THE CASE: Until a given parcel of land is released from its classificationas part of the military
reservation zone and reclassified by law or by presidential proclamation as disposable andalienable, its status as part of a
military reservation remains,even if incidentally it is devoted for a purpose other thanas a military camp or for defense.
Republic V T.A.N. Properties Inc.- G.R. No. 154953
FACTS: In 1999, T.A.N. Properties filed in the RTC of Batangas an application for the registration of a land, located at Sto.
Tomas Batangas and with an area of 56.4007. To support its application, it submitted two certificates, issued by CENRO
(Certification by the Community Environment and Natural Resources Offices) and FMS-DENR (Forest Management
Services of the DENR) and both certifying that the land applied for was alienable and disposable
ISSUE: Whether the certificates provided by the respondent were enough to dispose the said land.
RULING: No. CENRO certification was insufficient to prove the alienable and disposable character of the land sought to
be registered. The applicant must also show sufficient proof that the DENR Secretary approved the land classification and
released the land in question as alienable and disposable.
IMPORTANT POINT OF THE CASE: Since the said land was still a public domain, private corporationscannot apply for
registration of the land as it is prohibited under Sec 3 of Article XII
SECTION 4 (Perodapat Section 2 to)
La Bugal-Blaan Tribal Assn. V DENR
FACTS: On July 25, 1987, then President Corazon C. Aquino issued Executive Order (E.O.) No. 2796 authorizing the
DENR Secretary to accept, consider and evaluate proposals from foreign-owned corporations or foreign investors for
contracts or agreements involving either technical or financial assistance for large-scale exploration, development, and
utilization of minerals, which, upon appropriate recommendation of the Secretary, the President may execute with the
foreign proponent.
On March 3, 1995, then President Fidel V. Ramos approved R.A. No. 7942 to "govern the exploration, development,
utilization and processing of all mineral resources."
Shortly before the effectivity of R.A. No. 7942, however, or on March 30, 1995, the President entered into an FTAA with
WMCP, a corporation organized under Philippine laws, covering 99,387 hectares of land in South Cotabato, Sultan
Kudarat, Davao del Sur and North Cotabato.
ISSUE: Whether the Philippine Mining Act and FTAA is valid?
RULING: Foreign corporations may indeed participate in the exploitation, development and use of Philippine natural
resources but subject to the full control and supervision of the State.RA 7942, its implementing rules (DAO 96-40) and the
FTAA entered into by then Government and WMCP grant the Government full control and supervision over all aspects of
planned exploration, development and utilization activities.
SECTION 5
Cruz V Secretary of DENR
FACTS: Isagani Cruz assails the constitutionality of RA 8371 or the Indigenous Peoples Rights Act of 1997 (IPRA) and it
Implementing Rules and Regulations (IRRs) wherein stated therein are provisions regarding the ownership and the rights
given to indigenous people over their land. Petitioner contended that it deprives the State of its power over such land
which is considered as public domain as well as minerals and natural resources therein.
ISSUE: Whether RA 8371 violates the Constitution
RULING: No. What is granted to ICCs/IPs is the ownership over their ancestral domains and lands. On the other hand,
the State has still ownership over natural resources found therein. Furthermore, vested to indigenous people is mere
management prerogative and not ownership over natural resources.

Section 6. Common Good


Telecom v. COMELEC, 289 SCRA 337 (1998)
Facts: Petitioner Telecommunications and Broadcast Attorneys of the Philippines, Inc. and GMA Network, Inc. operates
radio and television broadcasting stations throughout the Philippines under a franchise granted by Congress. They
challenge the validity of 92 on the ground that it is in excess of the power given to the COMELEC to supervise or regulate
the operation of media of communication or information during the period of election. Petitioners contend that COMELECs
requirement that radio and television stations provide at least 30 minutes of prime time daily for the COMELEC Time
without payment of just compensation is unconstitutional.
Issue: Whether or not section 92 of B.P. Blg. 881 is constitutional (granting COMELEC free air time from radio and
television broadcasting stations)
Ruling: No. The validity of 92 is to uphold the peoples right to information on matters of public concern. The use of
property bears a social function and is subject to the states duty to intervene for the common good. It is to hold public
broadcasters to their obligation to see to it that the variety and vigor of public debate on issues in an election is
maintained. For while broadcast media are not mere common carriers but entities with free speech rights, they are also
public trustees charged with the duty of ensuring that the people have access to the diversity of views on political
issues. This right of the people is paramount to the autonomy of broadcast media.
More than merely depriving candidates of time for their ads, the failure of broadcast stations to provide air time unless
paid by the government would clearly deprive the people of their right to know. Art. XII, 6 states that the use of property
bears a social function [and] the right to own, establish, and operate economic enterprises [is] subject to the duty of the
State to promote distributive justice and to intervene when the common good so demands.
Section 7. Private Lands
Republic v CA 235 SCRA 567
Facts: Respondent spouses bought Lots 347 and 348, Cad. s38-D, as their residence with a total area of 91.77 sq. m.
situated in San Pablo City, from one Cristeta Dazo Belen, the predecessors-in-interest who have been in open,
continuous, exclusive and notorious possession of the disputed land not only since June 12, 1945. At the time of the
purchase, respondent spouses where then natural-born Filipino citizens. On February 5, 1987, the spouses filed an
application for registration of title of the two (2) parcels of land before the Regional Trial Court of San Pablo City , Branch
XXXI. This time, however, they were no longer Filipino citizens and have opted to embrace Canadian citizenship through
naturalization.
Petitioner contends that privately owned unregistered lands are presumed to be public lands under the principle that lands
of whatever classification belongs to the State under the Regalian doctrine. Thus, before the issuance of the certificate of
title, the occupant is not in the juridical sense the true owner of the land since it still pertains to the State. Petitioner further
argued that it is only when the court adjudicates the land to the applicant for confirmation of title would the land become
privately owned land, for in the same proceeding, the court may declare it public land, depending on the evidence.
Issue: Can a foreign national apply for registration of title over a parcel of land which he acquired by purchase while still a
citizen of the Philippines?
Ruling: YES. Even if private respondents were already Canadian citizens at the time they applied for registration of the
properties in question, said properties as discussed above were already private lands; consequently, there could be no
legal impediment for the registration thereof by respondents in view of what the Constitution ordains. The parcels of land
sought to be registered no longer form part of the public domain. They are already private in character since private
respondents' predecessors-in-interest have been in open, continuous and exclusive possession and occupation thereof
under claim of ownership prior to June 12, 1945 or since 1937.
Zaragoza v CA G.R no. 106401 September 29, 2000

Facts: Jose Eugenio Ramirez, a Filipino national, died in Spain on December 11, 1964, with only his widow as compulsory
heir. His will was admitted to probate by the Court of First Instance of Manila, Branch X, on July 27, 1965. Maria Luisa
Palacios was appointed administratrix of the estate. In due time she submitted an inventory of the estate and one third
(1/3) of the free portion is charged with the widow's usufruct and the remaining two-thirds (2/3) with a usufruct in favor of
Wanda, an Australian national. The appellants claim that the usufruct over real properties of the estate in favor of Wanda
is void because it violates the constitutional prohibition against the acquisition of lands by aliens.
Issue: WON the will granting a usufructuary rights over real property to an Austrian National is valid
Ruling: Yes. The court a quo upheld the validity of the usufruct given to Wanda on the ground that the Constitution covers
not only succession by operation of law but also testamentary succession. This opinion notwithstanding, uphold the
usufruct in favor of Wanda because a usufruct, albeit a real right, does not vest title to the land in the usufructuary and it is
the vesting of title to land in favor of aliens which is prohibited by the Constitution.
Halili v. CA 287 SCRA 465
Facts: Simeon De Guzman an American citizen, died leaving properties in the Philippines. His forced heirs were his
widow, Helen Meyers Guzman, and his son, David Rey Guzman, both of whom are also American citizens. On August 9,
1989, Helen executed a deed of quitclaim assigning, transferring and conveying to David Rey all her rights, titles and
interests in and over six parcels of land which the two of them inherited from Simeon. Thereafter David Rey Guzman sold
said parcel of land to Emiliano Cataniag, a Filipino Citizen.
Petitioners, who are owners of the adjoining lot, filed a complaint before the Regional Trial Court of Malolos, Bulacan,
questioning the constitutionality and validity of the two conveyances -- between Helen Guzman and David Rey Guzman,
and between the latter and Emiliano Cataniag -- and claiming ownership thereto based on their right of legal redemption.
Issue: WON a subsequent sale of land by the disqualified alien vendee to a qualified Filipino citizen valid
Ruling: Yes. Jurisprudence is consistent that if land is invalidly transferred to an alien who subsequently becomes a citizen
or transfers it to a citizen, the flaw in the original transaction is considered cured and the title of the transferee is rendered
valid. This principle is to preserve the nations lands for future generations of Filipinos, that aim or purpose would not be
thwarted but achieved by making lawful the acquisition of real estate by aliens who became Filipino citizens by
naturalization. Accordingly, since the disputed land is now owned by Private Respondent Cataniag, a Filipino citizen, the
prior invalid transfer can no longer be assailed. The objective of the constitutional provision -- to keep our land in Filipino
hands -- has been served.
Lee v Republic
Facts: Rafael, Carmen, Francisco, Jr., Ramon, Lourdes, Mercedes, Concepcion, Mariano, Jose, Loreto, Manuel, Rizal
and Jimmy, all surnamed Dinglasan sold to Lee Liong, a Chinese citizen, a parcel of land situated at the corner of Roxas
Avenue and Pavia Street, Roxas City.
However, in 1948, the former owners filed with the Court of First Instance, Capiz an action against the heirs of Lee
Liong, who are Filipino Citizens, for annulment of sale and recovery of land. The plaintiffs assailed the validity of the sale
because of the constitutional prohibition against aliens acquiring ownership of private agricultural land, including
residential, commercial or industrial land.
Issue: WON the sale of land to a Chinese national who subsequently bequeath the said land to Filipino Citizens can be
revoked
Ruling: No, The constitutional proscription on alien ownership of lands of the public or private domain was intended to
protect lands from falling in the hands of non-Filipinos. In this case, however, there would be no more public policy
violated since the land is in the hands of Filipinos qualified to acquire and own such land. If land is invalidly transferred to
an alien who subsequently becomes a citizen or transfers it to a citizen, the flaw in the original transaction is considered
cured and the title of the transferee is rendered valid. Thus, the subsequent transfer of the property to qualified Filipinos
may no longer be impugned on the basis of the invalidity of the initial transfer. The objective of the constitutional provision
to keep our lands in Filipino hands has been achieved.

Frenzel v Catito
Facts: Petitioner Alfred Fritz Frenzel is an Australian citizen of German descent had amorous relationship with Erlinda
Ederlina Catito, a Filipina and a native of Bajada, Davao City who then married to a German national. During the period of
their common-law relationship, plaintiff through his own efforts and resources acquired in the Philippines real and personal
properties and were named after Erlinda. When their relationship began deteriorating when the latter failed to obtained
divorce, the petitioner started to live apart with the respondent to avoid complications. He filed a case in RTC for the
recovery of real and personal properties located in Davao, Quezon City and Manila. The petitioner contends that he
purchased the three parcels of land subject of his complaint because of his desire to marry the respondent.
Issue: Whether or not an alien can claim compensation for the sale of properties he bought named after his Filipino
common law wife
Ruling: Lands of the public domain, which include private lands, may be transferred or conveyed only to individuals or
entities qualified to acquire or hold private lands or lands of the public domain. Aliens, whether individuals or corporations,
have been disqualified from acquiring lands of the public domain. Hence, they have also been disqualified from acquiring
private lands.
Even if, as claimed by the petitioner, the sales in question were entered into by him as the real vendee, the said
transactions are in violation of the Constitution; hence, are null and void ab initio. A contract that violates the Constitution
and the law, is null and void and vests no rights and creates no obligations. It produces no legal effect at all. The
petitioner, being a party to an illegal contract, cannot come into a court of law and ask to have his illegal objective carried
out. One who loses his money or property by knowingly engaging in a contract or transaction which involves his own
moral turpitude may not maintain an action for his losses.
Lentfer v Wolf
Facts: The petitioners are Gunter Lentfer, a German citizen; his Filipina wife, Victoria Moreo-Lentfer; and John
Craigie Young Cross, an Australian citizen.. Respondent Hans Jurgen Wolff is a German citizen.
Petitioners alleged that with respondent, they engaged the notarial services of Atty. Rodrigo C. Dimayacyac for: (1)
the sale of a beach house owned by petitioner Cross in Sabang, Puerto Galera, Oriental Mindoro, and (2) the assignment
of Cross contract of lease on the land where the house stood. The sale of the beach house and the assignment of the
lease right would be in the name of petitioner Victoria Moreo-Lentfer, but the total consideration of 220,000 Deutschmarks
(DM) would be paid by respondent Hans Jurgen Wolff.
According to respondent, however, the Lentfer spouses were his confidants who held in trust for him, a time deposit
account in the amount of DM 200,000 at Solid Bank Corporation. Apprised of his interest to own a house along a beach,
the Lentfer couple urged him to buy petitioner Cross beach house and lease rights in Puerto Galera. Respondent agreed
and through a bank-to-bank transaction, he paid Cross the amount of DM 221,700 as total consideration for the sale and
assignment of the lease rights. However, Cross, Moreo-Lentfer and Atty. Dimayacyac surreptitiously executed a deed of
sale whereby the beach house was made to appear as sold to Moreo-Lentfer for only P100,000. The assignment of the
lease right was likewise made in favor of Moreo-Lentfer. Upon learning of this, respondent filed a Complaint docketed as
Civil Case No. R-4219 with the lower court for annulment of sale and reconveyance of property with damages and prayer
for a writ of attachment.
Issue: Whether or not a foreign national can seek for an annulment of sale and reconveyance of property
Ruling: Yes. The subject properties consist of a beach house and the lease right over the land where the beach house
stands is not included in the constitutional prohibition against aliens from owning land in the Philippines. A clear distinction
exists between the ownership of a piece of land and the mere lease of the land where the foreigners house stands. Thus,
we see no legal reason why reconveyance could not be allowed. Since reconveyance is the proper remedy, respondents
expenses for the maintenance and repair of the beach house is for his own account as owner thereof.
Muller v Muller

Facts: Petitioner Elena Buenaventura Muller and respondent Helmut Muller were married in Hamburg, Germany on
September 22, 1989. Respondent had inherited a house in Germany from his parents which he sold and used the
proceeds for the purchase of a parcel of land in Antipolo, Rizal and the construction of a house. The Antipolo property was
registered in the name of petitioner. Due to incompatibilities and respondents alleged womanizing, drinking, and
maltreatment, the spouses eventually separated. On September 26, 1994, respondent filed a petition for separation of
properties before the Regional Trial Court of Quezon City. With regard to the Antipolo property, the court held that it was
acquired using paraphernal funds of the respondent. However, it ruled that respondent cannot recover his funds because
the property was purchased in violation of Section 7, Article XII of the Constitution. The petitioner now is seeking for
reimbursement.
Issue: Whether respondent is entitled to reimbursement of the funds used for the acquisition of the Antipolo property.
Ruling: No. The respondent cannot seek reimbursement on the ground of equity where it is clear that he willingly and
knowingly bought the property despite the constitutional prohibition also it would in effect permit respondent to enjoy the
fruits of a property which he is not allowed to own. Thus, it is likewise proscribed by law.
Aliens, whether individuals or corporations, are disqualified from acquiring lands of the public domain. Hence, they are
also disqualified from acquiring private lands. The primary purpose of the constitutional provision is the conservation of
the national patrimony.

Matthews v Taylor Spouses


Facts: Respondent Benjamin A. Taylor (Benjamin), a British subject, married Joselyn C. Taylor (Joselyn), a 17-year old
Filipina. While their marriage was subsisting, Joselyn bought from Diosa M. Martin a 1,294 square-meter lot (Boracay
property) situated at Manoc-Manoc, Boracay Island and converted the property to a vacation and tourist resort.
However, Benjamin and Joselyn had a falling out, and Joselyn ran away with Kim Philippsen. On June 8, 1992, Joselyn
executed a Special Power of Attorney (SPA) in favor of Benjamin, authorizing the latter to maintain, sell, lease, and sublease and otherwise enter into contract with third parties with respect to their Boracay property. On July 20, 1992, Joselyn
as lessor and petitioner Philip Matthews as lessee, entered into an Agreement of Lease (Agreement) involving the
Boracay property for a period of 25 years, with an annual rental of P12,000.00. The agreement was signed by the parties
and executed before a Notary Public. Petitioner thereafter took possession of the property and renamed the resort as
Music Garden Resort.
Claiming that the Agreement was null and void since it was entered into by Joselyn without his (Benjamins) consent,
Benjamin instituted an action for Declaration of Nullity of Agreement of Lease with Damages against Joselyn and the
petitioner. Benjamin claimed that his funds were used in the acquisition and improvement of the Boracay property, and
coupled with the fact that he was Joselyns husband, any transaction involving said property required his consent.
Issue: Whether or not Benjamin has a right to nullify the Agreement of Lease between Joselyn and petitioner.
Ruling: no. Benjamin has no right to nullify the Agreement of Lease between Joselyn and petitioner. Benjamin, being an
alien, is absolutely prohibited from acquiring private and public lands in the Philippines. Considering that Joselyn
appeared to be the designated "vendee" in the Deed of Sale of said property, she acquired sole ownership thereto. This is
true even if we sustain Benjamins claim that he provided the funds for such acquisition. By entering into such contract
knowing that it was illegal, no implied trust was created in his favor; no reimbursement for his expenses can be allowed;
and no declaration can be made that the subject property was part of the conjugal/community property of the spouses.

FACTS
JACOBUS BERNHARD HULST v. PR
BUILDERS INC.

ISSUE
Whether or not
the spouses
request for

HELD
Instant petition is hereby
granted.

DOCTRINE
Sec. 7 of Art XII of the 1987
Constitution

The Petitioner and his spouse, both Dutc


h Nationals,
entered into a Contract to Sell with PR
Builders, Inc. to purchase a 210-sq m
residential unit in the respondent's
townhouse project in Batangas.
When respondent failed to comply with
its verbal promise to complete the
project by June 1995, the spouses Hulst
filed before the Housing and Land Use
Regulatory Board (HLURB) a complaint
for rescission of contract with interest,
damages and attorney's fees

damages is
actionable

TING HO v. TENG
The controversy revolves around a
parcel of land, and the improvements
which should form part of the estate of
their deceased father, Felix Ting Ho, and
should be partitioned equally among
each of the siblings.

Assuming arguendo that the litigated lots


were actually the properties of Chiong
Tan Sy and that the same were only put
in the name of respondents father
because he was the only Filipino citizen
in the family at the time the properties
were purchased.
BEURMER v. AMORES
Willem (Beumer), a Dutch national,
married Avelina (Amores) on March 29,
1980. Their marriage was declared a

*A foreigner may own a unit in


a
condo
because
the
prohibition on aliens is only
from acquiring land. The land
which the condo stands is
owned by condo corp.

Instant petition is hereby


denied.
Whether or not
the said
property
should be
included in the
estate of the
deceased Felix
Ting Ho.

OSMENA v. OSMENA
The parties to this case are descendants
of spouses Quintin Chiong Osmea and
Chiong Tan Sy. Petitioner is the couples
daughter while respondents Nicasio and
Jose Osmea are their grandchildren.
The dispute revolves around two parcels
of land, Lots 4 and 5, and the ancestral
house standing on Lot 4.

It is significant to note
that
the
agreement
executed by the parties in
this case is a Contract to
Sell and not a contract of
sale.
A
distinction
between the two is
material
in
the
determination of when
ownership is deemed to
have been transferred to
the buyer or vendee and,
ultimately, the resolution
of the question on
whether the constitutional
proscription has been
breached.

This rule, however, is subject


to exceptions that permit the
return of that which may have
been given under a void
contract
to:
the
party
repudiating the void contract
before the illegal purpose is
accomplished
or
before
damage is caused to a third
person and if public interest
is sub served by allowing
recovery (Art. 1414, Civil
Code)

The purchase is made in


violation of an existing
statute and in evasion of
its express provision; no
trust can result in favor of
the party who is guilty of
the fraud. To hold
otherwise would allow
circumvention of the
constitutional prohibition.

Instant petition is hereby


denied.

Whether or not
the said
property
should be
included in the
estate of the
deceased
Chiong Tan Sy

Whether or not
Willem is
entitled to the
whole or at
least one half
of the purchase

By signing the deed of


sale dated April 26, 1982
(where
petitioner
transferred her share in
the ancestral house to
respondents
father),
petitioner would have
been a party to the
alleged
simulated
document. Thus, making
it clear that the disputed
property is owned by
respondents father with
the consent of the
petitioners.
Instant petition is hereby
denied.
Where it
willingly
bought
despite

is clear that he
and knowingly
the
property
the prohibition

Save for the exception


provided in cases of hereditary
succession, respondent's
disqualification from owning
lands in the Philippines is
absolute. Not even an
ownership in trust is
allowed.

Court will not consent to any


violation of the constitutional
prohibition on foreign
ownership of land.

The Constitution itself which


demarcates the rights of
citizens and non-citizens in
owning Philippine land. To be
sure, the constitutional ban
against foreigners applies only

nullity by the RTC on November 10, 2000


by reason of psychological incapacity,
thus Willem filed a petition for dissolution
of conjugal partnership and distribution of
properties which he claimed were
acquired during their marriage.

price of the lots


subject of the
case.

against foreign ownership


of land2 enshrined under
Section 7, Article XII of
the
1987
Philippine
Constitution

to ownership of Philippine land


and not to the improvements
built thereon.

SEC. 8
REPUBLIC V CA (235 scra 567)

Whether or not a
foreign national
can apply for
registration of title
over a parcel of
land which he
acquired by
purchase while
still a citizen of the
Philippines, from a
vendor who has
complied with the
requirements for
registration under
the Public Land
Act (CA 141)

On June 17, 1978, respondent spouses


bought Lots 347 and 348, Cad. s38-D,
as their residence with a total area of
91.77 sq. m. situated in San Pablo City,
from one Cristeta Dazo Belen (Rollo, p.
41). At the time of the purchase,
respondent spouses where then
natural-born Filipino citizens.
On February 5, 1987, the spouses filed
an application for registration of title of
the two (2) parcels of land before the
Regional Trial Court of San Pablo City,
Branch XXXI. This time, however, they
were no longer Filipino citizens and
have opted to embrace Canadian
citizenship through naturalization
MHC v. GSIS
Whether or not the
The Government Service
provisions of the
Insurance System (GSIS),
Constitution, particularly
pursuant to the privatization
Article XII Section 10, are
program of the Philippine
self-executing
Government
under
Proclamation 50 dated 8
December 1986, decided to
sell through public bidding
30% to 51% of the issued
and outstanding shares of
the Manila Hotel (MHC). In
a close bidding held on 18
September 1995 only two
bidders participated: Manila
Prince Hotel Corporation, a
Filipino corporation, which
offered to buy 51% of the
MHC or 15,300,000 shares
at P41.58 per share, and
Renong
Berhad,
a
Malaysian firm, with ITTSheraton as its hotel
operator, which bid for the
same number of shares at
P44.00 per share, or P2.42
more than the bid of
petitioner.

Decision appealed from is


affirmed.
In the case at bar, private
respondents were
undoubtedly natural-born
Filipino citizens at the time of
the acquisition of the
properties and by virtue
thereof, acquired vested
rights thereon, tacking in the
process, the possession in
the concept of owner and the
prescribed period of time held
by their predecessors-ininterest under the Public Land
Act
Instant petition is hereby
denied.
10, paragraph 2, Article
XII of the 1987 Constitution
is a self-executing provision
and
does
not
need
implementing legislation to
carry it into effect.
In the instant case, where a
foreign firm submits the
highest bid in a public
bidding concerning the grant
of rights, privileges and
concessions covering the
national
economy
and
patrimony,
thereby
exceeding the bid of a
Filipino, there is no question
that the Filipino will have to
be allowed to match the bid
of the foreign entity. And if
the Filipino matches the bid
of a foreign firm the award
should go to the Filipino. It
must be so if we are to give
life
and
meaning
to
the Filipino
First
Policy provision of the 1987
Constitution.

The time to determine


whether the person
acquiring land is
qualified is the time the
right to own it is acquired
and not the time to
register ownership.

The second paragraph is a


mandatory,
positive
command which is complete
in itself and which needs no
further
guidelines
or
implementing rules for its
enforcement. It is per se
judicially enforceable.
A constitutional provision
may be self-executing in
one part and non-selfexecuting in another.

SEC. 10

ARMY AND NAVY CLUB v. CA


City of Manila is the owner of a parcel of
land with an area of 12,705.30 sq. m.
located at South Boulevard corner
Manila Bay, Manila, covered by TCT No.
156868/1059 of the Register of Deeds
of Manila, together with the
improvements thereon known as the
Army and Navy of Manila;
Petitioner entered into a lease contract
with private respondent sometime in
January, 1983.

Whether or
not
Army
and Navy of
Manila the
lessee of the
public land
could
be
ejected on
the ground
of
nonpayment
rents

Petitioner failed to pay the rents for


seven (7) consecutive years.

(alam niyo na to)

Ordinance No. 121 was passed by the


city of Pasay for the reclamation of
foreshore lands within their jurisdiction
and entered into an agreement with
Republic Real Estate Corporation for
the said project.
Republic questioned the agreement. It
contended, among others, that the
agreement between RREC and the City
of Pasay was void for the object of the
contract is outside the commerce of
man, it being a foreshore land.

The argument
that it was
declared
a
historical
landmark, is
not
a
substantial
issue of fact
which
does
not, in any
way, alter or
affect
the
merit of the
ejectment suit.

As a rule, The Senate may play around a


mandatory provisions through a balancing of
values in the Constitution to allow of WTO
agreement

TANADA v. TUVERA

REPUBLIC v. CA (299 scra 199)

Decision appealed from is affirmed.


Be that as it may, at this last stage, after
herein petitioner has dealt with the private
respondent as the owner of the leased
premises and obtained benefits from said
acknowledgment of such ownership for almost
half a century, herein petitioner cannot be
permitted to assume an inconsistent position
by denying said private respondent's
ownership of the leased premises when the
situation calls for it. Herein petitioner cannot
be allowed to double deal, recognizing herein
private respondent's title over the leased
premises and entering into a lease contract
and other covenants, and thereafter after
failing to comply with its obligation provided
for in the lease agreement attempt to
repudiate the ownership of private respondent
of the subject property.

Whether or
not the term
foreshore
land
includes the
submerged
area.
Whether or
not
foreshore
land and
the
reclaimed
area is
within the
commerce
of man.

The Court ruled that it is erroneous and


unsustainable to uphold the opinion of the
respondent court that the term foreshore
land includes the submerged areas. To
repeat, the term "foreshore lands" refers to:
The strip of land that lies between the high
and low water marks and that is alternately
wet and dry according to the flow of the tide.

The State
shall protect
its foreshore
land as
considered
within the
commerce of
man which
would be a
subject for
proper
reclamation
agreement.

JG Summit v. CA
During bidding, Kawasaki/PHI Consortium
is the losing bidder. Even so, because of
the right to top by 5% percent the highest
bid, it was able to top JG Summits bid. JG
Summit protested, contending that
PHILSECO, as a shipyard is a public utility
and, hence, must observe the 60%-40%
Filipino-foreign capitalization. By buying
87.67% of PHILSECOs capital stock at
bidding, Kawasaki/PHI in effect now owns
more than 40% of the stock.

Bagatsing v. Committee on Privatization


PETRON was originally registered with
the Securities and
Exchange Commission (SEC)
in 1966 under the corporate name "Esso
Philippines, Inc." .ESSO became a whollyowned company of the government
under the corporate name PETRON and as
a subsidiary of
PNOC.PETRON owns the largest, most m
odern complex refinery
in the Philippines. It is listed as the
No. 1corporation in terms of assets and inc
ome in the Philippines in 1993

Whethe
r or not
Kawasa
ki/PHI
can
purchas
e
beyond
40% of
PHILSE
COs
stocks

A shipyard such as PHILSECO


being a public utility as
provided by law is therefore
required to comply with the
60%-40% capitalization under
the Constitution. Likewise, the
JVA between NIDC and
Kawasaki
manifests
an
intention of the parties to abide
by this constitutional mandate.

Foreign citizens, corporation or


association cannot purchase
beyond 40% of the capitalization
of the joint venture on account of
both constitutional and
contractual proscriptions.

Whethe
r or not
Petron
is a
public
utility

Petron, the refining company


of the government, is not
considered a public utility
because it does not engage in
oil refining for hire or
compensation

A public utility under the


Constitution and Pubic Service
law is one organized for hire or
compensation to serve the public

SECTION 11. PUBLIC UTILITIES


ALBANO V. REYES, 175 SCRA 36 (1997)
Facts: Congressman Albano is questioning the grant of the bidding for the development, management and operation of
the Manila International Container Terminal (MICT) at the Port of Manila to respondent International Container Terminal
Services, Inc. (ICTSI) because there is no legislative franchise for it the MICT.
Doctrine: Certain agencies are given the power to provide services. The Philippine Port Authority is authorized to provide
services within its Port Districts whether on its own, by contract, or otherwise. Special franchise is not needed if the
agency is given authority.
TATAD V. GARCIA, 243 SCRA 436 (1995)
Facts: The Government wanted to build a rail transit along EDSA. The EDSA LRT Consortium, organized under
Hongkong Laws, was the only corporation who qualified for the project. EDSA LRT Consortium and DOTC came into an
agreement which stipulated that:
EDSA LRT Consortium shall build the facilities, i.e., railways, and shall supply the train cabs. Every phase that is
completed shall be turned over to the DOTC and the latter shall pay rent for the same for 25 years. By the end of 25

years, it was projected that the government shall have fully paid EDSA LRT Consortium. Thereafter, EDSA LRT
Consortium shall sell the facilities to the government for $1.00.
The petitioners allege that the EDSA LRT Consortium is a foreign company and cannot own a public utility.

Doctrine:
It is the franchise to operate a public utility that cannot be granted to foreigners. Owning the facilities to build a
public utility is not prohibited. Besides, the DOTC will be the one to operate. They will just pay rent to the EDSA
LRT Consortium for 25 years as payment for building the rail transit.

TELECOM V. COMELEC, 289 SCRA 337 (1998)


Facts:GMA is saying that they have lost millions of pesos in 1992 and 1995 because the COMELEC required it to
broadcast free-of-charge through television or radio the campaigns of candidates (the COMELEC times will be divided
equally and impartially to all candidates). GMA is saying that Sec 92 of BP 881 (the law requiring it to broadcast ads) is
unjust.
Doctrine: GMA does not own the airwaves for which they broadcast. The franchise given is only temporary use of the
airwaves. It is not unjust to require GMA to broadcast free-of-charge on behalf of the State.
Note:Telecommunications and Broadcast Attorneys of the Philippines, Inc. (TELEBAP) is an organization of lawyers of
radio and television broadcasting companies. It was declared to be without legal standing to sue in this case as, among
other reasons, it was not able to show that it was to suffer from actual or threatened injury as a result of the subject law.
JG SUMMIT HOLDINGS V. CA, 345 SCRA 143 (2000)
Facts: The National Investment and Development Corporation (NIDC) and Kawasaki Heavy Industries entered into a joint
venture agreement for the construction, operation, and management of the Subic National Shipyard, Inc., which would
later become the Philippine Shipyard and Engineering Corporation (PHILSECO). The Government eventually owned the
title and interest of the entire PHILSECO.
PHILSECO was decided to be privatized for the interest of National Economy by selling 87.67% of total outstanding
capital stock. There was a bid and Kawasaki was given the right to top by 5% the highest bidder. Kawasaki lost to JG
Summit but Kawasaki availed of its right to top off JG Summits bid by 5%. JG summit contests that Kawasaki cannot own
more than 40% of PHILSECO because Kawasaki is a foreign company.
Doctrine: The law that declares shipyards as not public utility has been repealed. Thus, shipyards must conform to the
60-40 Filipino-foreign rule.

REPUBLIC V. EXPRESS TELECOM 373 SCRA 316


Facts:Bayan Telecommunications, Inc. filed an application with the National Telecommunications Commission (NTC) for a
Certificate of Public Convenience or Necessity (CPCN) to install, operate and maintain a digital Cellular Mobile Telephone
System/Service (CMTS) with prayer for a Provisional Authority (PA). This eventually was granted in 2000.
Extelcom contends that there are no available radio frequencies that could accommodate a new CMTS operator because
the frequency bands have been allocated to the existing CMTS Operators (Extelcom, Globe Telecom, Inc., Smart
Communication, Inc., Pilipino Telephone Corporation, and Isla Communication Corporation, Inc).
Doctrine:Extelcom does not enjoy the grant of any vested interest on the right to render a public service. The
Constitution is quite emphatic that the operation of a public utility shall not be exclusive. There is no compelling reason
then in barring Bayantel from operating a CMTS.
DEL MAR V. PAGCOR [2001]

Facts: In 2000, the SC has decided that PAGCOR was bereft of any franchise to operate, maintain or manage jai-alai
games whether by itself alone or in conjunction with its co-respondents. PAGCOR is seeking for reconsideration.
Doctrine: The franchise given to PAGCOR is to operate jai-alai games only by itself. It cannot include others in operating
such games.
Note:Petition is PARTIALLY granted in that PAGCOR alone is allowed to operate jai-alai games.
PTC V. NTC, GR 138295, AUG. 28, 2003
Facts: The National Telecommunications Commission (NTC) has granted the International Communications Corporation
(ICC) its application for Provisional Authority (PA) to construct, operate and maintain local exchange services in some of
the areas covered by PILTELs PA.Piltel is opposed to the grant.
The areas covered by PILTELs PA included Sulu, Zamboanga del Norte, Zamboanga del Sur, Tawi-Tawi, Misamis
Occidental, Davao del Sur, South Cotabato, Saranggani and Davao City while among the areas included in ICCs
application were Misamis Occidental, Zamboanga del Sur, Davao del Sur, South Cotabato and Saranggani.
Doctrine: PILTELs PA is not exclusive for exclusive franchises are prohibited. PILTEL cannot oppose the application and
the grant of PA to ICC even if Misamis Occidental, Zamboangadel Sur, Davao del Sur, South Cotabato and Saranggani
are service areas of PILTEL already.
ASSOCIATED COMMUNICATIONS V. NTC, GR NO. 144109, FEBRUARY 17, 2003
Facts:Associated Communications & Wireless Services United Broadcasting Network, Inc. (ACWS) was granted by
Congress a 50-year franchise to construct, install, maintain and operate public radiotelephone and radiotelegraph coastal
stations, and public fixed and public based and land mobile stations within the Philippines for the reception and
transmission of radiotelephone and radiotelegraph for domestic communications and provincial telephone systems in
certain provinces.
When P.D. No. 576-A was enacted, all franchises to operate radio or television broadcasting systems are terminate on
December 31, 1981. ACWS was not able to renew its franchise. They were ordered to CEASE and DESIST from further
operating Channel 25.
Doctrine: In order to operate a television station, the constitution requires legislative franchise. Without it, one cannot
operate. Statutes contrary to this cannot be upheld.
EASTERN TELECOM AND TELECOM TECHNOLOGIES V. INTERNATIONAL COMMUNICATION CORPORATION, GR
NO. 135992, JULY 23, 2004
Facts: The National Telecommunications Commission (NTC) has granted the International Communications Corporation
(ICC) its application for Provisional Authority (PA) to construct, operate and maintain local exchange services in some of
the areas covered by the PA of Telecommunications Technologies Philippines, Inc. (TTPI), an affiliate of petitioner Eastern
Telecommunications Philippines, Inc.
The areas covered by TTPIs PA are the Provinces of Batanes, Cagayan Valley, Isabela, Kalinga-Apayao, Nueva Vizcaya,
Ifugao, Quirino, the cities of Manila and Caloocan, and the Municipality of Navotas, Metro Manila. ICCs PA includes
Manila and Navotas. TTPI is contending the PA awarded to ICC.
Doctrine:TPPIs PA is not exclusive for exclusive franchises are prohibited. TPPI cannot oppose the application and the
grant of PA to ICC even if Manila and Navotas are service areas of TPPI already.
ROYAL CARGO CORP. V. CAB 421 SCRA 21
Facts: Royal Cargo Corp (RCC) was originally granted by the Civil Aeronautics Board (CAB) of an indefinite authority to
engage in international air freight forwarding. RCC is 70% owned by Filipinos. The President is a foreigner but all other
company officers (Chairman of the Board included) are Filipinos.
While renewing for permit, CAB recommended that the Presidency be transferred to a Filipino or the permit will be
cancelled.

Doctrine: All the executive and managing officers of such corporation or association must be citizens of the Philippines
pursuant to Sec 11.
Note: The permit of RCC expired before final resolution was drawn. The petition was dismissed. Still, it would have lost
anyway.
Metropolitan v. Adala 526 SCRA 465 [2007]
Facts:
The Metropolitan Cebu Water District, a public corporation, appealed the decision of RTC in favor of Margarita A. Adala by
the National Water Resources Board (NWRB), granting her a permit to supply water to three sitios in Bulacao, Cebu city.
MCWD was the exclusive distributor of water in the district. MCWD argued that the proposed waterworks would interfere
with their water supply which it has the right to protect, and the water needs of the residents in the subject area was
already being well served by petitioner. They also claim that they were granted by Section 47 of Presidential Decree 198,
granting exclusive franchise only to public utilities. MCWD alleged that the Board of Directors of MCWD did not give
consent to the issuance of the franchise applied for by the respondent and that consent is a mandatory requirement as
expressed in the P.D. 198.
Issue:
Whether or not Section 47 of Presidential Decree 198 grants exclusive franchise to public utilities.
Held:
The court dismissed the petition for the reason that the section 47 of Presidential Decree 198 is unconstitutional. Article
XIV Section 5 of the 1973 Constitution which was ratified on January 17, 1973 the constitution in force when P.D. 198 was
issued on May 25, 1973. Thus, Section 5 of Art. XIV of the 1973 Constitution reads:
SECTION 5. No franchise, certificate, or any other form of authorization for the operation of a public
utility shall be granted except to citizens of the Philippines or to corporations or associations organized
under the laws of the Philippines at least sixty per centum of the capital of which is owned by such
citizens, nor shall such franchise, certificate, or authorization be exclusive in character or for a
longer period than fifty years. Neither shall any such franchise or right be granted except under the
condition that it shall be subject to amendment, alteration, or repeal by the BatasangPambansa when the
public interest so requires. The State shall encourage equity participation in public utilities by the general
public. The participation of foreign investors in the governing body of any public utility enterprise shall be
limited to their proportionate share in the capital thereof.
The provision above clearly expressed the unconstitutionality of the Presidential Decree as it is in conflict with the 1973
constitution.

PAGCOR v. BIR, 645 SCRA 338


Facts:
When the Philippine Amusement and Gaming Corporation (PAGCOR) was created, P.D. No. 1067-B was issued
exempting the petitioner from the payment of any type of tax except a franchise tax of five percent(5%) of the gross
revenue. P.D. No. 1399 was also issued to further expand the scope of PAGCORs exemption. On January 1, 1998, R.A.
No. 8424, known as National Internal Revenue Code of 1997, took effect. Section 27(c) of R.A. No. 8424 provides that
Government Owned and Controlled Corporations (GOCCs) shall pay corporate income tax except PAGCOR, GSIS, SSS,
PHIC, and PCSO. With the enactment of R.A. No. 9887 on May 24, 2005, certain provisions of National Internal Revenue
Code of 1997 were amended. The amendment is Section 1 of R.A. No. 9337, which amended Section 27 (c) of the
National Internal Revenue Code of 1997 by excluding PAGCOR from the enumeration of GOCCs that are exempt from
payment of corporate income tax.
Issue:

Whether or not Section 1 (c) of R.A. No. 9337 is null and void ab initio for violating the non-impairment clause of the
Constitution.
Held:
The non-impairment clause is limited in application to laws. There is impairment if a law changes the terms of a contract
between the parties, imposes new conditions, dispenses with those agreed upon or withdraws remedies for the
enforcement of the rights of the parties. Article XII, Section 11, of the 1987 Constitution states that no franchise for the
operation of a public utility shall be granted except under the condition that such privilege shall be subject to amendment,
alteration or repeal by Congress as and when the common good so requires. Contractual Tax exemptions should not be
confused with tax exemptions granted under franchises. A franchise has the nature of a grant that is beyond the scope of
the non-impairment clause. In this case, PAGCOR was granted a franchise to operate and maintain gambling casinos,
clubs and other recreation or amusement places, sports, gaming pools, i.e., basketball, football, lotteries, etc., whether on
land or sea, within the territorial jurisdiction of the Republic of the Philippines.Under Section 11, Article XII of the
Constitution, PAGCORs franchise is subject to amendment, alteration or repeal by Congress such as the
amendment under Section 1 of R.A. No. 9377. Hence, the provision in Section 1 of R.A. No. 9337, amending
Section 27 (c) of R.A. No. 8424 by withdrawing the exemption of PAGCOR from corporate income tax, which may
affect any benefits to PAGCORs transactions with private parties, did not violate the non-impairment clause of
the Constitution.
Francisco v. TRB 633 SCRA 470 [2010]
Facts:
President Marcos issued P.D. 1112, authorizing the establishment of toll facilities of public improvements. It created the
Toll Regulatory Board (TRB) to attract private sector and allowed the collection of fees for the use of public improvements.
P.D. 1113 was also issued granting to Philippine National Construction Corporation (PNCC) a franchise to construct toll
facilities with a right to collect fees as TRB may fix. The two signed an agreement on the operation of the expressway. In
1983, the PNCC was granted a franchise over Metro Manila Expressway (MMEX). Under the provision, PNCC may sell its
franchise upon the approval of the president, and then came the 1987 Constitution with its franchise provision.
Issue:
Whether or not the TRB has the authority to grant franchise over toll way facilities
Held:
Though the petitioners argued that under the 1987 Constitution, only the Congress has the authority to grant franchise on
the operation of public utilities; but if the law already specifically authorizes an administrative body to grant a franchise or
to award a contract, the franchise emanating from the Congress is not anymore necessary. Under the power of
subordinate legislation, the Congress may validly delegate its legislative authority to issue franchises of certain public
utilizes to some administrative agencies like The TRB.

Wilson P. Gamboa v. Finance Secretary Magarito B. Teves, GR 176579, 28 June 2011.


Facts:
Petitioners wants to nullify the sale of shares of stock of Philippine Telecommunications Investment Corporation (PTIC) by
the government of the Republic of the Philippines, acting through the Inter-Agency Privatization Council (IPC), to Metro
Pacific Assets Holdings, Inc. (MPAH), an affiliate of First Pacific Company Limited (First Pacific), a Hong Kong-based
investment management and holding company and a shareholder of the Philippine Long Distance Telephone Company
(PLDT).
The petitioner questioned the sale on the ground that it also involved an indirect sale of 12 million shares (or about 6.3
percent of the outstanding common shares) of PLDT owned by PTIC to First Pacific. With this sale, First Pacifics common
shareholdings in PLDT increased from 30.7 percent to 37 percent, thereby increasing the total common shareholdings of
foreigners in PLDT to about 81.47%. According to the petitioner, this violates Section 11, Article XII of the 1987 Philippine
Constitution which limits foreign ownership of the capital of a public utility to not more than 40%.

Issue:
Whether or not the term capital in Section 11, Article XII of the Constitution refers to the total common shares only, or to
the total outstanding capital stock (combined total of common and non-voting preferred shares) of PLDT, a public utility
Held:
The Court partly granted the petition and held that the term capital in Section 11, Article XII of the Constitution refers only
to shares of stock entitled to vote in the election of directors of a public utility (to the total common shares in PLDT).
The Respondent Chairperson of the Securities and Exchange Commission was DIRECTED by the Court to apply the
foregoing definition of the term capital in determining the extent of allowable foreign ownership in respondent Philippine
Long Distance Telephone Company, and if there is a violation of Section 11, Article XII of the Constitution, to impose the
appropriate sanctions under the law.
Notes:
Definition of capital refers only to share of stock entitled to vote in the election of directors, and thus in the
present case only to common share, and not the total outstanding capital stock comprising
Express Investment v. Bayantel 687 SCRA 50 [2012]
Facts:
In this case, The Bank of New York filed a creditor-initiated petition to rehabilitate Bayantel. In due course, the receiver
recommended the rehabilitation of Bayantel by, among others, converting part of the companys debt into equity. However,
the rehabilitation receiver imposed, as a condition, that the resulting equity ownership of foreign creditors should not
exceed the 40-percent foreign ownership limit under the 1987 Constitution.
The Bank of New York disagreed, explaining that the acquisition of shares by foreign creditors would be done, both
directly and indirectly, to meet the control test under RA 7042, or the Foreign Investments Act of 1991 (FIA).
Under the proposed structure for Bayantel, the foreign creditors would convert part of Bayantels debt to common stock of
the company. As a result, they would own 40 percent of the outstanding capital stock of Bayantel, while the remaining 40
percent of the shares would be registered to a holding company that would retain the other 60 percent equity reserved for
Filipino citizens. According to The Bank of New York, this structure would comply with the control test under the FIA and,
therefore, would not violate the Filipinization requirement prescribed by the Constitution for public utilities.
Issue:
Whether or not the proposed structure would violate the foreign ownership limit imposed by the Constitution for public
utilities
Held:
The SC ruled out that the proposed structure would violate Article XII, Sec 11 of the 1987 Constitution. SC stated that if we
indulge petitioners on their proposal, the Omnibus Creditors which are foreign corporations, shall have control over 77.7%
of Bayantel, a public utility company. This is precisely the scenario proscribed by the Filipinization provision of the
Constitution. Therefore, the Court of Appeals acted correctly in sustaining the 40% debt-to-equity ceiling on conversion.
Section 12. Filipino First Policy
Tanada v. Angara, 272 SCRA 18 (1997)
Facts:
On April 15, 1994, Rizalino Navarro, representing the Government of the Philippines, signed in Marrakesh, Moricco, the
Final Act Embodying the Results of the Uruguay Round of Multilateral Negotiations.
On August 12, 1994, the members of the Philippine Senate received a letter from the President of the Philippines, stating
among others that the Uruguay Round Final Act was submitted to the Senate for its concurrence pursuant to Section 21,
Article VII of the Constitution. A day after, the members of the Philippine Senate received another letter from the President
of the Philippines which stated among others that the Uruguay Round Final Act, the Agreement Establishing the World
Trade Organization, the Ministerial Declarations and Decisions, and the Understanding on Commitments in Financial
Services are submitted to the Senate for its concurrence pursuant to Section 21, Article VII of the Constitution.

The Philippine Senate adopted Resolution No. 97 where it resolved the ratification by the President of the Philippines of
the Agreement Establishing the World Trade Organization.
It is petitioners position that the national treatment and parity provisions of the WTO Agreement place nationals and
products of member countries on the same footing as Filipinos and local products, in contravention of the Filipino First
policy of the Constitution.
Issue:
Whether or not the provisions of the Agreement Establishing the World Trade Organization contravene the provisions of
Sec. 19, Article II, and Secs.10 and 12, Article XII, of the 1987 Philippine Constitution.
Held:
The petition was dismissed for the lack of merit. The principles in Article II are not intended to be self-executing principles
ready for enforcement through the courts. They are used by the judiciary as aids or as guides in the exercise of its power
of judicial review, and by the legislature in its enactment of laws. The principles and state policies enumerated in Article II
and some sections of Article XII are not self-executing provisions, the disregard of which can give rise to a cause of action
in the courts. They do not embody judicially enforceable constitutional rights but guidelines for legislation.
Sections 10 and 12 of Article XII, apart from merely laying down general principles relating to the national economy and
patrimony, should be read and understood in relation to the other sections in said article, especially Sections 1 and 13.
While the Constitution indeed mandates a bias in favor of Filipino goods, services, labor and enterprises, at the same
time, it recognizes the need for business exchange with the rest of the world on the bases of equality and reciprocity and
limits protection of Filipino enterprises only against foreign competition and trade practices that are unfair. Therefore, the
Constitution did not shut out foreign investments, goods and services in the development of the Philippine
economy. While the Constitution does not encourage the unlimited entry of foreign goods, services and
investments into the country, it does not prohibit them either. It allows an exchange on the basis of equality and
reciprocity, demanding only on foreign competition that is unfair.

Section 13. Trade Policy


Espina v. Zamora, 631 SCRA 17
Facts:
On March 7, 2000, President Joseph E. Estrada signed into law Republic Act (R.A.) 8762, also known as the Retail Trade
Liberalization Act of 2000. It expressly repealed R.A. 1180, which absolutely prohibited foreign nationals from engaging in
the retail trade business. R.A. 8762 now allows them to do so under four categories. It also allows natural-born Filipino
citizens, who had lost their citizenship and now reside in the Philippines, to engage in the retail trade business with the
same rights as Filipino citizens. On October 11, 2000 petitioners Magtanggol T. Gunigundo I, Michael T. Defensor,
Gerardo S. Espina, all members of the House of Representatives, filed the present petition, assailing the constitutionality
of R.A. 8762.
Issue:
Whether or not R.A. 8762 is unconstitutional.
Held:
Petitioners invoked the provisions of the National Economy and Patrimony under Article XII of the 1987 Constitution. The
provisions of Article II of the 1987 Constitution, the declarations of principles and state policies, are not self-executing.
Legislative failure to pursue such policies cannot give rise to a cause of action in the courts. While Section 19, Article II of
the 1987 Constitution requires the development of a self-reliant and independent national economy effectively controlled
by Filipino entrepreneurs, it does not impose a policy of Filipino monopoly of the economic environment. The objective is
simply to prohibit foreign powers or interests from maneuvering our economic policies and ensure that Filipinos are given
preference in all areas of development. The 1987 Constitution takes into account the realities of the outside world as it
requires the pursuit of a trade policy that serves the general welfare and utilizes all forms and arrangements of exchange
on the basis of equality and reciprocity. Thus, while the Constitution mandates a bias in favor of Filipino goods,
services, labor and enterprises, it also recognizes the need for business exchange with the rest of the world and
limits protection of Filipino enterprises only against foreign competition and trade practices that are unfair.

Section 16. Corporations


NDC v. PVB, 192 SCRA 257 (1990
Facts:
The Agrix Marketing, Inc. (AGRIX) had executed in favor of respondent Philippine Veterans Bank a real estate mortgage
dated July 7, 1978, over three parcels of land located in Los Baos, Laguna. During the existence of the mortgage,
AGRIX went bankrupt.Then Predisdential Decree No. 1717 was issued by President Marcos for the purpose of salvaging
the agrix companies. It ordered the rehabilitation of the Agrix Group of Companies to be administered by the National
Development Company. The law outlined the procedure for filing claims against the Agrix companies and created a
Claims Committee to process these claims. Especially relevant to this case is the Sec. 4(1) of P.D. No. 1717 which
provides that "all mortgages and other liens presently attaching to any of the assets of the dissolved corporations are
hereby extinguished." The private respondent filed a claim with the AGRIX Claims Committee for the payment of its loan
credit. Then the New Agrix, Inc. and the National Development Company invoking Sec. 4 (1) of the decree, filed a petition
with the Regional Trial Court of Calamba, Laguna, for the cancellation of the mortgage lien in favor of the private
respondent.
Issue:
Whether or not the P.D. No. 1717 is unconstitutional
Held:
P.D. No. 1717 is an invalid exercise of the police power, not being in conformity with the traditional requirements of a
lawful subject and a lawful method. The extinction of the mortgage and other liens and of the interest and other charges
pertaining to the legitimate creditors of AGRIX constitutes taking without due process of law, and this is compounded by
the reduction of the secured creditors to the category of unsecured creditors in violation of the equal protection clause. It
also interferes with purely private agreements without any demonstrated connection with the public interest, thus, it
constitute an impairment of the obligation of the contract.
Note:
P.D. 1717 was passed to rehabilitate the Agrix Group of Companies which was a corporation neither owned nor
controlled by the government. As part of the rehabilitation process, the Agrix Group was dissolved by the decree
and the decree created a New Agrix, Inc., likewise neither owned nor controlled by the government. It violates
Article XIV, Section 4 of the 1973 Constitution now modified as Article XII, Section 16.
Boy Scouts of the Philippines v. COA, GR 177131, 07 June 2011.
Facts:
Petitioners filed a petition for prohibition and TRO against the issued resolution of COA, with the subject Defining the
Commissions policy with respect to the audit of the Boy Scouts of the Philippines. The resolution stated that COA would
conduct an annual financial audit of the Boy Scouts of the Philippines in accordance with generally accepted auditing
standards since BSP was created as a public corporation and is regarded as a government instrumentality under the
1987Administrative Code.
The BSP contends that it is not a government-owned or controlled corporation; neither is it an instrumentality, agency, or
subdivision of the government. Thus, they are not subject to COAs audit jurisdiction.
Issue:
Whether or not the BSP falls under the COAs audit jurisdiction
Held:
The SC ruled out that COA is a public corporation and it is subject to COAs audit jurisdiction. Not all corporations, which
are not government owned or controlled, are ipso facto to be considered private corporations as there exists another
distinct class of corporations or chartered institutions which are otherwise known as public corporations. These
corporations are treated by law as agencies or instrumentalities of the government which are not subject to the tests of
ownership or control and economic viability but to different criteria relating to their public purposes/interests or
constitutional policies and objectives and their administrative relationship to the government or any of its Departments or
Offices.

Dissent of Justice Carpio:


The dissenting opinion of Associate Justice Antonio T. Carpio, citing a line of cases, insists that the Constitution
recognizes only two classes of corporations: private corporations under a general law, and government-owned
or controlled corporations created by special charters.
Note:
Section 16, Article XII should not be construed so as to prohibit Congress from creating public corporation. In
fact, Congress has enacted numerous laws creating public corporations or government agencies or
instrumentalities vested with corporate powers. Moreover, Section 16, Article XII, which relates to National
Economy and Patrimony, could not have tied the hands of Congress in creating public corporation to serve any
of the constitutional policies or objective.
Section 17. Temporary Take-Over
Agan v. PIATCO, 420 SCRA 575
Facts:
On October 5, 1994, Asias Emerging Dragon Corp. (AEDC) submitted an unsolicited proposal to the Philippine
Government through the Department of Transportation and Communication (DOTC) and Manila International Airport
Authority (MIAA) for the construction and development of the NAIA IPT III under a build-operate-and-transfer arrangement
pursuant to R.A. No. 6957, as amended by R.A. No. 7718 (BOT Law). In accordance with the BOT Law and its
Implementing Rules and Regulations, the DOTC/MIAA invited the public for submission of competitive and comparative
proposals to the unsolicited proposal of AEDC. A consortium composed of the Peoples Air Cargo and Warehousing Co.,
Inc. (Paircargo), Phil. Air and Grounds Services, Inc. (PAGS) and Security Bank Corp. (Security Bank) (collectively,
Paircargo Consortium), submitted their competitive proposal to the Prequalification Bids and Awards Committee (PBAC).
The Paircargo Consortium submitted a bid superior to the proposal of AEDC. The DOTC issued the notice of award for the
NAIA IPT III project to the Paircargo Consortium, which later organized into Philippine International Air Terminals Co.
(PIATCO). The 1997 Concession Agreement was superseded by the Amended and Restated Concession Agreement
(ARCA) containing certain revisions and modifications from the original contract. A series of supplemental agreements
was also entered into by the Government and PIATCO. On September 17, 2002, various petitions were filed before this
Court to annul the 1997 Concession Agreement, the ARCA and the Supplements and to prohibit the public respondents
DOTC and MIAA from implementing them.
Issue:
Whether or not PIATCO is entitled to just compensation for the temporary takeover of the terminal of the Philippine
Government
Held:
The motion for reconsideration of the respondents (PIATCO) was denied by the Court. SC stated that PIATCO was
erroneous in invoking the principle of eminent domain whereas it is the police power of the state which is involved in this
case. Section 17, Article XII of the 1987 Constitution grants the State in times of national emergency the right to
temporarily take over the operation of any business affected with public interest. This right is an exercise of police power
which is one of the inherent powers of the State.
Police power has been defined as the "state authority to enact legislation that may interfere with personal liberty or
property in order to promote the general welfare. It consists of two essential elements. First, it is an imposition of restraint
upon liberty or property. Second, the power is exercised for the benefit of the common good. It is and still is the most
essential, insistent, and illimitable of the States powers. Unlike the power of eminent domain, police power is exercised
without provision for just compensation for its paramount consideration is public welfare.
Section 5.10(c), Article V of the ARCA provides that respondent PIATCO shall be entitled to reasonable compensation for
the duration of the temporary takeover by Government of the Republic of the Philippines, which compensation shall take
into account the reasonable cost for the use of the Terminal and/or Terminal Complex. It clearly obligates the government
in the exercise of its police power to compensate respondent PIATCO and this obligation is offensive to the Constitution.
Police power cannot be diminished, let alone defeated by any contract for its paramount consideration is public welfare
and interest.
Note:

Public interest on the occasion of a national emergency is the primary consideration when the government
decides to temporarily take over or direct the operation of a public utility or a business affected with public
interest. The nature and extent of the emergency is the measure of the duration of the takeover as well as the
terms thereof. It is the State that prescribes such reasonable terms which will guide the implementation of the
temporary takeover as dictated by the exigencies of the time. This power of the State cannot be negated by any
party nor should its exercise obligate the State to compensate for the value of the property.
David v Arroyo 2006
On February 24, 2006, President Arroyo issued PP No. 1017 declaring a state of emergency. On the same day, PGMA
issued G.O. No. 5 implementing PP1017, directing the members of the AFP and PNP "to immediately carry out the
necessary and appropriate actions and measures to suppress and prevent acts of terrorism and lawless violence." Third
provision of the operative portion of PP1017 reads: as provided in Section 17, Article XII of the Constitution do hereby
declare a State of National Emergency. David et al questioned its constitutionality on 3 grounds (3rd ground/3rd provision
ang related sa sec 17)
Section 17, Article XII must be understood as an aspect of the emergency powers clause. The taking over of private
business affected with public interest is just another facet of the emergency powers generally reposed upon Congress.
Now, whether or not the President may exercise such power is dependent on whether Congress may delegate it to him
pursuant to a law prescribing the reasonable terms thereof. The Proclamation does not authorize her during the
emergency to temporarily take over or direct the operation of any privately owned public utility or business affected with
public interest without authority from Congress.
Main point: In short, the President has no absolute authority to exercise all the powers of the State under Section 17,
Article VII in the absence of an emergency powers act passed by Congress.
Sec 18 nationalization
Republic v PLDT 1968
Public petitioner commenced a suit against private respondent praying for the right of the Bureau of Telecommunications
to demand interconnection between the Government Telephone System and that of PLDT, so that the Government
Telephone System could make use of the lines and facilities of the PLDT. Pldt contends that it cannot be compelled to
enter into a contract where no agreement is had between them.
Whether or not interconnection between PLDT and the Government Telephone System can be a valid object for
expropriation.
Yes, in the exercise of the sovereign power of eminent domain, the Republic may require the telephone company to
permit interconnection as the needs of the government service may require, subject to the payment of just compensation.
The use of lines and services to allow inter-service connection between the both telephone systems, through
expropriation is based on Sec 18 of article xii.
Pldt v Ntc 1990 (almost same langnungnasataas)
Private respondent Express Telecommunications Co., Inc. (ETCI) obtained from Congress Republic Act No. 2090 a
franchise to establish radio stations for domestic and transoceanic telecommunications. Petitioner PLDT invoked the prior
operator or protection of investment doctrine in its opposition to ETCIs subsequent application for Certificate of Public
Convenience and Necessity (CPCN). The National Telecommunications Commission (NTC) granted provisional authority
to ETCI subject to the condition that it shall enter into interconnection agreement with PLDT. PLDT elevated the case to
the Supreme Court pointing out ETCIs defective legislative franchise to operate telecommunications system, among
others. ETCI contends that PLDTs special civil action must deal only on issues whether the NTC acted without jurisdiction
of with grave abuse of discretion in granting ETCI the assailed provisional authority.
Issue (related sa sec 18): Whether NTC acted without jurisdiction or with grave abuse of discretion amounting to lack or
excess of jurisdiction in granting provisional authority to ETCI.
No. There was no grave abuse of discretion, tantamount to lack of or excess of jurisdiction, on the part of the National
Telecommunications Commission.The decisive considerations are public need, public interest, and the common good.
Those were the overriding factors which motivated NTC in granting provisional authority to ETCI. It is likewise a State
policy to provide the environment for the emergence of communications structures suitable to the balanced flow of
information into, out of, and across the country. Service in the telephone system is sadly inadequate. Customer demands
are hardly met, whether fixed or mobile. There is a unanimous cry to hasten the development of a modern, efficient,
satisfactory and continuous telecommunications service not only in Metro Manila but throughout the archipelago in the
interest of national welfare. Thus, the state may intervene by virtue of sec 18.
*may part din na related sa sec 19, ilagaykonalangpara mas maimaginenyoyungkaso:
Free competition in the industry may also provide the answer to a much-desired improvement in the quality and delivery of
this type of public utility, to improved technology, fast and handy mobile service, and reduced user dissatisfaction. After all,
neither PLDT nor any other public utility has a constitutional right to a monopoly position in view of the Constitutional

proscription that no franchise certificate or authorization shall be exclusive in character or shall last longer than fifty (50)
years. Additionally, the State is empowered to decide whether public interest demands that monopolies be regulated or
prohibited (1987 Constitution. Article XII, Section 19).
Pldt v eastern telecom 1992 (pareholang din nungnasataasna case, grabe. Naibalang e yungkinasuhanngpldttsaka
decision)
The petition seeks to set aside and annul the decision of the National Telecommunications Commission (NTC) signed by
former NTC Commissioner Jose Luis Alcuaz dated November 14, 1989 as well as the order dated July 16, 1990 of the
NTC En Banc.
Eastern Telecommunications Philippines, Inc. filed with the NTC an application for a Certificate of Public Convenience and
Necessity (CPCN) to construct, maintain and operate an International Digital Gateway Facility (IDGF). In its application,
Eastern alleged that it is a domestic corporation and that it has the "franchise to land, construct, maintain and operate
telecommunications systems by cable, or any other means now known to science or which in the future may be
developed for the reception and transmission of messages to and between any point in the Philippines to point exterior
thereto . . . (R.A. 5002)
On June 23, 1988, petitioner Philippine Long Distance Telephone Company (PLDT) filed an "Opposition to Main
Application and to Prayer for Provisional Authority" dated June 20, 1988 to NTC against Eastern Telecom.
On November 10, 1989, the NTC through Commissioner Jose Luis A. Alcuaz rendered a decision granting Eastern's
application. But on November 15, 1989, the NTC received a latter dated November 13, 1989 from the Office of the
President addressed to Commissioner Alcuaz stating that he had been replaced as head of NTC. President Aquino stated,
that she was designating Undersecretary of Transportation Josefina T. Lichauco as acting Commissioner of the NTC.
PLDT then filed a "Motion To Declare So-Called Alcuaz Decision Void/Inexistent" as well as a motion for reconsideration of
that decision. After further pleadings and counter pleadings, the NTC by an order dated July 16, 1990 En Banc denied the
PLDT motions assailing the validity of the said decision and confirmed the November 14, 1989 decision signed by
Commissioner Alcuaz.
whether or not the rendition of the assailed NTC decision and order was attended by grave abuse of discretion amounting
to lack of jurisdiction on the part of the NTC
Yes, there is merit in the petition.
In the absence of legislative authority to operate a telephone system, it was therefore grave abuse of discretion for NTC to
order Eastern to interconnect with PLDT. Furthermore, Eastern has no telephone system in the Philippines or anywhere
else in the world. It does not have a single telephone subscriber. Yet, it has been authorized by NTC to open a telephone
exchange connecting PLDT subscribers with telephone systems around the world. Legally, Eastern has no franchise. It
must first go to Congress. An additional point, however, is that there is no urgent public need for the establishment of
another IGF (international gateway facility). PLDT puts forth the argument that at present it has five (5) existing gateway
facilities to meet the needs of the people but NTC nevertheless approved Eastern's application in the event that PLDT's
operations might be suspended i.e. labor dispute, fortuitous event, technical causes etc.
(pinagkaibanetosapldtsataas, yung decision dito e in favor sapldt. Peroparang mas related kasesyasa sec 19.
Maiinfringemasyadoyung rights ngpldt since yungpagapprovengntcsa cert of franchise ng eastern e
parangmakiparasitesa investment namatagalnangnaestablishngpldt. Di pwedeiassertyung idea sataasna applicable
ditoyungsa free competition. Yes, walang right angpldtsa monopoly, pero di pwedemagdecideyungntc in favor sa eastern
dahillangsa idea nayun (sec 19 monopolies). Di masyadongnaconsiderngntcyung situation. And walanamankaseng urgent
need for another igf, pagpinayagansila, may possibility nalumakilanglaloyung fees nanashoshoulderngmgatao. Sguro
main idea in relation sa 18 dito e since walang need for another igf(something for interest of national welfare), di
dapatmagdecideyung state (represented by ntc in this case). Mguloyungkaso e peroyankasepagkakaintindiko)
Sec 19: monopolies and combinations
Emergency regulatory board v CA 2001
On June 30,1983, Shell filed with the quondam Bureau of Energy Utilization (BEU) an application for authority to relocate
its Shell Service Station at Tambo, Paraaque, Metro Manila, to Imelda Marcos Avenue of the same municipality. The
application, was initially rejected by the BEU because Shell's old site had been closed for five (5) years such that the
relocation of the same to a new site would amount to a new construction of a gasoline outlet, which construction was then
the subject of a moratorium. Subsequently, however, BEU relaxed its position and gave due course to the application. But
on June 3, 1986, the BEU rendered a decision denying Shell's application on a finding that there was "no necessity for an

additional petroleum products retail outlet in Imelda Marcos Avenue, Paraaque." Dissatisfied, Shell appealed to the
Office of Energy Affairs (OEA).
Meanwhile, on May 8, 1987, Executive Order No. 172 was issued creating the Energy Regulatory Board (ERB) and
transferring to it the regulatory and adjudicatory functions of the BEU.
On May 9, 1988, the OEA rendered a decision denying the appeal of Shell and affirming the BEU decision. Shell moved
for reconsideration and prayed for a new hearing or the remand of the case for further proceedings. In a supplement to
said motion, Shell submitted a new feasibility study to justify its application. However, on September 17, 1991, the ERB
rendered a Decision allowing Shell to establish the service station in Benigno Aquino, Jr. Avenue.
Garcia v ES 1999
As a result of the Tatad decision declaring RA 8180 entitled An Act Deregulating the Downstream Oil Industry and For
Other Purposes as unconstitutional, Congress enacted Republic Act No. 8479, a new deregulation law without the
offending provisions of the earlier law. Petitioner Enrique T. Garcia, a member of Congress, has now brought this petition
seeking to declare Section 19 thereof, which sets the time of full deregulation, unconstitutional. After failing in his attempts
to have Congress incorporate in the law the economic theory he espouses, petitioner now asks the Court, in the name of
upholding the Constitution, to undo a violation which he claims Congress has committed.
The petition states that if the constitutional mandate against monopolies and combinations in restraint of trade is to be
obeyed, there should be indefinite and open-ended price controls on gasoline and other oil products for as long as
necessary. This will allegedly prevent the Big 3 --- Shell, Caltex and Petron --- from price-fixing and overpricing. Petitioner
calls the indefinite retention of price controls as partial deregulation.
Whether or not sec 19 of RA 8479 is unconstitutional for being pro-oligopoly, anti-competition and anti-people, in gross
violation of sec 19 art xii
The most important part of deregulation is freedom from price control. Indeed, the free play of market forces through
deregulation and when to implement it represent one option to solve the problems of the oil-consuming public. There are
other considerations which may be taken into account such as the reduction of taxes on oil products, the reinstitution of an
Oil Price Stabilization Fund, the choice between government subsidies taken from the regular taxpaying public on one
hand and the increased costs being shouldered only by users of oil products on the other, and most important, the
immediate repeal of the oil deregulation law as wrong policy.
R.A. 8479, the present deregulation law, was enacted to implement Article XII, Section 19 of the Constitution which
provides:
The State shall regulate or prohibit monopolies when the public interest so requires. No combinations in restraint of trade
or unfair competition shall be allowed.
This is so because the Government believes that deregulation will eventually prevent monopoly. Where two or three or a
few companies act in concert to control market prices and resultant profits, the monopoly is called an oligopoly or cartel.
And before the start of deregulation, the three private respondents controlled the entire oil industry in the Philippines.
The Court's ruling in Tatad is categorical that the Constitution's Article XII, Section 19, is anti-trust in history and spirit. It
espouses competition. We have stated that only competition which is fair can release the creative forces of the market.
We ruled that the principle which underlies the constitutional provision (sec 19) is competition.

Tatad v Sec of Energy 1997


The petitioner challenges the constitutionality of RA No. 8180 entitled An Act Deregulating the Downstream Oil Industry
and For Other Purposes. The deregulation process has two phases: (a) the transition phase (Aug. 12, 1996) and the (b)
full deregulation phase (Feb. 8, 1997 through EO No. 372). It was stated that the Act does not provide any specific
standard to determine when the prices of crude oil in the world market are considered to be declining nor when the
exchange rate of the peso to the US dollar is considered stable.
R.A. 8180 was struck down as invalid because three key provisions intended to promote free competition were shown to
achieve the opposite result. More specifically, this Court ruled that its provisions on tariff differential, stocking of
inventories, and predatory pricing inhibit fair competition, encourage monopolistic power, and interfere with the free
interaction of the market forces.
While R.A. 8180 contained a separability clause, it was declared unconstitutional in its entirety since the three (3)
offending provisions so permeated the law that they were so intimately the esse of the law. Thus, the whole statute had to
be invalidated. Thus, EO 372 is considered void.
Eastern Assurance v LTFRB
The operation of monopolies is not totally banned by the Constitution. However, the State shall regulate them when public
interest so requires. In the present case, the two consortia of insurance companies that have been authorized to issue
passenger insurance policies are adequately regulated by the Land Transportation Franchising and Regulatory Board
(LTFRB) to protect the riding public. While individual insurance companies may somehow be adversely affected by this

scheme, the paramount public interest involved must be upheld. In any event, all legitimate insurance companies are
allowed to become members of the consortia. Thus, there is no restraint of trade or unfair competition involved.
The operation of monopolies is not totally banned by the Constitution. However, the State shall regulate them when public
interest so requires. In the present case, the two consortia of insurance companies that have been authorized to issue
passenger insurance policies are adequately regulated by the Land Transportation Franchising and Regulatory Board
(LTFRB) to protect the riding public. While individual insurance companies may somehow be adversely affected by this
scheme, the paramount public interest involved must be upheld. In any event, all legitimate insurance companies are
allowed to become members of the consortia. Thus, there is no restraint of trade or unfair competition involved.
Both Shell and ERB elevated the matter to this Court by way of these petitions, which were ordered consolidated by the
Court in a Resolution dated July 25,1994.
Whether or not COURT OF APPEALS GRAVELY ERRED IN FINDING THAT THE PROPOSED SERVICE STATION OF
PETITIONER WOULD POSE RUINOUS COMPETITION TO PRIVATE RESPONDENT'S SERVICE STATION BASED
MAINLY ON EVIDENCE SUBMITTED FOR THE FIRST TIME WITH THE SAID COURT AND WITHOUT CONDUCTING A
HEARING THEREON..
The policy of the government in this regard has been to allow a free interplay of market forces with minimal government
supervision. The purpose of governing legislation is to liberalize the downstream oil industry in order to ensure a truly
competitive market under a regime of fair prices, adequate and continuous supply, environmentally clean and high-quality
petroleum products. Indeed, exclusivity of any franchise has not been favored by the Court, which is keen on promoting
free competition and the development of a free market consistent with the legislative policy of deregulation as an answer
to the problems of the oil industry.
While it is probable that the operation of the proposed Shell outlet may, to a certain extent, affect PDSC's business,
private respondent nevertheless failed to show that its business would not have sufficient profit to have a fair return of its
investment. The mere possibility of reduction in the earnings of a business is not sufficient to prove ruinous competition.
Indeed
In order that the opposition based on ruinous competition may prosper, it must be shown that the opponent would be
deprived of fair profits on the capital invested in its business. The mere possibility of reduction in the earnings of a
business is not sufficient to prove ruinous competition. It must be shown that the business would not have sufficient gains
to pay a fair rate of interest on its capital investment. Mere allegations by the oppositor that its business would be ruined
by the establishment of the ice plants proposed by the applicants are not sufficient to warrant this Court to revoke the
order of the Public Service Commission.
All told, a climate of fear and pessimism generated by unsubstantiated claims of ruinous competition already rejected in
the past should not be made to retard free competition, consistently with legislative policy of deregulating and liberalizing
the oil industry to ensure a truly competitive market under a regime of fair prices, adequate and continuous supply,
environmentally clean and high-quality petroleum products.
Avon v Luna
Avon Cosmetics, Inc. (Avon), acquired and took over the management and operations of Beautifont, Inc. in 1978.
Nonetheless, respondent Luna continued working for said successor company. Thereafter, petitioner Avon and
respondent Luna entered into an agreement, entitled Supervisors Agreement and by virtue of the execution of the
aforementioned Supervisors Agreement, respondent Luna became part of the independent sales force of petitioner
Avon._Luna became a Group Franchise Director of Sandre Philippines, Inc. concurrently with being a Group Supervisor of
petitioner Avon and began selling and/or promoting Sandre products to other Avon employees and friends. In 1988, she
requested a law firm to render a legal opinion as to the legal consequence of the Supervisors Agreement she executed
with petitioner Avon. In response to her query, a lawyer of the firm opined that the Supervisors Agreement was "contrary
to law and public policy".
Wanting to share the legal opinion she obtained from her legal counsel, respondent Luna wrote a letter to her colleagues
and attached mimeographed copies of the opinion and then circulated them. Afterwards, Petitioner Avon, through its
President and General Manager, Jose Mari Franco, notified respondent Luna of the termination or cancellation of her
Supervisors Agreement with petitioner Avon. Aggrieved, respondent Luna filed a complaint for damages and the RTC
rendered judgment in favor of respondent Luna which was assailed by the Court of appeals.
Whether or not the Supervisors Agreement is valid and not against public policy
_No. SEction 5 of the Agreement is called an exclusivity clause. This exclusivity clause is more often the subject of critical
scrutiny when it is perceived to collide with the Constitutional proscription against "reasonable restraint of trade or
occupation" which is sec 19 of art xii of the constitution.
First off, restraint of trade or occupation embraces acts, contracts, agreements or combinations which restrict competition
or obstruct due course of trade. From the wordings of the Constitution, truly then, what is brought about to lay the test on
whether a given agreement constitutes an unlawful machination or combination in restraint of trade is whether under the
particular circumstances of the case and the nature of the particular contract involved, such contract is, or is not, against
public interest.
Thus, restrictions upon trade may be upheld when not contrary to public welfare and not greater than is necessary to
afford a fair and reasonable protection to the party in whose favor it is imposed. Even contracts which prohibit an

employee from engaging in business in competition with the employer are not necessarily void for being in restraint of
trade.
In sum, contracts requiring exclusivity are not per se void. Each contract must be viewed vis--vis all the circumstances
surrounding such agreement in deciding whether a restrictive practice should be prohibited as imposing an unreasonable
restraint on competition.
The main objection of exclusive dealing (reflected in the Agreement's exclusivity clause) is its tendency to foreclose
existing competitors or new entrants from competition in the covered portion of the relevant market during the term of the
agreement. Only those arrangements whose probable effect is to foreclose competition in a substantial share of the line of
commerce affected can be considered as void for being against public policy._Applying the preceding principles to the
case at bar, there is nothing invalid or contrary to public policy either in the objectives sought to be attained by paragraph
5, the exclusivity clause, in prohibiting respondent Luna, and all other Avon supervisors, from selling products other than
those manufactured by petitioner Avon.
Article xvi.
Sec 3. Immunity from suit
Jeffrey Liang is an economist for ADB who was charged by the Metropolitan Trial Court of Mandaluyong city for allegedly
uttering defamatory words against her fellow worker with two counts of grave oral defamation. The MTC judge then
received an office of protocol from the Department of Foreign Affairs, stating that petitioner is covered by immunity from
legal process under section 45 of the agreement between ADB and the government. Upon receipt of the protocol the MTC
Judge dismissed the two criminal cases against Liang. Prosecution filed writ of mandamus and certiorari and ordered the
MTC to enforce the warrant of arrest.__Whether or not the petitioner is covered by immunity under the agreement.__He is
not covered by immunity because the commission of a crime is not part of the performance of his official duty. Courts
cannot blindly adhere and take on its face the communication from the DFA that a certain person is covered by immunity.
Slandering a person is not covered by the agreement because our laws do not allow the commission of a crime such as
defamation in the name of official duty. Under the Vienna convention on Diplomatic Relations, commission of a crime is
not part of official duty.
Article 16. Sec. 3 (The state cannot be sued without its consent)
Calubvs Court of Appeals
Facts:
In the exercise of their duty to uphold the law, DENR officer Calub and other department authorities, apprehended
two motor vehicles loading lumber with corresponding amount. According it was a violation of the Revised Forestry Code.
The private respondents seek a writ of replevin in the RTC and it was granted. The Public Officers petitioners appealed in
the Court of appeals to deny the said court order, and it is a violation of the constitutional provision, that the state cannot
be sued without its consent.
Issue: Whether or not the complaint for the recovery of possession of impounded vehicles, with an application for replevin,
is a suit against the State?
Held:
The court held that it is a well-established doctrine that the State may not be sued without its consent.
Andasuitagainstapublicofficerforhisofficialactsis,ineffect,asuitagainst theStateifitspurpose is to hold the State ultimately
liable. However,the protection afforded to public officers by this doctrine generally applies only to activities within the
scope of their authority in good faith and without willfulness, malice or corruption. Inthepresentcase,they performed the
acts for which the petitioners are being called to account in the discharge of their official duties. The acts inquestion are
clearly official in nature. Inimplementingandenforcing Sections 78-A and 89 of the Forestry Code through the seizure
carried out, petitioners were performing their duties and functions as officers of the DENR, and did so within the limits of
their authority. There was no malice or bad faith on his or her part. Hence, a suit against the petitioners who represent the
DENR is a suit against the State. It cannot prosper without the States consent.

Lansangvs Court of Appeals


Facts:
Private respondent, Iglesias is the president of the General Assembly of the Blind Inc. (GABI) that was given a
verbal contract of lease by National Parks Development Committee, a government initiated civic group for the for the
development of National Parks. Under the new administration of Chairman Lansang, he terminated the verbal agreement.

GABI got a TRO from the RTC, but upon its expiration they were ejected as well. The Court of Appeals grants the petition
of GABI against NPDC chairman and all other persons involved in the complaint they made and petitioner were held liable
to indemnify the private respondent. The
Issue: Whether or not the case filed against the petitioner is in effect a suit against the state, which cannot prosper
without its consent?
Held:
The doctrine of state immunity from suit applies to complaints filed against public officials for acts done in the
performance of their duties. The rule is that the suit must be regarded as one against the state where satisfaction of the
judgment against the public official concerned will require the state itself to perform a positive act, such as appropriation of
the amount necessary to pay the damages awarded to the plaintiff.16The rule does not apply where the public official is
charged in his official capacity for acts that are unlawful and injurious to the rights of others.17 Public officials are not
exempt, in their personal capacity, from liability arising from acts committed in bad faith.18Neither does it apply where the
public official is clearly being sued not in his official capacity but in his personal capacity, although the acts complained of
may have been committed while he occupied a public position.
We are convinced that petitioner is being sued not in his capacity as NPDC chairman but in his personal capacity. The
complaint filed by private respondents in the RTC merely identified petitioner as chairman of the NPDC, but did not
categorically state that he is being sued in that capacity.19 Also, it is evident from paragraph 4 of said complaint that
petitioner was sued allegedly for having personal motives in ordering the ejectment of GABI from Rizal Park.
Mancenidovs Court of Appeals
Facts:
The petitioners at bar filed an action for mandamus and damages in the RTC of Camarines Norte against the
Provincial Board of Camarines Norte, the school board, Provincial Governor, Provincial Treasurer, and Provincial Auditor
to pay the teachers claim for unpaid salary increases. The private respondents submitted a notice of appeal through their
counsel Atty. Jose Lapak. Petitioners contend that Atty. Jose Lapak could not represent the respondents Provincial
Treasurer and Provincial School Board, because both are instrumentalities of the National Government and may be
represented only by the Office of the Solicitor General pursuant to Section 35, Chapter 12, Title 3, Book 4 of the
Administrative Code of 1987
Issue: Whether or not court of appeals erred in recognizing a private counsel as the representative of municipal officials
sued in their official capacities?
Held:
The court denied the petition and affirmed the decision of the Court of Appeals.
The Court has previously ruled on the representation of a local government unit by a private attorney. In Albuera
v. Torres, 102 Phil. 211 (1957), the court approved the representation by private counsel of a provincial governor sued in
his official capacity, where the complaint contained other allegations and a prayer for moral damages, which, if due from
the defendants, must be satisfied by them in their private capacity.
The present case had its origins in Civil Case No. 5864 filed before the RTC of Camarines Norte, Branch 38, for
mandamus and damages. Notwithstanding the fact that the trial court granted mandamus, petitioners appealed to the
Court of Appeals since the trial court did not award damages. In view of the damages sought which, if granted, could
result in personal liability, the herein respondentscould not be deemed to have been improperly represented by private
counsel. No error may thus be attributed to the appellate court when it recognized the right of respondents to be
represented by private counsel.
Shell Philippines vsJalos
Facts:
The Shell Philippines Exploration entered into Service Contract 38 for the exploration and extraction of petroleum
in northwestern Palawan. They have discovered naturals gas over the Camago-Malampaya area by which, shell started to
construct pipeline for its extraction. The respondent at bar filed a petition alleging that the construction made by Shell
causes damage to their daily means of living which is fishing. Shell filed for the injunction of the complaint and contested
that the trial court has no jurisdiction of the case and they also claimed that it could not be sued pursuant to the doctrine of
state immunity without the States consent. Shell said that under Service Contract 38, it served merely as an agent of the
Philippine government in the development of the Malampaya gas reserves.

Issue: Whether or not Shell Philippines cannot be sued merely because it is an agent of the state, and it is being barred
by doctrine of state immunity?
Held:
The court said that, Shell is not an agent of the Philippine government, but a provider of services, technology and
financingfor the Malampaya Natural Gas Project. It is not immune from suit and may be sued for claims even without the
States consent. Notably, the Philippine government itself recognized that Shell could be sued in relation to the project.
This is evident in the stipulations agreed upon by the parties under Service Contract 38.

China National Machinery and Equipment Corp. (CNMEG)vs Hon. Cesar Santamaria
Facts:
The Northrail (North Luzon Railway Corporation) and the petitioner at bar executed a Contract Agreement for the
construction of the North Railway project. The CNMEG was designated as the Prime Contractor of the Northrail project.
After the contract agreement for the construction of the first phase of the project, the respondents at bar file an annulment
of contract and injunction because of constitutional, administrative, procurement reform act violations. The CNMEG filed a
motion to dismiss alleging that they are agent of Chinese government and immune from suit.
Issue:Whether or not, CNMEG is immune from suit in pursuant to Article XVI Sec. 3?
Held:
The court denied the petition and states that CNMEG is not entitled to immunity from suit. By the terms of the
Contract Agreement, Northrail is a government-owned or -controlled corporation, while CNMEG is a corporation duly
organized and created under the laws of the Peoples Republic of China. Thus, both Northrail and CNMEG entered into
the Contract Agreement as entities with personalities distinct and separate from the Philippine and Chinese governments,
respectively.
Neither can it be said that CNMEG acted as agent of the Chinese government. As previously discussed, the fact
that Chinese Amb. Wang, in his letter dated 1 October 2003,described CNMEG as a state corporation and declared its
designation, as the Primary Contractor in the Northrail Project did not mean it was to perform sovereign functions on
behalf of China. That label was only descriptive of its nature as a state-owned corporation, and did not preclude it from
engaging in purely commercial or proprietary ventures.

Santos vs Santos
Facts:
The petitioner at bar Teodora Santos and her nieces filed a complaint against Leoncio Santos, respondent and
brother of Teodora, alleging for the partition of the undivided portion of land inherited from their father. Leoncio collect
rentals from the alleged parcel of land. He refused for the partition of his undivided inheritance because it was already
sold to the Civil Aeronautics Administration (CAA), National Airport Corporation before. The petitioners prayed for the
partition of the land and declare the selling of that land to CAA as null and void. The CAA moved to dismiss the complaint
for insufficiency and lack of jurisdiction invoking that the suit was against the state, which could not be brought without its
consent.
Issue:Whether or not CAA is entitled for immunity from suit?
Held:
The order appealed from dismissing the complaint as to the Civil Aeronautics Administration is reversed and the
case remanded to the lower court for further proceedings in accordance with law.
In National Airports Corporation vs. Teodoro, the court held that the Civil Aeronautics Administration may be sued
and that the principle of state immunity from suit does not apply to it.The Civil Aeronautics Administration, even if it is not a
juridical entity, cannot legally prevent a party or parties from enforcing their propriety rights under the cloak or shield of

lack of juridical personality, because it took over all the powers and assumed all the obligations of the defunct corporation,
which had entered into the contract in question.
Holy See v. Rosario
FACTS: Petitioner filed for certiorari over Civil Case No. 90-183 and to invoke its immunity to waiver the complaint for the
civil case.
This petition is regarding a parcel of land that was neighboring two other lots. The land was donated by the
Archdiocese of Manila to the Papal Nuncio, (which represents the Holy See that exercises sovereignty in Rome) for its
residence.
The lots were sold to Ramon Licup, including the lot for the Papal Nuncio since the squatters would not want to
evacuate the lands, and the land isnt suitable anymore for the construction of the residence of the Papal Nuncio.
Ramon Licup gave his rights over the lots of Starbright Sales Enterprises, Inc. Starbright filed a complaint for the
annulment of the sale of the 3 parcels of land, and damages against the petitioner.
In June 20, 1991, the trial court denied the motion to dismiss after finding that petitioner shed off its foreign
immunity by entering into the business contract in question. The petitioners action was declared as an act of jure
gestionis.
ISSUE: WON petitioner entered into a business transaction
WON petitioner can invoke the doctrine of immunity
HELD: Petitioners merely wanted to dispose the parcel of land donated by the Archdiocese because the squatters dont
want to leave the land and the said land cant be used for its main purpose. Petitioners act was not undertaken for gain or
profit. Therefore, the act was jure imperii.
And since the land was made not for commercial purpose, but for the use of petitioner to construct thereon the official
place of residence of the Papal Nuncio, the decision to transfer the property and the subsequent disposal thereof are
likewise clothed with a governmental character. Again, petitioner did not sell the lot for profit or gain. It merely wanted to
dispose of the same because the squatters living thereon made it almost impossible for petitioner to use it for the purpose
of the donation thus Holy See may properly invoke immunity.

DFA v. NLRC
FACTS: On 27 January 1993, private respondent Magnayi filed an illegal dismissal case against Asian Development
Bank. ADB and the DFA notified respondent Labor Arbiter that the ADB, as well as its President and Officers, were
covered by an immunity from legal process except for borrowings, guaranties or the sale of securities pursuant to Article
50(1) and Article 55 of the Agreement Establishing the Asian Development Bank (the "Charter") in relation to Section 5
and Section 44 of the Agreement Between The Bank and The Government Of The Philippines Regarding The Bank's
Headquarters (the "Headquarters Agreement").
The Labor Arbiter took cognizance of the complaint on the impression that the ADB had waived its diplomatic immunity
from suit and, in time, rendered a decision in favor Magnayi. The ADB did not appeal the decision. Instead, on 03
November 1993, the DFA referred the matter to the NLRC; in its referral, the DFA sought a "formal vacation of the void
judgment." When DFA failed to obtain a favorable decision from the NLRC, it filed a petition for certiorari.
ISSUE: WON ADB is immune from suit
WON by entering into service contracts with different private companies, ADB has descended to the level of an ordinary
party to a commercial transaction giving rise to a waiver of its immunity from suit
HELD: Under the Charter and Headquarters Agreement, the ADB enjoys immunity from legal process of every form,
except in the specified cases of borrowing and guarantee operations, as well as the purchase, sale and underwriting of
securities. The Banks officers, on their part, enjoy immunity in respect of all acts performed by them in their official
capacity. The Charter and the Headquarters Agreement granting these immunities and privileges are treaty covenants and
commitments voluntarily assumed by the Philippine government which must be respected. Being an international
organization that has been extended a diplomatic status, the ADB is independent of the municipal law. The obvious
reason for this is that the subjection of such an organization to the authority of the local courts would afford a convenient

medium thru which the host government may interfere in their operations or even influence or control its policies and
decisions of the organization; besides, such subjection to local jurisdiction would impair the capacity of such body to
discharge its responsibilities impartially on behalf of its member-states."
No, the ADB didn't descend to the level of an ordinary party to a commercial transaction, which should have constituted a
waiver of its immunity from suit, by entering into service contracts with different private companies.
DOCTRINE: Certainly, the mere entering into a contract by a foreign state with a private party cannot be the ultimate test.
Such an act can only be the start of the inquiry. The logical question is whether the foreign state is engaged in the activity
in the regular course of business. If the foreign state is not engaged regularly in a business or trade, the particular act or
transaction must then be tested by its nature. If the act is in pursuit of a sovereign activity, or an incident thereof, then it is
an act jure imperii, especially when it is not undertaken for gain or profit.

Santos v. Santos
FACTS: An undivided parcel of land situated in the Municipality of Las Pias was owned by the petitioners and
the respondent in the proportion of 1/7 undivided share for Teodora Santos and 1/14 undivided share each for Josefina
Santos and Emiliana Santos and 5/7 undivided share for Leoncio Santos.
Petitioners complained that from 1945 to 1949 Leoncio Santos collected from the Army of the
United States of America rentals for the use and occupation of a parcel of land and later sold the lot the Administrator of
the Civil Aeronautics Administration on or about 13 May 1949.
Petitioners demand for the a c c o u n t i n g o f t h e p a y m e n t s f o r t h e r e n t a l s o f t h e l o t a n d t o
g i v e t o t h e p o r t i o n o f t h e f r u i t s o f t h e r e n t a l s according to their portion of the lot. They also prayed to
restore to their ownership the portions of the land that belongs to them contending that the said contract of
sale is null and void because it is performed w i t h o u t t h e i r c o n s e n t .
A d m i n i s t r a t o r o f t h e C i v i l Aeronautics Administration moved to dismiss the complaint for
lack of jurisdiction and insufficiency of the complaint against him. This motion was granted on the
ground that the Civil Aeronautics Administration not b e i n g a j u r i d i c a l p e r s o n h a s n o c a p a c i t y t o
s u e a n d b e s u e d a n d f o r t h a t r e a s o n i t c a n n o t c o m e u n d e r t h e jurisdiction of the court.
ISSUE: WON the petitioners can sue the Civil Aeronautics Administration who is not a juridical entity
HELD: If the state or its government enters into a contract, through its officers or agents, in furtherance
of a legitimate aim and purpose, whereby mutual benefits accrue and rights and obligations arise there
from, and if the law granting the authority to enter into such contract does not provide for or name the
officer against whom action may be brought in the event of a breach thereof, the state itself may be sued
even without its consent .
The Civil Aeronautics Administration, even if it is not a juridical entity, cannot legally prevent a party from
enforcing their propriety rights under the shield of lack of juridical personality, because it took o v e r
all the powers and assumed all the obligations of the defunct corporation which had
e n t e r e d i n t o t h e contract in question.
DOCTRINE: Consent is implied when the state enters into a private contract unless the contract is merely incidental to the
performance of a governmental function.

Republic v. Feliciano
FACTS: Feliciano was holding property title to which was evidenced by an informacion posesoria. Proclamation No. 90 of
President Magsaysay included it among the properties for subdivision and distribution. Feliciano sued the Republic,
represented by the Land Authority, to recover possession of the land. The state pleaded immunity from suit.
ISSUE: WON the state can be sued for recovery and possession of a parcel of land.
HELD: No. Also, the informacion posesoria had not been shown to have been converted into a record of ownership. It is
nothing more than prima facie evidence of possession.

DOCTRINE: A suit against the state is not permitted, except upon a showing that the state has consented to be sued,
either expressly or by implication through the use of statutory language too plain to be misinterpreted.

Republic of Indonesia v. Vinzons


FACTS: Petitioner, Republic of Indonesia, represented by its Counsellor, Siti Partinah, entered into a Maintenance
Agreement in August 1995 with respondent James Vinzon, sole proprietor of Vinzon Trade and Services. The equipment
covered by the Maintenance Agreement are air conditioning units and was to take effect in a period of four years.
When Indonesian Minister Counsellor Kasim assumed the position of Chief of Administration in March 2000, he
allegedly found Vinzons work and services unsatisfactory and not in compliance with the standards set in the
Maintenance Agreement. Hence, the Indonesian Embassy terminated the agreement in a letter dated August 31, 2000.
Respondent filed a complaint claiming that the aforesaid termination was arbitrary and unlawful. Petitioners filed a Motion
to dismiss, assailing that Republic of Indonesia, as a foreign sovereign State, has sovereign immunity from suit and
cannot be sued as a party-defendant in the Philippines.
ISSUE: WON the Republic of Indonesia can be sued.
HELD: The Republic of Indonesia is acting in pursuit of a sovereign activity when it entered into a contract with the
respondent. The maintenance agreement was entered into by the Republic of Indonesia in the discharge of its
governmental functions. It cannot be deemed to have waived its immunity from suit.
DOCTRINE: The mere entering into a contract by a foreign State with a private party cannot be construed as the ultimate
test of whether or not it is an act jure imperii or jure gestionis. Such act is only the start of the inquiry. If the foreign State is
not engaged regularly in a business or commercial activity, and in this case it has not been shown to be so engaged, the
particular act or transaction must then be tested by its nature. If the act is in pursuit of a sovereign activity, or an incident
thereof, then it is an act jure imperii.
Hence, the existence alone of a paragraph in a contract stating that any legal action arising out of the agreement
shall be settled according to the laws of the Philippines and by a specified court of the Philippines is not necessarily a
waiver of sovereign immunity from suit. The aforesaid provision contains language not necessarily inconsistent with
sovereign immunity. On the other hand, such provision may also be meant to apply where the sovereign party elects to
sue in the local courts, or otherwise waives its immunity by any subsequent act. The applicability of Philippine laws must
be deemed to include Philippine laws in its totality, including the principle recognizing sovereign immunity. Hence, the
proper court may have no proper action, by way of settling the case, except to dismiss it.
Submission by a foreign state to local jurisdiction must be clear and unequivocal. It must be given explicitly or by
necessary implication. We find no such waiver in this case.

Metran v. Paredes
FACTS: A petition was filed in the Court of Industrial Relations titled National Labor Union vs. Metropolitan Transportation
Service (METRAN), wherein petitioner alleged that it was a legitimate labor organization, thirty of whose affiliated
members were working and under the employment of the respondent. That the respondent is a semi-governmental
transportation entity, popularly known as Metran, and after several other allegations concluded with the prayer that its
nine demands at length set forth in said petition be granted.
In behalf of the so-called respondent an oral petition for dismissal of the case was made before the court on
October 22, 1946, on the ground that the respondent belongs to the Republic of the Philippines and as such, it cannot be
sued.
ISSUE: WON Metran can invoke the doctrine of immunity from suit

HELD: The members of the respondent Labor Union themselves are part of the people who have freely formed that
government and participated in that solemn undertaking. In this sense and a very real one it is they are in effect
attempting to sue themselves along with the rest of the people represented by their common government an
anomalous and absurd situation indeed.
The Supreme Court ruled that the proceedings in the Court of Industrial Relations and now subject of this appeal
are null and void and enjoined from taking any further action.
DOCTRINE: In a republican state, like the Philippines, government immunity from suit without its consent is derived from
the will of the people themselves in freely creating a government of the people, by the people, and for the people a
representative government through which they have agreed to exercise the powers and discharge the duties of their
sovereignty for the common good and general welfare. In so agreeing, the citizens have solemnly undertaken to surrender
some of their private rights and interest which were calculated to conflict with the higher rights and larger interests of the
people as a whole, represented by the government thus established by them all. One of those higher rights, based upon
those larger interests is that government immunity.

NAC v. Teodoro
FACTS: The National Airports Corporation was organized under Republic Act No. 224, which expressly made the
provisions of the Corporation Law applicable to the said corporation. It was abolished by Executive Order No. 365 and to
take its place the Civil Aeronautics Administration was created.
Before the abolition, the Philippine Airlines, Inc. paid to the National Airports Corporation P65,245 as fees for landing and
parking for the period up to and including July 31, 1948. These fees are said to have been due and payable to the Capitol
Subdivision, Inc., who owned the land used by the National Airports Corporation as airport. The owner commenced an
action in the court against the Philippine Airlines, Inc.
The Philippine Airlines, Inc. countered with a third-party complaint against the National Airports Corporation, which by that
time had been dissolved, and served summons on the Civil Aeronautics Administration. The third party plaintiff alleged
that it had paid to the National Airports Corporation the fees claimed by the Capitol Subdivision, Inc. on the belief and
assumption that the third party defendant was the lessee of the lands subject of the complaint and that the third party
defendant and its predecessors in interest were the operators and maintainers of said airport and, further, that the third
party defendant would pay to the landowners, particularly the Capitol Subdivision, Inc., the reasonable rentals for the use
of their lands.
The Solicitor General, after answering the third party complaint, filed a motion to dismiss on the ground that the court
lacks jurisdiction to entertain the third-party complaint, first, because the National Airports Corporation has lost its juridical
personality, and, second, because agency of the Republic of the Philippines, unincorporated and not possessing juridical
personality under the law, is incapable of suing and being sued.
ISSUE: WON the Civil Aeronautics Administration should be regarded as engaged in private functions and therefore
subject to suit
HELD: Yes, the Supreme Court ruled that the Civil Aeronautics Administration comes under the category of a private
entity. Although not a body corporate it was created, like the National Airports Corporation, not to maintain a necessary
function of government, but to run what is essentially a business, even if revenues be not its prime objective but rather the
promotion of travel and the convenience of the traveling public. It is engaged in an enterprise which, far from being the
exclusive prerogative of state, may, more than the construction of public roads, be undertaken by private concerns.
In the light of a well-established precedents, and as a matter of simple justice to the parties who dealt with the National
Airports Corporation on the faith of equality in the enforcement of their mutual commitments, the Civil Aeronautics
Administration may not, and should not, claim for itself the privileges and immunities of the sovereign state.

DOCTRINE: Not all government entities, whether corporate or non corporate, are immune from suits. The power to sue
and be sued is implied from the power to transact private business.

Mobil Philippines v. Customs Arrestre


FACTS: Four cases of rotary drill parts were shipped from abroad on S.S. "Leoville", consigned to Mobil Philippines
Exploration, Inc., Manila. The shipment was discharged to the custody of the Customs Arrastre Service, the unit of the
Bureau of Customs then handling arrastre operations therein.
The Customs Arrastre Service later delivered to the broker of the consignee three cases only of the shipment.
Mobil Philippines Exploration, Inc., filed suit in the Court of First Instance of Manila against the Customs Arrastre Service
and the Bureau of Customs to recover the value of the undelivered case in the amount of P18,493.37 plus other
damages.
Defendants filed a motion to dismiss the complaint on the ground that not being persons under the law,
defendants cannot be sued. Appellant contends that not all government entities are immune from suit; that defendant
Bureau of Customs as operator of the arrastre service at the Port of Manila, is discharging proprietary functions and as
such, can be sued by private individuals.
ISSUE: WON the defendants can invoke the doctrine of immunity from suit
HELD: The fact that a non-corporate government entity performs a function proprietary in nature does not necessarily
result in its being suable. If said non-governmental function is undertaken as an incident to its governmental function,
there is no waiver thereby of the sovereign immunity from suit extended to such government entity.
The Bureau of Customs, to repeat, is part of the Department of Finance, with no personality of its own apart
from that of the national government. Its primary function is governmental, that of assessing and collecting lawful
revenues from imported articles and all other tariff and customs duties, fees, charges, fines and penalties (Sec. 602, R.A.
1937).
To this function, arrastre service is a necessary incident. Clearly, therefore, although said arrastre function may
be deemed proprietary, it is a necessary incident of the governmental function of the Bureau of Customs, so that engaging
in the same does not necessarily render said Bureau liable to suit. For otherwise, it could not perform its governmental
function without necessarily exposing itself to suit. Sovereign immunity, granted as to the end, should not be denied as to
the necessary means to that end.
DOCTRINE: There is implied consent when the state enters into an operation that is essentially a business operation,
unless the business operation is merely incidental to the performance of a governmental function, as for instance, arrastre
service.

Del Mar v. PVA


FACTS: Petitioner filed a petition for mandamus against respondent to compel the latter to continue paying him his monthly life pension of P50 from
March 1950 (when it was cancelled) to June20, 1957 and from June 22, 1957, his monthly life pension, as increased by Republic Act 1920,of P100, and
to pay to him as well the monthly living allowance of P10 for each of his unmarried minor children below eighteen years of age, pursuant to the said
Republic Act1920 which took effect on June 22, 1957. Del Mar also asked for compensatory, moral and exemplary damages.
On the other hand, respondent contended that:

petitioner is barred from claiming and receiving since he is also receiving similarpension from the US.
PVAcannot be sued because it is an government agency.

Trial court ruled in favor of petitioner granting his prayers except for compensatory, moral and exemplary damages.
ISSUE: WON PVAcan invoke the doctrine of immunity from suit
RULING: As a general proposition, the immunity of the Government from suit without its consent holds true in all actions
resulting in "adverse consequences on the public treasury, whether in the disbursements of funds or loss of property."
Needless to state, in such actions, which, in effect, constitute suits against the Government, the court has no option but to
dismiss them.

Nonetheless, the rule admits of an exception. It finds no application where a claimant institutes an action against a
functionary who fails to comply with his statutory duty to release the amount claimed from the public funds already
appropriated by statute for the benefit of the said claimant. As clearly discernible from the circumstances, the case at bar
falls under the exception.

GOVERNMENT OFFICERS
Regional Director Teofilo Gomez VS CA
The private respondents, together with other Negros Oriental public school teachers, held, starting 19 September 1990
and lasting until 21 September 1990, a mass action, or a strike from their school classes, to demand the release of their
salaries by the Department of Budget.
A return-to-work order was promptly issued by one of the petitioners, Regional Director Teofilo Gomez of the Department
of Education, Culture and Sports ("DECS"), with a warning that if the "striking" school teachers were not to resume their
classes within twenty-four hours, administrative charges would be filed. Since the order was not heeded, administrative
complaints against the teachers concerned were thereupon filed. The teachers were each given five days from receipt of
said complaints within which to submit their respective answers and supporting documents. An investigation panel,
composed of three DECS lawyers (the other petitioners herein), namely, Marcelo Baclaso, Nieva Montes and
GenerosoCapuyan, was constituted to look into the case.
Main point: (2) Public officials are not necessarily immune from damages in their personal capacities arising from acts
done in bad faith, for if malice is indeed established, public officials can no longer be said to have acted within the scope
of official authority so as to still find protection under the mantle of immunity for official actions
DOH v. PHIL. PHARMAWEALTH, INC
Defense of state immunity does not apply where the public official is charged in his official capacity for acts that are
unauthorized or unlawful and injurious to the rights of others neither does it apply where the public official is clearly being
sued not in his official capacity but in his personal capacity, although the acts complained of may have been committed
while he occupied a public position.
FACTS: Secretary of Health Alberto G. Romualdez, Jr. issued an Administrative Order providing for additional guidelines
for accreditation of drug suppliers aimed at ensuring that only qualified bidders can transact business with petitioner
Department of Health (DOH). Respondent Phil. Pharmawealth, Inc. (Pharmawealth) submitted to DOH a request for the
inclusion of additional items in its list of accredited drug products, including the antibiotic Penicillin G Benzathine.
Petitioner DOH issued an Invitation for Bids for the procurement of 1.2 million units vials of Penicillin G Benzathine.
Despite the lack of response from DOH regarding Pharmawealths request for inclusion of additional items in its list of
accredited products, the latter submitted its bid for the Penicillin G Benzathine contract and gave the lowest bid thereof. .
In view, however, of the non-accreditation of respondents Penicillin G Benzathine product, the contract was awarded to
Cathay/YSSLaboratories (YSS).
Respondent Pharmawealth filed a complaint for injunction, mandamus and damages with prayer for the issuance of a writ
of preliminary injunction and/or temporary restraining order with the Regional Trial praying, inter alia, that the trial court
nullify the award of the Penicillin G Benzathine contract to YSS Laboratories, Inc. and direct petitioners DOH et al. to
declare Pharmawealth as the lowest complying responsible bidder for the Benzathine contract, and that they accordingly
award the same to plaintiff company and adjudge defendants Romualdez, Galon and Lopez liable, jointly and severally
to plaintiff. Petitioners DOH et al. subsequently filed a motion to dismiss praying for the dismissal of the complaint based
on the doctrine of state immunity. The trial court, however, denied the motion to dismiss. The Court of Appeals (CA)
denied DOHs petition for review which affirmed the order issued Regional Trial Court of Pasig City denying petitioners
motion to dismiss the case.
ISSUE: Whether or not the charge against the public officers acting in their official capacity will prosper
HELD: The suability of a government official depends on whether the official concerned was acting within his official or
jurisdictional capacity, and whether the acts done in the performance of official functions will result in a charge or financial
liability against the government. In its complaint, DOH sufficiently imputes grave abuse of discretion against petitioners in
their official capacity. Since judicial review of acts alleged to have been tainted with grave abuse of discretion is
guaranteed by the Constitution, it necessarily follows that it is the official concerned who should be impleaded as
defendant or respondent in an appropriate suit.

As regards petitioner DOH, the defense of immunity from suit will not avail despite its being an unincorporated agency of
the government, for the only causes of action directed against it are preliminary injunction and mandamus. Under Section
1, Rule 58 of the Rules of Court, preliminary injunction may be directed against a party or a court, agency or a person.
Moreover, the defense of state immunity from suit does not apply in causes of action which do not seek to impose a
charge or financial liability against the State.
Hence, the rule does not apply where the public official is charged in his official capacity for acts that are unauthorized or
unlawful and injurious to the rights of others. Neither does it apply where the public official is clearly being sued not in his
official capacity but in his personal capacity, although the acts complained of may have been committed while he occupied
a public position.
In the present case, suing individual petitioners in their personal capacities for damages in connection with their alleged
act of illegally abusing their official positions to make sure that plaintiff Pharmawealth would not be awarded the
Benzathine contract [which act was] done in bad faith and with full knowledge of the limits and breadth of their powers
given by law is permissible, in consonance with the foregoing principles. For an officer who exceeds the power conferred
on him by law cannot hide behind the plea of sovereign immunity and must bear the liability personally.
FOREIGN GOV
BAER vs. TIZONG.R. No. L-24294May 3, 1974
FACTS:Respondent Edgardo Gener, as plaintiff, filed a complaint for injunction with the Court of First Instance of Bataan
against petitioner, Donald Baer, Commander of the United States Naval Base in Olongapo.

He alleged that he was engaged in the business of logging and that the American Naval Base authorities stopped his
logging operations.
He prayed for a writ of preliminary injunction restraining petitioner from interfering with his logging operations.
A restraining order was issued by respondent Judge
Counsel for petitioner, upon instructions of the American Ambassador to the Philippines, entered their appearance for the
purpose of contesting the jurisdiction of respondent Judge on the ground that the suit was one against a foreign sovereign
without its consent.
ISSUE:Whether the contention of the petitioner that the respondent judge acquires no jurisdiction on the ground that the
suit was one against a foreign sovereign without its consent.
HELD:YES. The contention of the petitioner is tenable.The writ of certiorari prayed for is granted, nullifying and setting
aside the writ of preliminary injunction.
The invocation of the doctrine of immunity from suit of a foreign state without its consent is appropriate.
The doctrine of state immunity is not limited to cases which would result in a pecuniary charge against the sovereign or
would require the doing of an affirmative act by it. Prevention of a sovereign from doing an affirmative act pertaining
directly and immediately to the most important public function of any government - the defense of the state - is equally as
untenable as requiring it to do an affirmative act."That such an appraisal is not opposed to the interpretation of the
relevant treaty provision by our government is made clear in the aforesaid manifestation and memorandum as amicus
curiae, wherein it joined petitioner for the grant of the remedy prayed for.
There should be no misinterpretation of the scope of the decision reached by this Court. Petitioner, as the
Commander of the United States Naval Base in Olongapo, does not possess diplomatic immunity. He may therefore be
proceeded against in his personal capacity, or when the action taken by him cannot be imputed to the government which
he represents.
**** USA vs. RuizG.R. No. L-35645, May 22, 1985
Facts:
The United States of America had a naval base in Subic, Zambales. The base was one of thoseprovided in the Military
Bases Agreement between the Philippines and the US. Respondent alleges thatit won in the bidding conducted by the US
from the construction of wharves in said base that was wronglyawarded to another group. For this reason, a suit for
specific performance was filed by him against theUS.

Issue:
Whether the United States Naval Base in bidding for said contracts exercise governmentalfunctions to be able to invoke
state immunity.
Held:
The traditional rule of State immunity exempts a state from being sued in the courts of anotherstate without its consent or
waiver. This rule is a necessary consequence of the principles of independence and equality of states. However, the rules
of international law are not petrified; theyare constantly developing and evolving. And because the activities of states have
multiplied, it hasbeen necessary to distinguish thembetween sovereign and governmental acts and private,commercial
and proprietary acts. The result is that state immunity now extends only to sovereign andgovernmental acts.The restrictive
application of state immunity is proper only when the proceedings arise out of commercial transactions of the foreign
sovereign, its commercial activities or economic affairs. A statemay be said to have descended to the level of an individual
and can thus be deemed to have tacitlygiven its consent to be sued only when it enters into business contracts. It does
not apply where thecontract relates the exercise of its sovereign function. In this case, the projects are an integral part of
the naval base which is devoted to the defense of both the US and the Philippines, indisputably afunction of the
government of the highest order; they are not utilized for nor dedicated to commercialor business purposes.
DALE SANDERS, AND A.S. MOREAU, JR vs. HON. REGINO T. VERIDIANO
FACTS:
Petitioner Sanders was the special services director of the U.S. Naval Station. Petitioner Moreau was the commanding
officer of the Subic Naval Base. Private respondent Rossi is an American citizen with permanent residence in the
Philippines.
Private respondent Rossi and Wyer were both employed as game room attendants in the special services department of
the NAVSTA. On October 3, 1975, the private respondents were advised that their employment had been converted from
permanent full-time to permanent part-time. They instituted grievance proceedings to the rules and regulations of the U.S.
Department of Defense. The hearing officer recommended for reinstatement of their permanent full-time status. However,
in a letter addressed to petitioner Moreau, Sanders disagreed with the hearing officer's report. The letter contained the
statements that: a ) "Mr. Rossi tends to alienate most co-workers and supervisors;" b) "Messrs. Rossi and Wyers have
proven, according to their immediate supervisors, to be difficult employees to supervise;" and c) "even though the grievant
were under oath not to discuss the case with anyone, (they) placed the records in public places where others not involved
in the case could hear." Before the start of the grievance hearings, a-letter from petitioner Moreau was sent to the Chief of
Naval Personnel explaining the change of the private respondent's employment status. So, private respondent filed for
damages alleging that the letters contained libelous imputations and that the prejudgment of the grievance proceedings
was an invasion of their personal and proprietary rights. However, petitioners argued that the acts complained of were
performed by them in the discharge of their official duties and that, consequently, the court had no jurisdiction over them
under the doctrine of state immunity. However, the motion was denied on the main ground that the petitioners had not
presented any evidence that their acts were official in nature.
ISSUE:
Whether or not the petitioners were performing their official duties?
RULING:
Yes. Sanders, as director of the special services department of NAVSTA, undoubtedly had supervision over its personnel,
including the private respondents. Given the official character of the letters, the petitioners were being sued as officers of
the United States government because they have acted on behalf of that government and within the scope of their
authority. Thus, it is that government and not the petitioners personally that is responsible for their acts. It is stressed at
the outset that the mere allegation that a government functionary is being sued in his personal capacity will not
automatically remove him from the protection of the law of public officers and, if appropriate, the doctrine of state
immunity. By the same token, the mere invocation of official character will not suffice to insulate him from suability and
liability for an act imputed to him as a personal tort committed without or in excess of his authority. These well-settled
principles are applicable not only to the officers of the local state but also where the person sued in its courts pertains to
the government of a foreign state, as in the present case. Assuming that the trial can proceed and it is proved that the
claimants have a right to the payment of damages, such award will have to be satisfied not by the petitioners in their
personal capacities but by the United States government as their principal. This will require that government to perform
an affirmative act to satisfy the judgment, viz, the appropriation of the necessary amount to cover the damages
awarded, thus making the action a suit against that government without its consent.
USA and MAXINE BRADFORD, petitioners, vs. HON. LUIS R. REYES, as Presiding Judge of Branch 22, Regional Trial
Court of Cavite, andNELIA T. MONTOYA, respondents.

FACTS:Private respondent, hereinafter referred to as Montoya, is an American citizen who, at thetime material to this
case, was employed as an identification (I.D.) checker at the U.S. NavyExchange (NEX) at the Joint United States Military
Assistance Group (JUSMAG) headquarters inQuezon City. She is married to one Edgardo H. Montoya, a FilipinoAmerican serviceman employedby the U.S. Navy and stationed in San Francisco, California. Petitioner Maxine Bradford,
hereinafterreferred to as Bradford, is likewise an American citizen who was the activity exchange manager atthe said
JUSMAG Headquarters.As a consequence of an incident which occurred on 22 January 1987 whereby her body
andbelongings were searched after she had bought some items from the retail store of the NEXJUSMAG, where she had
purchasing privileges, and while she was already at the parking area,Montoya filed on7 May 1987 a complaint 1 with the
Regional Trial Court of her place of residenceCavite against Bradford for damages due to the oppressive and
discriminatory acts committed by the latterin excess of her authority as store manager of the NEXJUSMAG.In support of
the motion, the petitioners claimed that JUSMAG, composed of an Army, Navy and AirGroup, had been established under
the Philippine-United States Military Assistance Agreemententered into on 21 March 1947 to implement the United States'
program of rendering military assistance to the Philippines. Its headquarters in Quezon City is considered a temporary
installationunder the provisions of Article XXI of the Military Bases Agreement of 1947. Thereunder, "it ismutually agreed
that the United States shall have the rights, power and authority within the baseswhich are necessary for the
establishment, use and operation and defense thereof or appropriate forthe control thereof." The 1979 amendment of the
Military Bases Agreement made it clear that theUnited States shall have "the use of certain facilities and areas within the
bases and shall haveeffective command and control over such facilities and over United States personnel,
employees,equipment and material." JUSMAG maintains, at its Quezon City headquarters, a Navy Exchangereferred to
as the NEX-JUSMAG. Checking of purchases at the NEX is a routine procedureobserved at base retail outlets to protect
and safeguard merchandise, cash and equipment pursuantto paragraphs 2 and 4(b) of NAVRESALEACT SUBIC INST.
5500.1. 7 Thus, Bradford's order tohave purchases of all employees checked on 22 January 1987 was made in the
exercise of herduties as Manager of the NEX-JUSMAG.
ISSUES:
whether or not the trial court committed grave abuse of discretion in denying the motion to dismiss based on the following
grounds:
(a) the complaint in Civil Case No. 224-87 is in effect a suit against the public petitioner, a foreign sovereign immune from
suit which has not given consent to such suit and
(b) Bradford is immune from suit for acts done by her in the performance of her official functions as manager of the U.S.
Navy Exchange of JUSMAG pursuant to the Philippines-United States Military Assistance Agreement of 1947 and the
Military Bases Agreement of 1947, as amended.
HELD:
The petition was DENIED for lack of merit. There can be no doubt that on the basis of the allegations in the complaint,
Montoya has a sufficient and viable cause of action. Bradford's purported non-suability on the ground of state immunity is
then a defense which may be pleaded in the answer and proven at the trial. Since Bradford did not file her Answer within
the reglamentary period, the trial court correctly declared her in default upon motion of the private respondent. The
judgment then rendered against her on 10 September 1987 after the ex parte reception of the evidence for the private
respondent and before this Court issued the Temporary Restraining Order on 7 December 1987 cannot be impugned. The
filing of the instant petition and the knowledge thereof by the trial court did not prevent the latter from proceeding with Civil
Case No.224-87. "It is elementary that the mere pendency of a special civil action for certiorari, commenced in relation to
a case pending before a lower Court, does not interrupt the course of the latter when there is no writ of injunction
restraining it."
THE HOLY SEE vs. THE HON. ERIBERTO U. ROSARIO, JR., as Presiding Judge of theRegional Trial Court of
Makati, Branch 61 and STAR BRIGHT SALES ENTERPRISES, INC.
FACTS: This petition arose from a controversy over a parcel of land, Lot 5-A, located in the Municipality of Paraaque,
Metro Manila and registered in the name of petitioner. Said Lot 5-A is contiguous to Lots 5-B and 5-D registered in the
name of the Philippine Realty Corporation (PRC). The three lots were sold to Ramon Licup, through Msgr. Domingo A.
Cirilos, Jr., acting as agent to the sellers. Later, Licup assigned his rights to the sale to private respondent, Star bright
Enterprises. The squatters refused to vacate the lots sold to private respondent so a dispute arose as to who of the parties
has the responsibility of evicting and clearing the land of squatters occurred. Complicating the relations of the parties was
the sale by petitioner of Lot 5-A to Tropicana Properties and Development Corporation (Tropicana).Private respondent
filed a complaint for annulment of the sale of the three parcels of land, and specific performance and damages against
petitioner, represented by the Papal Nuncio, and three other defendants: namely, Msgr. Domingo A. Cirilos, Jr., the PRC
and Tropicana.
ISSUE: WON the petitioner Holy See is immune from suit.

HELD: YES. The logical question is whether the foreign state is engaged in the activity in the regular course of business. If
the foreign state is not engaged regularly in a business or trade, the particular act or transaction must then be tested by
its nature. If the act is in pursuit of a sovereign activity, or an incident thereof, then it is an act
jureimperii , especially when it is not undertaken for gain or profit. Lot5-A was acquired by petitioner as a donation from
the Archdiocese of Manila. The donation was made not for commercial purpose, but for the use of petitioner to construct
thereon the official place of residence of the Papal Nuncio. The right of a foreign sovereign to acquire property, real or
personal, in a receiving state, necessary for the creation and maintenance of its diplomatic mission, is recognized in the
1961 Vienna Convention on Diplomatic Relations. In Article 31(a) of the Convention, a diplomatic envoy is granted
immunity from the civil and administrative jurisdiction of the receiving state over any real action relating to private
immovable property situated in the territory of the receiving state which the envoy holds on behalf of the sending state for
the purposes of the mission. If this immunity is provided for a diplomatic envoy, with all the more reason should immunity
be recognized as regards the sovereign itself, which in this case is the Holy See.
JUSMAG
Government)

Philippines

vs.

NLRC

(Article

XVI

Sec.

3)

(Foreign

Facts:
Florencio
Sacramento
(private
respondent)
was
one
of
the
74
security
assistance
support
personnel
(SASP)
working
at
the
Joint
United
States
Military
Assistance
Group
to
the
Philippines
(JUSMAGPhils.). He had been with JUSMAG from 1969-1992. When dismissed, he held the position of Illustrator 2 and incumbent
Pres.
of
JUSMAG
Phils.Filipino
Civilian
Employees
Assoc.,
a
labor
org.
duly
registered
with
DOLE.
His
services
were
terminated
allegedly
due
to
the abolition of his position.
Sacramento
filed
complaint
w/
DOLE
on
the
ground
that
he
was
illegally
suspended
and
dismissed
from
service
by
JUSMAG.
He
asked
for
reinstatement.
JUSMAG
filed
Motion
to
Dismiss
invoking
immunity
from
suit.
Labor
arbiter
Cueto
in
an
Order
dismissed
the
complaint
"for
want
of
jurisdiction".
Sacramento
appealed
to
NLRC.
NLRC reversed the ruling of the labor arbiter and held that
petitioner had lost its right not to be sued because (1) the principle
of estoppel-that JUSMAG failed to refute the existence of employeremployee relation (2) JUSMAG has waived its right to immunity from suit when it hired the services of private respondent.
Hence, this
petition.
Issue:W/N JUSMAG has immunity from suit
Held: Yes. When JUSMAG took the services of private respondent, it was performing a governmental function on behalf of
the United
States pursuant to the Military Assistance Agreement between the
Phils. and America* JUSMAG consists of Air, Naval and Army group
and its primary task was to advise and assist the Phils. on air
force, army and naval matters. A suit against JUSMAG is one against the United States government, and in the absence
of any waiver or consent of the latter to the suit, the complaint against JUSMAG cannot prosper
Immunity of State from suit is one of the universally recognized principles of international law that the Phils. recognizes
and adopts as part of the law of the land. Immunity is commonly understood as the exemption of the state and its
organs from the judicial jurisdiction of another state and anchored on the principle of the sovereign equality of states
under which one state cannot assert jurisdiction over another in violation of the maxim par in parem non habeat imperium
(an equal has no power over an equal)
As it stands now, the application of the doctrine of immunity from suit has been restricted to sovereign or
governmental activities and does not extend to commercial, private and proprietary acts.
Larkins vs NLRC
Facts:

On August 12, 1988, private respondents filed a complaint with the Regional ArbitrationBranch No. III of the NLRC, San
Fernando, Pampanga for illegal dismissal andunderpayment of wages.
Charges were against petitionerT/SgtAldora Larkinswho was a member of the UnitedStates Air Force (USAF) assigned to
oversee the dormitories of the Third AircraftGeneration Squadron (3 AGS) at Clark Air Base, Pampanga.,Lt. Col.
Frankhauster, andJoselito Cunanan, the new contractor (JAC Maintenance Services) employed for 3 AGS
Petitioner and Lt. Col. Frankhauserfailed to answer the complaint and to appear at thehearings. They, likewise, failed to
submit their position paper, which theLabor Arbiter deemed a waiver on their part to do so. The case was therefore
submitted for decisionon the basis of private respondents' position paper and supporting documentswhichtherefore on
November 21, 1988, the Labor Arbiter rendered a decision granting all theclaims of private respondents. He found both Lt.
Col. Frankhauser and petitioner"guiltyof illegal dismissal"and ordered them to reinstate private respondents with full
backwages, or if that is no longer possible, to pay private respondents' separation pay.
Petitioner appealed to the NLRC claiming that the Labor Arbiter never acquired jurisdiction over her person because no
summons or copies of the complaints, bothoriginal and amended, were ever served on her. In her "Supplemental
Memorandum of Appeal," petitioner argued that the attempts to serve her with notices of hearing werenot in accordance
with the provisions of theR.P. U.S. Military Bases Agreement of 1947.
Held: Petitioner's contention that the questioned resolutions are
null and void because respondent Labor Arbiter did not acquire jurisdiction to entertain and decide the case.Petitioner
alleges that she never received nor was served, any summons or copies of theoriginal and amended complaints, and
therefore the Labor Arbiter had no jurisdictionover her person under Article XIV of the R.P.U.S. Military Bases
Agreement.. . . [N]o process, civil or criminal, shall be served within any base except with thepermission of the
commanding officer of such base; but should the commanding officerrefuse to grant such permission he shall forthwith
take the necessary steps . . . to servesuch process, as the case may be, and to provide the attendance of the server
of suchprocess before the appropriate court in the Philippines or procure such server to makethe necessary affidavit or
declaration to prove such service as the case may require."
Ruling:
Labor Arbiter has no jurisdiction over the case as summonses and other processesissued by Philippine courts
and administrative agencies for United States Armed Forcespersonnel within any U.S. base in the Philippines could be
served therein only with thepermission of the Base Commander. If he withholds giving his permission, he shouldinstead
designate another person to serve the process, and obtain the server's affidavit for filing with the appropriate court.
Respondent Labor Arbiter did not follow
Saidprocedure. He instead, addressed the summons to Lt. Col. Frankhauser and not the BaseCommander.
Respondents do not dispute petitioner's claim that no summons was ever issued andserved on her. They contend,
however, that they sent notices of the hearings to herBUTas contended notices of hearing are not summonses. The
provisions and revailing jurisprudence in Civil Procedure may be applied by analogy to NLRC proceedings(Revised Rules
of the NLRC, Rule I, Sec. 3). It is basic that the Labor Arbiter cannotacquire jurisdiction over the person of the respondent
without the latter being servedwith summons. In theabsence of service of summons or a valid waiver thereof, the hearings
and judgmentrendered by the Labor Arbiter are null and void.
Petitioner, in the case at bench, appealed to the NLRC and participated in the oralargument before the said body. This,
however, does not constitute a waiver of the lackof summons and a voluntary submission of her person to the jurisdiction
of the LaborArbiter. She may have raised in her pleadings grounds other than lack of jurisdiction, butthese grounds were
discussed in relation to and as a result of the issue of the lack of jurisdiction. In effect, petitioner set forth only one issue
and that is the absence of jurisdiction over her person. If an appearance before the NLRC is precisely to questionthe
jurisdiction of the said agency over the person of the defendant, then thisappearance is not equivalent to service of
summons.
Be that as it may, on the assumption that petitioner validly waived service of summonson her, still the case could not
prosper. There is no allegation from the pleadings filedthat Lt. Col. Frankhauser and petitioner were being sued in their
personal capacities for tortious acts. However,private respondents named 3 AGS as one of the respondents in their
complaint.
Indeed, assuming that jurisdiction was acquired over the United States Government andthe monetary claims of private
respondents proved, such awards will have to besatisfied not by Lt. Col. Frankhauser and petitioner in their personal
capacities, but by the United States government.

Article XIV GENERAL PROVISIONS


Sec.3 Immunity from Sui
PNR v. IAC
FACTS: In this case PNR train and Baliwag Bus collided at the railroad crossing resulting damage and take lives of their
respective passengers where18 died and 53 are injured in the incident. The plantiff-Malong (respondent) alleged that it
was due to the negligence of PNR for not operating the crossing at its precautionary measures however the trial court
dismissed the case because it has no jurisdiction over PNR for being government instrumentality and the action will be
against the state.
ISSUE: Whether PNR is immune from suit.
HELD: No, the PNR is not immune from suit as to its predecessor charter the Manila Road Company with the same
carrier under Act No.1510 as a purely government owned and controlled corporation duly registered and existing virtue of
Presidential Decree No. 741, with capacity to sue and be sued, and is likewise engaged in transporting passengers and
cargoes by trains and buses and that, it operate.
Doctrine: Implied powers to the effect that the power to sue and be sued is implicit from the faculty to transact private
business the railroad company as a private entity created not to discharge a governmental function but, among other
things, to operate a transport service which is essentially a business concern, and thus barred from invoking immunity
from suit.
REPUBLIC v. NOLASCO
FACTS: A petition made by Nolasco alleging the anomaly between the DPWH and BAC(Bids and Award Committee) for
awarding a contract DAEWOO that the contract is putative, illegal and prejudicial to the government and Filipino
taxpayers.
However
BAC
resolved
the
issue
that
DAEWOO
has
lowest
bid.
ISSUE: Whether the petition was a suit against the State.
HELD:Yes, the petition is against the State because DPWH is unincorporated agency that has no separate juridical
capacity of its own and hence enjoys immunity from suit.
Doctrine:The unincorporated agency is so called because it has no separate juridical personality but is merged in the
general machinery of the government.

REPUBLIC v. UNIMEX
FACTS: A petition was filed by the UNIMEX against BOC (Bureau of Customs) as the Customs seized and forfeited the
shipment of the UNIMEX however CTA (Court of Tax Appeals) ordered the release of the shipment or to pay the
commercial value of the goods with interest.
ISSUE: Whether the state is immune from suit and damages incurred.
HELD:No, because BOCs ineptitude and gross negligence in the safekeeping of respondents goods. We are not likewise
unaware of its lackadaisical attitude in failing to provide a cogent explanation on the goods disappearance, considering
that they were in its custody and that they were in fact the subject of litigation.
Doctrine: The doctrine of state immunity must be fairly observed and the State should not avail itself of this prerogative to
take undue advantage of parties that may have legitimate claims against it.

PROFESSIONAL VIDEO v. TESDA


FACTS:A petition was made by PROVI against TESDA as its bidder for making the PVC IDs allegedly TESDA didnt pay
PROVI for corresponding amount that they have contracted and the petition ask for the writ of preliminary attachment for
TESDAs public funds.

ISSUE: Whether the suit is against the state.


HELD: Yes, TESDA is unincorporated instrumentality attached to DOLE operating under the government and equipped
with express and implied powers and state immunities apply to it.
Doctrine: The unincorporated agency is so called because it has no separate juridical personality but is merged in the
general machinery of the government.
Government Officers
MINISTERO v. CFI
FACTS:Petitioners sought the payment of just compensation for registered lot alleging that 1927 the National Government
through its authorized representatives took physical and material possession of its used for road widening and they
demand the payment for its price but Public Highways refused.
ISSUE: Whether the Public Highway Commissioner is immune for suit.
HELD: No, the state immunity cannot be an instrument to prejudice petitioners who was legally owned the lot and taking
way property from a private landowner for public use without through the legal process of expropriation or negotiated sale
it violates the doctrine of immunity from suit
Doctrine: The doctrine of governmental immunity from suit cannot serve as an instrument for perpetrating an injustice on
a citizen.
SYQUIA v. ALMEDA-LOPEZ
FACTS:Petitioners are owner of an apartment in City of Manila and contracted with the U.S. government for the lease of
the apartment accommodating U.S. soldiers allegedly the U.S. government upon after the Japanese war incurred
damages and unpaid rent from the petitioner
ISSUE: Whether the U.S. government with its agents can be held liable for the suit and damages.
HELD: No, The U. S. Government has not given its consent to the filing of this suit which is essentially against her, though
not in name. Moreover, this is not only a case of a citizen filing a suit against his own Government without the latter's
consent but it is of citizen filing an action against a foreign government without said government's consent, which renders
more obvious the lack of jurisdiction of the courts of his country.
Doctrine: When the state enters into a private contract the consent to be sued is given, however if the contract is merely
incidental to the performance of a governmental function.
FESTEJO v. FERNANDO
FACTS:The petitioner comes to court contending that the Director of the Bureau of Public Works took possession of her 3
parcels of land without obtaining the right of way, the consent and knowledge of petitioner where she demands the land to
be restored and to pay her for the damages of unlawful possession.
ISSUE: Whether the Director as government officer can be held liable for suit and damages.
HELD: Yes, It is a general rule that an officer-executive, administrative quasi-judicial, ministerial, or otherwise who acts
outside the scope of his jurisdiction and without authorization of law may thereby render himself amenable to personal
liability in a civil suit.
Doctrine: If a government officer exceed the power conferred on him by law, he cannot shelter himself by the plea that he
is a public agent acting under the color of his office, and not personally. In the eye of the law, his acts then are wholly
without authority.

ABERCA v. VER
FACTS:Petitioners were arrested without proper arrest warrant for a period and they are interrogated by the military
personnel of Armed Forces of the Philippines ordered by General Fabian Ver to conduct pre-emptive strike against known
communist-terrorist and then they filed an actions for damages against the respondent officers of AFP however
respondent AFP officers invoke that they are immune from liability for acts done in the performance of their official duties

ISSUE: Whether the AFP is immune from suit in the performance of their official duty.
HELD: No, as members of the AFP, were merely responding to their duty, as they claim, "to prevent or suppress lawless
violence, insurrection, rebellion and subversion" in accordance with Proclamation No. 2054 of President Marcos, despite
the lifting of martial law on January 27, 1981, and in pursuance of such objective, to launch pre- emptive strikes against
alleged communist terrorist underground houses. But this cannot be construed as a blanket license or a roving
commission untrammeled by any constitutional restraint, to disregard or transgress upon the rights and liberties of the
individual citizen enshrined in and protected by the Constitution.
Doctrine: If a government officer exceed the power conferred on him by law, he cannot shelter himself by the plea that he
is a public agent acting under the color of his office, and not personally. In the eye of the law, his acts then are wholly
without authority.
SHAUF v. CA
FACTS:Petitioner, a Filipino by origin and married to an American who is a member of the US Air Force, was rejected for a
position of Guidance Counselor in the Base Education Office at Clark Air Base. According to applicable regulations, where
there are qualified dependents of military or civilian personnel, who are locally available, appointments to positions shall
be limited to the dependents. As per records, she possessed all the qualifications for the job at that time. Instead of hiring
Shauf, however, one Mr. Isakson was selected for the position who was not a dependent of a military or civilian personnel.
In addition, Mr. Isakson apparently lacked certain qualifications by these reasons she filed a complaint for damages and
an equal employment opportunity against respondents for alleged discrimination by reason of her national origin as
Filipino, complexion and sex.
ISSUE: Whether the officers of U.S. armed forces are immune from suit.
HELD: No, the respondents cannot rely on the U.S. blanket of diplomatic immunity for all its acts or the acts of its agent in
the Philippines they are personally liable for the damages.
Doctrine: It is further contended that the rule allowing suits against public officers and employees for unauthorized acts,
torts and criminal acts is a rule of domestic law, not of international law. It applies to cases involving the relations between
private suitors and their government or state, not the relations between one government and another from which springs
the doctrine of immunity of a foreign sovereign.

VIDAD v. RTC
FACTS:A group of public school teachers in Negros Oriental held, starting 19 September 1990 and lasting until 21
September 1990, a must action, or a strike from their school classes, to demand the release of their salaries by the
Department of Budget. The teachers also assailed alleged corruption in the Department of Education, Culture and Sports
(DECS).
ISSUE: Whether the officials of DECS can be liable for the suit and damages.
HELD: Yes, Public officials are certainly not immune from damages in their personal capacities arising from the acts done
in bad faith; in these and similar cases, the public officials may not be said to have acted within the scope of their official
authority, and no longer are they protected by the mantle of immunity for official actions.
Doctrine: If a government officer exceed the power conferred on him by law, he cannot shelter himself by the plea that he
is a public agent acting under the color of his office, and not personally. In the eye of the law, his acts then are wholly
without authority.
Waiver
KHOSROW MINUCHER, vs. HON. COURT OF APPEALS and ARTHUR SCALZO,
FACTS:
Khosrow Minucher, an Iranian national and a Labor Attach for the Iranian Embassies in Tokyo, Japan and Manila came
to the country to study in 1974 and continued to stay as head of the Iranian National Resistance Movement.
In May 1986, Minucher was charged with an Information for violation of Republic Act No. 6425, Dangerous Drugs Act of
1972. The criminal charge followed a buy-bust operation conducted by the Philippine police narcotic agents in his house

where a quantity of heroin was said to have been seized. The narcotic agents were accompanied by private respondent
Arthur Scalzo who became one of the principal witnesses for the prosecution.
In August 1988, Minucher filed Civil Case before the Regional Trial Court (RTC) for damages on the trumped-up charges
of drug trafficking made by Arthur Scalzo.
ISSUE:
WON private respondent Arthur Scalzo can be sued provided his alleged diplomatic immunity conformably with the Vienna
Convention on Diplomatic Relations
RULING:
The SC DENIED the petition.
Conformably with the Vienna Convention, the functions of the diplomatic mission involve, the representation of the
interests of the sending state and promoting friendly relations with the receiving state. Only diplomatic agents, are
vested with blanket diplomatic immunity from civil and criminal suits. Indeed, the main yardstick in ascertaining whether a
person is a diplomat entitled to immunity is the determination of whether or not he performs duties of diplomatic nature.
Being an Attache, Scalzos main function is to observe, analyze and interpret trends and developments in their respective
fields in the host country and submit reports to their own ministries or departments in the home government. He is not
generally regarded as a member of the diplomatic mission. On the basis of an erroneous assumption that simply because
of the diplomatic note, divesting the trial court of jurisdiction over his person, his diplomatic immunity is contentious.
Under the related doctrine of State Immunity from Suit, the precept that a State cannot be sued in the courts of a foreign
state is a long-standing rule of customary international law. If the acts giving rise to a suit are those of a foreign
government done by its foreign agent, although not necessarily a diplomatic personage, but acting in his official capacity,
the complaint could be barred by the immunity of the foreign sovereign from suit without its consent. Suing a
representative of a state is believed to be, in effect, suing the state itself. The proscription is not accorded for the benefit of
an individual but for the State, in whose service he is, under the maxim par in parem, non habet imperium that all
states are sovereign equals and cannot assert jurisdiction over one another. The implication is that if the judgment against
an official would require the state itself to perform an affirmative act to satisfy the award, such as the appropriation of the
amount needed to pay the damages decreed against him, the suit must be regarded as being against the state itself,
although it has not been formally impleaded
A foreign agent, operating within a territory, can be cloaked with immunity from suit but only as long as it can be
established that he is acting within the directives of the sending state. The consent of the host state is an indispensable
requirement of basic courtesy between the two sovereigns.
The buy-bust operation and other such acts are indication that the Philippine government has given its imprimatur, if not
consent, to the activities within Philippine territory of agent Scalzo of the United States Drug Enforcement Agency. In
conducting surveillance activities on Minucher, later acting as the poseur-buyer during the buy-bust operation, and then
becoming a principal witness in the criminal case against Minucher, Scalzo hardly can be said to have acted beyond the
scope of his official function or duties.
CARABAO v ARGICULTURAL
Carabao filed a complaint to recover the sum of P238,500.00 representing the unpaid price of 300 units of fire
extinguishers sold and delivered by it to Agricultural Productivity Commission.
Agricultural moved for the dismissal of the case principally on the ground of the lower court's lack of jurisdiction over the
subject matter, alleging that under sections 2 and 3, Article XI of the Philippine Constitution, creating the General Auditing
Office as a constitutional office and defining its functions, in relation to Commonwealth Act No. 327 enacted in 1938 as an
implementing law, and under Rule 44 of the Revised Rules of Court, the settlement of money claims against the
Government of the Philippines has been placed under the exclusive original jurisdiction of the Auditor General to the
exclusion of courts of first instance, while the Supreme Court is vested with appellate jurisdiction over the Auditor
General's decision involving claims of private persons or entities SC held with this opinion
Arcega vs Court of Appleals
FACT:

Arcega, doing business under the firm name Fairmont Ice Cream Company, filed a complaint before the court against
the respondents Central Bank of the Philippines and Philippine National Bank, for the refund from allegedly unauthorized
payments made by her of the 17% special excise tax on foreign exchange.
The Central Bank moved to dismiss the complaint on the grounds, among others, that the trial court has no jurisdiction
over the subject-matter of the action, because the judgment sought will constitute a financial charge against the
Government, and therefore the suit is one against the Government, which cannot prosper without its consent, and in this
case no such consent has been given.
The petitioner appealed, but the court dismissed the complaint on the ground set forth in the Central Banks motion to
dismiss.
Arcega filed a motion for reconsideration of the resolution to which an opposition was filed by the Central Bank. This time,
the Central Bank submitted a certification that the balance of the collected special excise tax on sales of foreign exchange
was turned over to the Treasurer of the Philippines. Then the court denied the petitioners motion for reconsideration as a
result Arcega appealed to the Court of Appeals.
Holding that the suit is indirectly against the Republic of the Philippines which cannot be sued without its consent, the
Court of Appeals affirmed the dismissal of the complaint.
Finally the petitioner filed an appeal before the Supreme Court.
Issue: Whether a suit against the Central Bank for refund a suit against the State?
Held:
It is not a suit against the state. The charted of the Central Bank of the Philippines authorize that it can to sue and be
sued. The consent of the State to be sued, therefore, has been given.
RAYO vs. CFI of BULACAN
Facts:
1. During the height of typhoon Kading, the National Power Corporations plant superintendent Chavez opened
simultaneously all the three floodgates of the Angat Dam.
2. As a direct and immediate result, several towns in Bulacan were flooded (particularly Norzagaray). About a
hundred of its residents died and properties worth million of pesos were destroyed.
3. The petitioners, who are among the unfortunate victims of the man-caused flood, filed several complaints for
damages against NPC and the plant superintendent.
4. NPC claimed, as its defense, that in the operation of the Angat Dam, it is performing a purely governmental
function. Thus, it cannot be sued without the express consent of the State.
5. The petitioners opposed the claim of NPC and claimed that it is performing not governmental but merely
proprietary functions and that based on the organic charter (charter - a legal document that provides for the
creation of a corporate entity) of NPC, it can be sued and be sued in any court.

Issue: Whether or not the power of NPC to sue and be sued under its organic charter includes the power to be sued for
tort.

Held: The government has organized a private corporation, put money in it and has allowed it to sue and be sued in any
court under its charter. NPC, as a government owned and controlled corporation, has a personality of its own, distinct and
separate from that of the Government. In any court, NPC can sue and be sued for tort. The petition of the petitioners was
granted.

Notes:

Government-owned and controlled corporations have a personality of their own, separate and distinct from the
government. Therefore, although they are considered to be public in character, they are not exempt from garnishment
(legal proceedings).
MUNICIPALITY OF SAN FERNANDO, LA UNION vs. FIRME
G.R. No. L-52179 April 8, 1991
Facts:
A collision occurred involving a passenger jeepney owned by the Estate of MacarioNieveras, a gravel and sand truck
owned by Tanquilino Velasquez and a dump truck of the Municipality of San Fernando, La Union and driven by Alfredo
Bislig. Due to the impact, several passengers of the jeepney including LaureanoBania Sr. died as a result of the injuries
they sustained and four (4) others suffered varying degrees of physical injuries.
On December 11, 1966, the private respondents instituted a compliant for damages against the Estate of Macario
Nieveras and Bernardo Balagot, owner and driver, respectively, of the passenger jeepney. However, the aforesaid
defendants filed a Third Party Complaint against the petitioner and the driver of a dump truck of petitioner.
Petitioner filed its answer and raised affirmative defenses such as lack of cause of action, non-suability of the State,
prescription of cause of action and the negligence of the owner and driver of the passenger jeepney as the proximate
cause of the collision.
Respondent Judge Romeo N. Firme ordered defendants Municipality of San Fernando, La Union and Alfredo Bislig to pay,
jointly and severally, the plaintiffs for funeral expenses.
Private respondents stress that petitioner has not considered that every court, including respondent court, has the
inherent power to amend and control its process and orders so as to make them conformable to law and justice.
Issue: Whether or not the respondent court committed grave abuse of discretion when it deferred and failed to resolve the
defense of non-suability of the State amounting to lack of jurisdiction in a motion to dismiss.
Ruling:
Non-suability of the state.
The doctrine of non-suability of the State is expressly provided for in Article XVI, Section 3 of the Constitution, to wit: "the
State may not be sued without its consent."Consent takes the form of express or implied consent.
Municipal corporations, for example, like provinces and cities, are agencies of the State when they are engaged in
governmental functions and therefore should enjoy the sovereign immunity from suit. Nevertheless, they are subject to
suit even in the performance of such functions because their charter provided that they can sue and be sued.
"Suability depends on the consent of the state to be sued, liability on the applicable law and the established facts. The
circumstance that a state is suable does not necessarily mean that it is liable; on the other hand, it can never be held
liable if it does not first consent to be sued. Liability is not conceded by the mere fact that the state has allowed itself to be
sued. When the state does waive its sovereign immunity, it is only giving the plaintiff the chance to prove, if it can, that the
defendant is liable."
Anent the issue of whether or not the municipality is liable for the torts committed by its employee, the test of liability of the
municipality depends on whether or not the driver, acting in behalf of the municipality, is performing governmental or
proprietary functions.
RP v NLRC
the full ownership of PNEI was transferred to its creditor, the National Investment Development Corporation (NIDC), a
subsidiary of the Philippine National Bank (PNB),[6] following the latters foreclosure of PNEI assets.

APT has tahen over tha managament of PNEI. However due to the financial decline of the company retreachment of 500
employees have been approved
The filing of various labor complaints against PNEI was the immediate result. The labor Arbiter order for a position paper
from the PNEI but APT did not comply. Labor arbiter decided in favor of the complaints
Upon public auction of the PNEI property. ATP moved to quash execution of the oorder of the NLRC.
SC held that Delving into the other crucial question, it would be, in our view, a clear and patent error to construe the
decreed joint and solidary liability of APT (in NLRC NCR Case No. 00-08-05380-93) as extending beyond what APT has
held in or acquired from PNEI. A matter that mus not be overlooked is the fact that the inclusion of APT as a respondent in
the monetary claims against PNEI is merely the consequence of its being a conservator of assets, a role that APT
normally plays in, or the relationship that ordinarily it maintains with, corporations identified for and while under
privatization. The liability of APT under this particular arrangement, nothing else having been shown, should be coextensive with the amount of assets taken over form the privatized firm. PNEIs assets obviously remain to be subject to
execution by judgment creditors of PNEI. Accordingly, the levy and auction sale of the property of PNEI to satisfy the
monetary judgment rendered in favor of PNEI employees can be sustained since such assets are to be deemed subject to
all valid claims against PNEI

DOH v CANchela
Petitioner entered into three Owner-Consultant Agreements (Agreements) with private respondents covering infrastructure
projects for the Baguio General Hospital and Medical Center (Baguio Project), the Batangas Regional Hospital (Batangas
Project) and the Corazon L. Montelibano Memorial Regional Hospital in Bacolod City (Bacolod Project)
The Agreements, which contained almost identical language, required the preparation by private respondents of the
following documents: detailed architectural and engineering design plans; technical specifications and detailed estimates
of cost of construction of the hospital, including the preparation of bid documents and requirements; and construction
supervision until completion of hand-over and issuance of final certificate.
During the construction of the projects, various deficiencies in the performance of the agreed scope of private
respondents work were allegedly discovered[17] which were not, however, communicated to private respondents.[18] Due
to such deficiencies, petitioner withheld payment of the consultancy fees due to private respondents. And petitioner did
not return the documents, plans, specifications and estimates submitted by private respondents.
As despite written demands for payment,[19] petitioner continued to withhold payment of their professional fees, private
respondents appealed, by letter dated August 29, 1997, to then Department of Health Secretary Carmencita C. Reodica,
they stating that their appeal was purposely done as our ultimate administrative remedy before resorting to arbitration
under E.O. 1008.
SC held that
Even if each of the Agreements did not incorporate the provision calling for compliance with the above-said Codes, the
provisions thereof, as well as those of the 1987 Constitution and LOI No. 968, must be deemed to form part of, and coexist with, the Agreements. Applicable peremptory provisions of law of this nature, affecting as they do public policy or
impressed as they are with public interest, are held to be written into the contract.[58]
The illegality of the subject Agreements proceeds, it bears emphasis, from an express declaration or prohibition by law,
[59] not from any intrinsic illegality. As such, the Agreements are not illegal per se[60] and the party claiming thereunder
may recover what had been paid or delivered.[61]
The Court thus finds that private respondents are entitled to be compensated for the services they actually performed for
the benefit of petitioner, as shown by petitioners acceptance and use[62] of the complete Contract or Bid Documents
including the A & E Design Plans and Technical Specifications and the Detailed Cost Estimates for each project that
private respondents promptly submitted, as in fact petitioner itself recommends that private respondents be paid therefor.
The compensation must, however, exclude services for periodic visits which the records irrefutably show not to have been
rendered.
USA v. GUINTO
182 SCRA 644

FACTS:
The cases have been consolidated because they all involve the doctrine of state immunity. In GR No. 76607, private
respondents re suing several officers of the US Air Force in connection with the bidding for barbering services in Clark Air
Base. In GR No. 80018, Luis Bautista was arrested following a buy-bust operation for violation of the Dangerous Drugs
Act. Bautista then filed a complaint for damages claiming that because of the acts of the respondents, he lost his job. In
GR No. 79470, Fabian Genove filed a complaint for damages against petitioner for his dismissal as cook in the US Air
Force. In GR No. 80258, complaint for damage was filed by the respondents against petitioners for injuries allegedly
sustained by plaintiffs. All cases invoke the doctrine of state immunity as ground to dismiss the same.
ISSUE:
Are the petitioners immune from suit?
HELD:
It is clear that the petitioners in GR No. 80018 were acting in the exercise of their official functions. They cannot be directly
impleaded for the US government has not given its consent to be sued. In GR No. 79470, petitioners are not immune for
restaurants are commercial enterprises, however, claim of damages by Genove cannot be allowed on the strength of the
evidence presented. Barber shops are also commercial enterprises operated by private persons, thus, petitioners in GR
No. 76607 cannot plead any immunity from the complaint filed. In GR No. 80258, the respondent court will have to receive
the evidence of the alleged irregularity in the grant of the barbershop concessions before it can be known in what capacity
the petitioners were acting at the time of the incident.
SPOUSES FONTANILLA VS HON. MALIAMAN,
FACTS: National Irrigation Administration (NIA), a government agency, was held liable for damages resulting to the death
of the son of herein petitioner spouses caused by the fault and/or negligence of the driver of the said agency. NIA
maintains that it is not liable for the act of its driver because the former does not perform primarily proprietorship functions
but governmental functions.
ISSUE: Whether or not NIA may be held liable for damages caused by its driver.
HELD: Yes. NIA is a government agency with a corporate personality separate and distinct from the government, because
its community services are only incidental functions to the principal aim which is irrigation of lands, thus, making it an
agency with proprietary functions governed by Corporation Law and is liable for actions of their employees.
PHILIPPINE REFINING COMPANY v. CA
FACTS:
This is an appeal by certiorari from the decision of respondent Court of Appeals 1 affirming the decision of the Court of Tax
Appeals which disallowed petitioner's claim for deduction as bad debts of several accounts in the total sum of
P395,324.27, and imposing a 25% surcharge and 20% annual delinquency interest on the alleged deficiency income tax
liability of petitioner.
Petitioner Philippine Refining Company (PRC) was assessed by respondent Commissioner of Internal Revenue
(Commissioner) to pay a deficiency tax for the year 1985 in the amount of P1,892,584.00.
The assessment was timely protested by petitioner on April 26, 1989, on the ground that it was based on the erroneous
disallowances of "bad debts" and "interest expense" although the same are both allowable and legal deductions.
Respondent Commissioner, however, issued a warrant of garnishment against the deposits of petitioner at a branch of
City Trust Bank, in Makati, Metro Manila, which action the latter considered as a denial of its protest.
Petitioner accordingly filed a petition for review with the Court of Tax Appeals (CTA) on the same assignment of error, that
is, that the "bad debts" and "interest expense" are legal and allowable deductions.In its decision 3 of February 3, 1993 in
C.T.A. Case No. 4408, the CTA modified the findings of the Commissioner by reducing the deficiency income tax
assessment to P237,381.26, with surcharge and interest incident to delinquency. In said decision, the Tax Court reversed
and set aside the Commissioner's disallowance of the interest expense of P2,666,545.19 but maintained the disallowance
of the supposed bad debts of thirteen (13) debtors in the total sum of P395,324.27.
Petitioner then elevated the case to respondent Court of Appeals which, as earlier stated, denied due course to the
petition for review and dismissed the same on August 24, 1994.the reason of the court was that Out of the sixteen (16)
accounts alleged as bad debts, We find that only three (3) accounts have met the requirements of the worthlessness of
the accounts, hence were properly written off as: bad debts.Mere testimony of the Financial Accountant of the Petitioner

explaining the worthlessness of said debts is seen by this Court as nothing more than a self-serving exercise which lacks
probative value. There was no iota of documentary evidence (e.g., collection letters sent, report from investigating
fieldmen, letter of referral to their legal department, police report/affidavit that the owners were bankrupt due to fire that
engulfed their stores or that the owner has been murdered. etc.), to give support to the testimony of an employee of the
Petitioner. Mere allegations cannot prove the worthlessness of such debts in 1985. Hence, the claim for deduction of
these thirteen (13) debts should be rejected.
ISSUE: WON all bad debts should be treated as deductions.
RULING:
This pronouncement of respondent Court of Appeals relied on the ruling of this Court in Collector vs. Goodrich
International Rubber Co., 6 which established the rule in determining the "worthlessness of a debt." In said case, we held
that for debts to be considered as "worthless," and thereby qualify as "bad debts" making them deductible, the taxpayer
should show that (1) there is a valid and subsisting debt. (2) the debt must be actually ascertained to be worthless and
uncollectible during the taxable year; (3) the debt must be charged off during the taxable year; and (4) the debt must arise
from the business or trade of the taxpayer. Additionally, before a debt can be considered worthless, the taxpayer must also
show that it is indeed uncollectible even in the future.
Furthermore, there are steps outlined to be undertaken by the taxpayer to prove that he exerted diligent efforts to collect
the debts, viz.: (1) sending of statement of accounts; (2) sending of collection letters; (3) giving the account to a lawyer for
collection; and (4) filing a collection case in court.
On the foregoing considerations, respondent Court of Appeals held that petitioner did not satisfy the requirements of
"worthlessness of a debt" as to the thirteen (13) accounts disallowed as deductions.
It appears that the only evidentiary support given by PRC for its aforesaid claimed deductions was the explanation or
justification posited by its financial adviser or accountant, Guia D. Masagana. Her allegations were not supported by any
documentary evidence, hence both the Court of Appeals and the CTA ruled that said contentions per se cannot prove that
the debts were indeed uncollectible and can be considered as bad debts as to make them deductible. That both lower
courts are correct is shown by petitioner's own submission and the discussion thereof which we have taken time and
patience to cull from the antecedent proceedings in this case, albeit bordering on factual settings.
The contentions of PRC that nobody is in a better position to determine when an obligation becomes a bad debt than the
creditor itself, and that its judgment should not be substituted by that of respondent court as it is PRC which has the
facilities in ascertaining the collectibility or uncollectibility of these debts, are presumptuous and uncalled for. The Court of
Tax Appeals is a highly specialized body specifically created for the purpose of reviewing tax cases. Through its expertise,
it is undeniably competent to determine the
issue of whether or not the debt is deductible through the evidence presented before it. 8
Because of this recognized expertise, the findings of the CTA will not ordinarily be reviewed absent a showing of gross
error or abuse on its part. 9 The findings of fact of the CTA are binding on this Court and in the absence of strong reasons
for this Court to delve into facts, only questions of law are open for determination. 10 Were it not, therefore, due to the
desire of this Court to satisfy petitioner's calls for clarification and to use this case as a vehicle for exemplification, this
appeal could very well have been summarily dismissed.
Republic v. Purisima 78 SCRA 470 [1977]
Facts: Respondent Judge Amante P. Purisima of the Court of First Instance of Manila failed to apply the well-known and
of-reiterated doctrine of the non-suability of a State, including its offices and agencies, from suit without its consent. it was
so alleged in a motion to dismiss filed by defendant Rice and Corn Administration in a pending civil suit in the sala of
respondent Judge for the collection of a money claim arising from an alleged breach of contract, the plaintiff being private
respondent Yellow Ball Freight Lines, Inc.. Such a motion to dismiss was filed on September 7, 1972. At that time, the
leading case of Mobil Philippines Exploration, Inc. v. Customs Arrastre Service, were Justice Bengzon stressed the lack of
jurisdiction of a court to pass on the merits of a claim against any office or entity acting as part of the machinery of the
national government unless consent be shown, had been applied in 53 other decisions. There is thus more than sufficient
basis for an allegation of jurisdiction infirmity against the order of respondent Judge denying the motion to dismiss dated
October 4, 1972. What is more, the position of the Republic has been fortified with the explicit affirmation found in this
provision of the present Constitution: "The State may not be sued without its consent."
Issue: Whetherthe government has waived its immunity from suit.
Held: Apparently respondent Judge was misled by the terms of the contract between the private respondent, plaintiff in his
sala, and defendant Rice and Corn Administration which, according to him, anticipated the case of a breach of contract
within the parties and the suits that may thereafter arise. The consent, to be effective though, must come from the State
acting through a duly enacted statute as pointed out by Justice Bengzon in Mobil. Thus, whatever counsel for defendant

Rice and Corn Administration agreed to had no binding force on the government. That was clearly beyond the scope of his
authority. At any rate, Justice Sanchez, in Ramos v. Court of Industrial Relations, was quite categorical as to its "not
[being] possessed of a separate and distinct corporate existence. On the contrary, by the law of its creation, it is an office
directly 'under the Office of the President of the Philippines."
Santiago vs Republic 87 SCRA 294 [1978]
Facts:Petitioner Ildefonso Santiago donated a parcel of land to the Bureau of Plant Industry on the terms that the Bureau
should construct a building and install lighting facilities on the said lot.
When time passed and there were still no improvements on the lot, Santiago filed a case pleading for the revocation of
such contract of donation but the trial court dismissed the petition claiming that it is a suit against the government and
should not prosper without the consent of the government.
Issue:Whether the respondent government has waived its immunity from suit.
Held: Yes, The government's waiver of immunity was implied by virtue of the terms provided in the deed of donation. The
government is a beneficiary of the terms of the donation but it did not comply with such terms. Thus, the donor Santiago
has the right to be heard in the court. Also, to not allow the donor to be heard would be unethical and contrary to equity
which the government so advances. The Court of First Instance is hereby directed to proceed with the case.
Traders Royal Bank v. IAC 192 SCRA 305 [1990]
Fact:
Main Point: Immunity from suit may be waived by implied consent to be sued as when the state enters into a contract that
is proprietary in character. The evidence is that such is the contract here. Hence, immunity was effectively waived.
Republic v. Sandoval 220 SCRA 124 [1993]
Facts:The heirs of the deceased of the January 22, 1987 Mendiola massacre, together with those injured (Caylao group),
instituted the petition, seeking the reversal and setting aside of the orders of respondent Judge Sandoval (May 31 and
Aug 8, 1988) in "Erlinda Caylao, et al. vs. Republic of the Philippines, et al." which dismissed the case against the
Republic of the Philippines. May 31 order: Because the impleaded military officers are being charged in their personal and
official capacity, holding them liable, if at all, would not result in financial responsibility of the government. It was this
portion that petitioners (Caylao group) invoke in their claim for damages from the government. No concrete form of
compensation was received by the victims. On January, 1988, petitioners instituted an action for damages against the
Republic of the Philippines, together with the military officers, and personnel involved in the Mendiola incident. Petitioners
said that the State has waived its immunity from suit. Judge Sandoval dismissed the case on the ground that there was no
such waiver
Issues:Whether the State has waived its immunity from suit (i.e. Whether or not this is a suit against the State with its
consent)
Held: No. This is not a suit against the State with its consent.Art. XIV, Sec. 3, 1987 Constitution: The State may not be
sued without its consent.The recommendations by the Commission does not in any way mean that liability automatically
attaches to the State. The Commission was simply a fact-finding body; its findings shall serve only as cause of action for
litigation; it does not bind the State immediately. President Aquino's speeches are likewise not binding on the State; they
are not tantamount to a waiver by the State
Some instances when a suit against the State is proper:
1
When the Republic is sued by name;
2
When the suit is against an unincorporated government agency
3
When the suit is on its face against a government officer but the case is such that the ultimate liability will
belong not to the officer but to the government
Although the military officers and personnel were discharging their official functions during the incident, their functions
ceased to be official the moment they exceeded their authority. There was lack of justification by the government forces in
the use of firearms.Their main purpose in the rally was to ensure peace and order, but they fired at the crowd instead
No reversible error by the respondent Judge found. Petitions dismissed.
Delos Santos v. IAC 223 SCRA 11 [1993]
DA v. NLRC 227 SCRA 693 [1993]
Facts: Petitioner Department of Agriculture (DA) and Sultan Security Agency entered into a contract for security services
to be provided by the latter to the said governmental entity. Pursuant to their arrangements, guards were deployed by
Sultan Security Agency in the various premises of the DA. Thereafter, several guards filed a complaint for underpayment
of wages, nonpayment of 13th month pay, uniform allowances, night shift differential pay, holiday pay, and overtime pay,
as well as for damages against the DA and the security agency.

The Labor Arbiter rendered a decision finding the DA jointly and severally liable with the security agency for the payment
of money claims of the complainant security guards. The DA and the security agency did not appeal the decision. Thus,
the decision became final and executory. The Labor Arbiter issued a writ of execution to enforce and execute the
judgment against the property of the DA and the security agency. Thereafter, the City Sheriff levied on execution the motor
vehicles of the DA.
Issue: Whether or not the doctrine of non-suability of the State applies in the case
Held: The basic postulate enshrined in the Constitution that the State may not be sued without its consent reflects
nothing less than a recognition of the sovereign character of the State and an express affirmation of the unwritten rule
effectively insulating it from the jurisdiction of courts. It is based on the very essence of sovereignty. A sovereign is exempt
from suit based on the logical and practical ground that there can be no legal right as against the authority that makes the
law on which the right depends.
The rule is not really absolute for it does not say that the State may not be sued under any circumstances. The State may
at times be sued. The States consent may be given expressly or impliedly. Express consent may be made through a
general law or a special law. Implied consent, on the other hand, is conceded when the State itself commences litigation,
thus opening itself to a counterclaim, or when it enters into a contract. In this situation, the government is deemed to have
descended to the level of the other contracting party and to have divested itself of its sovereign immunity.
But not all contracts entered into by the government operate as a waiver of its non-suability; distinction must still be made
between one which is executed in the exercise of its sovereign function and another which is done in its proprietary
capacity. A State may be said to have descended to the level of an individual and can this be deemed to have actually
given its consent to be sued only when it enters into business contracts. It does not apply where the contract relates to the
exercise of its sovereign functions.
In the case, the DA has not pretended to have assumed a capacity apart from its being a governmental entity when it
entered into the questioned contract; nor that it could have, in fact, performed any act proprietary in character.
But, be that as it may, the claims of the complainant security guards clearly constitute money claims. Act No. 3083 gives
the consent of the State to be sued upon any moneyed claim involving liability arising from contract, express or implied.
Pursuant, however, to Commonwealth Act 327, as amended by PD 1145, the money claim must first be brought to the
Commission on Audit.
EPG v. Sec. of DPWH 354 SCRA 566 [2001]
Facts:
In 1983, the Ministry of Human Settlement (MHS), through the BLISS Development Corporation, intiated a housing project
on a government property along the east bank of Manggahan Floodway in Pasig. The MHS entered into a Memorandum
of Agreement (MOA) with Ministry of Public Works and Highways (MPWH) where the latter undertook to develop the
housing site and construct thereon 145 housing units. By virtue of the MOA, MPWH forged individual contracts with
petitioners EPG, Ciper, Septa, Phil. Plumbing, Home Construction, World Builders, Glass World, Performance Builders,
and De Leon Araneta Construction for the construction of the housing units.Under the contracts, the scope of construction
and funding covered only around "2/3 of each housing unit Petitioners agreed to undertake and perform "additional
constructions" for the completion of the housing units despite the fact that there was only a verbal promise, and not a
written contract, by the MPWH Undersecretary Aber Canlas that additional funds will be available and forthcoming.
Unpaid balance for the additional constructions amounted to P5,918,315.63
Upon a demand letter from the petitioners, on November 14, 1988, DPWH Asst. Secretary Madamba opined that payment
of petitioners' money claims should be based on quantum meruit (what one has earned) and should be forwarded to the
Commission on Audit (COA)
In a Letter of the Undersecretary of Budget and Management dated December 20, 1994, the amount of P5,819,316.00
was then released for the payment of the petitioners' money claims under Advise of Allotment No. A4-1303-04-41-303
In an indorsement dated December 27, 1995, the COA referred anew the money claims to the DPWH. In a letter dated
August 26, 1996, respondent Secretary Gregorio Vigilar denied the subject money claims. Petitioners filed before the RTC
of QC, Branch 226 a Petition for Mandamus to order the respondent to pay petitioners their money claims plus damages
and attorney's fees.Lower court denied the petition on February 18, 1997
Issue:Whether or not the State is immune from suit
Holding:No, While the court agrees with the respondent that the implied contracts are void, in view of violation of
applicable laws, auditing rules, and lack of legal requirements, it still finds merit in the instant petitionThe illegality of the
implied contracts proceeds from an express declaration or prohibition by law, not from any intrinsic illegality"in the interest
of substantial justice," petitioners-contractors' right to be compensated is upheld, applying the principle of quantum
meritEven the DPWH Asst. Sec. for Legal Affairs recommends their compensation; even the DPWH Auditor did not object
to the payment of the money claims.The respondent may not conveniently hide under the State's cloak of invincibility

against suit, considering that this principle yields to certain settled exceptions.The State's immunity cannot serve as an
instrument perpetrating injustice. Petition granted. RTC decision reversed and set aside.
Resulting Liability
Philrock v. Board of Liquidators 180 SCRA 171 [1989]
Main Point: The Reparation Commission, a government agency with no juridical personality, enjoyed the governments
immunity from suit. The sale contract entered into with PHILROCK was merely incidental to the performance of its
governmental function. Hence, it was not a waiver of immunity. Moreover, even if REPACOM is deemed to have waived its
immunity by entertaining the suit of PHILROCK, governments consent to be sued does not mean it concedes liability.
Without an act of Congress appropriating the amount due, there can be no disbursement of public fund.
Facts:
PHILROCK filed in the RTC of Manila a complaint against the Board of Liquidators, as liquidator of the defunct
REPACOM, for: (1) the replacement of the defective rock pulverizing machinery purchased from REPACOM, or, as
alternative, to refund the purchase at 31% of its contract price; (2) reparation for losses incurred due to the increased
expenses of maintaining the plant at Php5,000 a month and Php4,000 per day as unrealized profits and exemplary
damages; and (3) Php50,000 attorney fees plus expenses and costs of the suit.
The RTC decided in favor of PHILROCK. The Solicitor General, in behalf of the State, filed a notice of appeal on the
ground that the payment for damages are public funds, hence, exempt from attachment and execution. Nevertheless, the
RTC judge issued a Writ of Execution. Subsequently the Board of Liquidators filed a petition for certiorari and prohibition
in the Court of Appeals where the Court of Appeals set aside the Writ of Execution by the RTC. Hence, this petition for
review.
Issue: Whether or not the Board of Liquidators, as a government agency without juridical capacity, may be sued and held
liable as litigators of REPACOM.
Ruling:No. The Board of Liquidators is a government agency, created under E.O. 372 to administer the assets and pay the
liabilities of the defunct REPACOM, thus it has no juridical personality, separate and distinct from the government, and
therefore, as a general rule, suing it is akin to suing the State. The State enjoys immunity from suit except when it
conducts business through a government-owned and controlled corporation or a non-corporate agency set up primarily for
a business purpose, and even then, the State may not be liable for damages since the purse of the State, or the
disbursement of public funds is in the discretion of the Legislature. The functions and public services rendered by the
State cannot be allowed to be paralyzed or disrupted by the diversion of public funds from their legitimate specific
objectives, as appropriated by law. Although the liability of REPACOM has been ascertained, the State is at liberty to
determine for itself how to satisfy such liability. Funds should be appropriated by the Legislature for the specific purpose of
satisfying the judgement in favor of PHILROCK before said judgement may be paid.
Liang v. People GR 125865 [January 28, 2000] ADB immunity)
Facts:Two criminal informations for grave oral defamation were filed against Liang, a Chinese national who was employed
asan Economist by the Asian Development Bank (ADB), by his secretary Joyce Cabal, before the MeTC Mandaluyong
City.The MeTC, acting pursuant to an advice from the DFA that Liang enjoyed immunity from legal processes, dismissed
thecriminal informations against him. The RTC Pasig City annulled and set aside the MeTCs dismissal. Hence, Liang filed
apetition for review before the SC which was denied ruling that the immunity granted to officers and staff of the ADB is
notabsolute; it is limited to acts performed in an official capacity. Hence, the present MR.
Issue:WON Liang is immune from suit.
Held:No.The Court found no reason to disturb the earlier decision.The slander of a person, by any stretch, cannot
beconsidered as falling within the purview of the immunity granted to ADB officers and personnel.The issue of whether or
not Liangs utterances constituted oral defamation is still for the trial court to determineJ. Punos concurring opinion:Liang
contends that a determination of a person's diplomatic immunity by the DFA is apolitical question. It is solely within
theprerogative of the executive department and is conclusive upon the courts. Furthermore, the immunity conferred under
the ADBCharter and the Headquarters Agreement is absolute. It is designed to safeguard the autonomy and
independence of international organizations against interference from any authority external to the organizations. It is
necessary to allow suchorganizations to discharge their entrusted functions effectively. The only exception to this
immunity is when there is an implied or express waiver or when the immunity is expressly limited by statute. The
exception allegedly has no application to the case atbar."
It is a recognized principle of international law and under our system of separation of powers that diplomatic immunityis
essentially a political questionand courts should refuse to look beyond a determination by the executive

branchthegovernment, and where the plea of diplomatic immunity is recognized and affirmed by the executive branch of
the governmentas in the case at bar, it is then the duty of the courts to accept the claim of immunity upon appropriate
suggestion by theprincipal law officer of the government, the Solicitor General in this case, or other officer acting under his
direction. Hence, inadherence to the settled principle that courts may not so exercise their jurisdiction by seizure and
detention of property, as toembarrass the executive arm of the government in conducting foreign relations, it is accepted
doctrine that in such cases the judicial department of the government follows the action of the political branch and will not
embarrass the latter by assuming anantagonistic jurisdiction."
Liang, a bank official of ADB, is not entitled to diplomatic immunity and hence his immunity is not absolute.Under
theVienna Convention on Diplomatic Relations, a diplomatic envoy is immune from criminal jurisdiction of the receiving
State for allacts, whether private or official, and hence he cannot be arrested, prosecuted and punished for any offense he
may commit,unless his diplomatic immunity is waived. On the other hand, officials of international organizations enjoy
"functional" immunities,that is, only those necessary for the exercise of the functions of the organization and the fulfillment
of its purposes. This is thereason why the ADB Charter and Headquarters Agreement explicitly grant immunity from legal
process to bank officers andemployees only with respect to acts performed by them in their official capacity, except when
the Bank waives immunity. In other words, officials and employees of the ADB are subject to the jurisdiction of the local
courts for their private acts, notwithstandingthe absence of a waiver of immunity.
Liang cannot also seek relief under the mantle of "immunity from every form of legal process" accorded to ADB as
aninternational organization.The immunity of ADB is absolute whereas the immunity of its officials and employees is
restrictedonly to official acts. He stands charged of grave slander for allegedly uttering defamatory remarks against his
secretary.Considering that the immunity accorded to petitioner is limited only to acts performed in his official capacity, it
becomesnecessary to make a factual determination of whether or not the defamatory utterances were made pursuant and
in relation tohis official functions as a senior economist.
Republic v. Hidalgo 477 SCRA 12 [2005] (writ execution)
FACTS:Tarcila Laperal Mendoza filed an actionfor the annulment or declaration of nullity of the title and deed of sale,
reconveyance and/orrecovery of ownership and possession aproperty against the Republic of thePhilippinesin the RTC of
Manila.It is also known as the Arlegui Residencewhich housed two Philippinepresidents and which now holds the Office of
the Press Secretary and the News InformationBureau.The case was initially dismissed by thepresiding Judge of the
Manila RTC (Branch 35)on the ground of state immunity. The case wasre-raffled to the Manila RTC (Branch 37),
withrespondent Vicente A. Hidalgo as presidingJudge. In an Order, Judge Hidalgo declared theRepublic in default for
failure of SolicitorGabriel Francisco Ramirez, the handlingsolicitor, to file the required Answer within theperiod prayed for
in his motion for extension.It is contended that the respondentJudge violated the Constitution and thefundamental rule that
government funds areexempt from execution or garnishment whenhe caused the issuance of the writ of executionagainst
the Republic.
ISSUE: WON the Republic can invoke immunityfrom suit.
HELD:It is settled that when the State givesits consent to be sued, it does not therebynecessarily consent to an
unrestrainedexecution against it. Tersely put, when theState waives its immunity, all it does, in effect,is to give the other
party an opportunity toprove, if it can, that the state has a liability.The functions and public servicesrendered by the State
cannot be allowed toparalyzed or disrupted by the diversion of public funds from their legitimate and specificobjects, as
appropriated by law
Phil Agila Satellite v. Lichauco
PASI was established by a consortium of private telecommunications carriers which in 1994 had entered into a
Memorandum of Understanding with the DOTC, through its then Secretary Jesus Garcia, concerning the planned launch
of a Philippine-owned satellite into outer space. PASI averred that after having secured the confirmation from the
Philippine government, it proceeded with preparations for the launching, operation and management of its satellites,
including the availment of loans, the increase in its capital.
However, respondent Lichauco, then DOTC Undersecretary for Communications, allegedly embarked on a crusade to
malign the name of Michael de Guzman (one lf the owner of PASI) and sabotage the business of PASI. Aggrieved by
Lichaucos actions, PASI and De Guzman instituted a civil complaint against Lichauco, by then the Acting Secretary of the
DOTC.
ISSUE Is the suit one against the state?
RULING the hornbook rule is that a suit for acts done in the performance of official functions against an officer of the
government by a private citizen that would result in a charge against or financial liability to the government must be

regarded as a suit against the State itself, although the latter has not been formally impleaded. However, government
immunity from suit will not shield the public official being sued if the government no longer has an interest to protect in the
outcome of a suit; or if the liability of the officer is personal because it arises from a tortious act in the performance of his
duties.
RATIONALE: When a public officer acts without or in excess of jurisdiction, any injury caused by him is his own personal
liability and cannot be imputed to the State. (p.34, Political Law, Isagani Cruz)
CURATA v PHILIPPINE PORTS AUTHORITY, \
EO 385, and EO 431, delineated the BPZ and placed it under the PPA for administrative jurisdiction of its proper zoning,
planning, development, and utilization. PPA filed cases before RTC for the expropriation of a number of lots.
While procedural and collateral issues abound, central to these petitions, however, is the matter of just compensation for
the lots sought to be expropriated by PPA for the Batangas Port Zone (BPZ). The problem arises on what of the basis of
the payment for the lot owners.
SC held that the PPA should pay the lot owners with the zonal value of the land to be expropriated, as just compensation
for the lands
UP v. Dizon, G.R. No. 171182, August 23, 2012, 679 SCRA 54
FACTS: On August 30, 1990, UP entered into an agreement with Stern builders Corp for the construction of extension
building in UPLB. Stern Builders submitted 3 billings but UP only paid for 2, the 3rd was not paid due to disallowance of
COA. When the disallowance was lifted, UP still failed to pay. . The third billing worth was not paid due to its disallowance
by the Commission on Audit (COA). Despite the lifting of the disallowance, the UP failed to pay the billing, prompting Stern
Builders and dela Cruz to sue the UP and its correspondent officials to collect the unpaid billing and to recover various
damages .RTC rendered judgement ordering UP to pay the third billing, actual and moral damages, attorneys fees and
cost of suit. UP filed a motion for reconsideration which was denied. RTC issued a writ of execution for the garnishment of
UPs depository banks.UP filed petition for certiorari with CA. CA rendered judgment against UP .
ISSUE: WON CA committed grave error in allowing garnishment of state universitys funds .
Held : Yes. Despite its establishment as a body corporate,64 the UP remains to be a chartered institution performing a
legitimate government function. The funds of the UP are government funds that are public in character. Hence, the funds
subject of this action could not be validly made the subject of the RTCs writ of execution or garnishment. suability of the
State did not necessarily mean its liability. The UP correctly submits here that the garnishment of its funds to satisfy the
judgment awards of actual and moral damages (including attorneys fees) was not validly made if there was no special
appropriation by Congress to cover the liability. An appropriation by Congress was required before the judgment that
rendered the UP liable for moral and actual damages. Orders for garnishment of funds of UP is annulled.
The universal rule that where the State gives its consent to be sued by private parties either by general or special law, it
may limit claimants action only up to the completion of proceedings anterior to the stage of execution and that the power
of the Courts ends when the judgment is rendered, since government funds and properties may not be seized under writs
of execution or garnishment to satisfy such judgments, is based on obvious considerations of public policy. Disbursements
of public funds must be covered by the corresponding appropriation as required by law. The functions and public services
rendered by the State cannot be allowed to be paralyzed or disrupted by the diversion of public funds from their legitimate
and specific objects, as appropriated by law.
Sec 6 Police Force
The State shall establish and maintain one police force, which shall be national in scope and civilian in character, to be
administered and controlled by a national police commission. The authority of local executives over the police units in
their jurisdiction shall be provided by law.

Quilonia vs Gen Court Martial G.R. No. 96607 March 4, 1992


FACTS : Petitioner was assigned at the Western Police District (WPD), was charged before respondent General Court
Martial with the crime of murder on two (2) counts. He wrote Pres. Corazon Aquino to be tried by a civilian court and to
seek a waiver of a military jurisdiction. petitioner, through counsel, wrote a letter 1 to President Corazon C. Aquino,
expressing his desire to be tried by a civilian court and sought a waiver of a military jurisdiction, for the reason, among
others, that the "enactment of the Philippine National Police Law RA. 6975 creates his honest belief that he should now be
under the actual and real jurisdiction of a civilian court. 28 December 1990 Respondent court, however, proceeded to

read the charges and specifications to petitioner. Petitioner has filed this petition for certiorari and prohibition with
preliminary injunction and/or restraining order, alleging that respondent court acted with grave abuse of discretion
ISSUE: WON General Court Martial acted with grave abuse of discretion.
HELD : Yes. Although Republic Act No. 6975 was not yet in effect when petitioner was arraigned on 28 December 1990,
nevertheless, respondent court martial knew or should have known that the said Act had already been signed or approved
by the President on 13 December 1990 and that the same was published in two (2) national newspaper of general
circulation on 17 December 1990 and that it would take effect on 1 January 1991. It is precisely for this reason that
respondent court martial decided to have the petitioner's motion to inhibit argued on 28 December 1990 and thereafter
arraigned the petitioner on the same day despite his vehement refusal to enter a plea.
The civilian character with which the PNP is expressly invested is declared by RA 6975 as paramount, and, in line
therewith, the law mandates the transfer of criminal cases against its members to civilian courts. By closing its eyes to the
provisions of RA 6975, indelicately asserting its military jurisdiction rather than letting go of the case to civilian jurisdiction
to effectuate and give flesh to the avowed policy and intent of the law, respondent Court committed grave abuse of
discretion

Carpio v. Executive Secretary, G.R. No. 96409, 14 February 1992, 206 SCRA 290, 302

FACTS : Petitioner Antonio Carpio as citizen, taxpayer and member of the Philippine Bar, filed this petition, questioning
the constitutionality of RA 6975 with a prayer for TRO.

RA 6875, entitled AN ACT ESTABLISHIGN THE PHILIPPINE NATIONAL POLICE .. allegedly contravened Art. XVI, sec.
6 of the Constitution. Limited the power of PNP to administrative control . There is usurpation in the power of control of
NAPOLCOM.

ISSUE: Whether or not RA 6975 is contrary to the Constitution

HELD : No. NAPOLCOM and the PNP are placed under the reorganized Department of Interior and Local Government is
merely an administrative realignment that would bolster a system of coordination and cooperation. Such organizational
set-up does not detract from the mandate of the Constitution that the national police force shall be administered and
controlled by a national police commission as at any rate

There is no usurpation of the power of control of the NAPOLCOM . it is clear that the local executives are only acting as
representatives of the NAPOLCOM. As such deputies, they are answerable to the NAPOLCOM for their actions in the
exercise of their functions under that section. Thus, unless countermanded by the NAPOLCOM, their acts are valid and
binding as acts of the NAPOLCOM. It is significant to note that the local officials, as NAPOLCOM representatives, will
choose the officers concerned from a list of eligibles (those who meet the general qualifications for appointment to the
PNP) 30 to be recommended by PNP officials. The same holding is true with respect to the contention on the operational
supervision and control exercised by the local officials. Those officials would simply be acting as representatives of the
Commission. The grant of disciplinary powers over PNP members to the "People's Law Enforcement Boards" (or the
PLEB) and city and municipal mayors is also not in derogation of the commission's power of control over the PNP.As a

disciplinary board primarily created to hear and decide citizen's complaints against erring officers and members of the
PNP, the establishment of PLEBs in every city, and municipality would all the more help professionalize the police force.

Dept of Budget vs Manilas Finest Retirees Asso GR 169466 2007


FACTS : R.A. No. 6975 reengineered the retirement scheme in the police organization. PNP personnel, under the new
law, stood to collect more retirement benefits than what INP members of equivalent rank, who had retired under the INP
Law, received. All INP retirees, spearheaded by the Manilas Finest filed a petition for declaratory relief. Pray for :
DECLARATORY JUDGMENT be rendered in their favor, DECLARING with certainty that they, as INP-retirees, are truly
absorbed and equally considered as PNP-retirees and thus, entitled to enjoy the SAME or IDENTICAL retirement benefits
being bestowed to PNP-retirees by virtue of said PNP Law. RTC rendered decision in favour of Manilas Finest et al. CA
affirmed decision of RTC. DBM et al comes to this court. DBM alleged that decision of RTC is contrary to law.
ISSUE : WON INP retirees should ge the same retirement benefits accorded to PNP retirees.
HELD : Yes. WITH RA 6975 INP , at most, was transformed to become PNP, minus or course its military character and
complexion. INP was, in effect, merely absorbed by the PNP and not abolished. Members of the INP which include the
herein respondents are, therefore, not excluded from availing themselves of the retirement benefits accorded to PNP
retirees .Indeed, to bar payment of retirement pay differential to INP members who were already retired before R.A. No.
6975 became effective would even run counter to the purpose of NAPOLCOM Resolution No. 8 itself, which is to
rationalize the retirement system of the PNP taking into consideration existing retirement and benefit systems (including
R.A. No. 6975 and P.D. No. 1184) of the different components thereof to ensure that no member of the PNP shall suffer
any diminution in the retirement benefits due them before the creation of the PNP
Mendoza vs PNP GR 139658 2005
FACTS : This case stemmed from the affidavit-complaint for illegal arrest, illegal detention, physical injuries, and robbery
filed by Teodoro V. Conti against PO3 William M. Mendoza, now petitioner, and PO2 Angelita Ramos. Both were members
of the PNP .Allege that Mendoza et al forcibly arrested Conti . While in detention Conti was ordred to swallow his gold
necklace and when he resisted Ramos struck him with a gun and inserted the barrel of the gun to this mouth. Thereafter
he was mauled . Said police also took his money and jewelries. Mendoza was dismissed from service. He filed an
appeal with Regional Appellate Board (RAB) , which was denied. Petitioner filed with RTC after his motion for
reconsideration was denied by RAB . RAB filed a motion to dismiss the petition which was denied by RTC. CA granted
the motion of RAB that contending that petitioner failed to exhaust all administrative remedies.
ISSUE : WON Petitioner needs to undertake all administrative remedies before seeking judicial intervention.
HELD : YES . It is clear from the above provisions that the Decision of the PNP Regional Director imposing upon a PNP
member the administrative penalty of dismissal from the service is appealable to the RAB. From the RAB Decision, the
aggrieved party may then appeal to the Secretary of the DILG. Petitioner did not interpose an appeal to the DILG
Secretary. It bears emphasis that in the event the Secretary renders an unfavorable decision, petitioner may still elevate
his case to the Civil Service Commission. In cases where the decision rendered by a bureau or office is appealable to the
Civil Service Commission, the same may initially be appealed to the Department and finally to the Commission.
Petitioners failure to exhaust all administrative remedies is fatal to his cause. It is elementary that where, as here,
a remedy is available within the administrative machinery, this should first be resorted to. Petition Dismissed.
XVII AMENDMENT AND REVISION
Section 1 Amendment and Revision
IMBONG VS COMELEC GR 32432 1970
FACTS : This is a petition for declaratory judgment. These are 2 separate but related petitions of running candidates for
delegates to the Constitutional Convention assailing the validity of RA 6132.Gonzales: Sec, 2, 4, 5 and Par 1 Sec 8, and
validity of entire law Imbong: Par 1 Sec 8. Claiming that it prejudices their rights as such candidates
RA 6132-Concon Act 1970, repealed RA 4919, implemented Res No. 2 & 4.
Sec 4: considers all public officers/employees as resigned when they file their candidacy
Sec 2: apportionment of delegates
Sec 5: Disqualifies any elected delegate from running for any public office in the election or from assuming any appointive
office/position until the final adournment of the ConCon.
Par 1 Sec 8: ban against all political parties/organized groups from giving support/representing a delegate to the
convention.

ISSUE: Does the Congress have the right to call for a constitutional convention and set the parameters of such
convention?
HELD : 1. The Congress has authority to call a constitutional convention as the constituent assembly. The Congress also
has the authority to enact implementing details, contained in Res. Nos. 2 and 4 .
and R.A. 6132, since such details are within the competence of the Congress in exercise of its legislative power.2. The
provisions are constitutional. Sec. 4 of R.A. 6132 is merely in application with Sec. 2 of Art. XII of the Constitution and
does not constitute a denial of due process or equal protection of the law. Sec 2 RA 6132: it is a mere implementation of
Resolution 4 and is enough that the basis employed
for such apportions is reasonable. Macias case relied by Gonzales is not reasonable for that case granted
more representatives to provinces with less population and vice versa. In this case, Batanes is equal to the number of
delegates I other provinces with more population.
- Sec 5: State has right to create office and parameters to qualify/disqualify members thereof.
Furthermore, this disqualification is only temporary. This is a safety mechanism to prevent political
figures from controlling elections and to allow them to devote more time to the Constituional Convention.
- Par 1 Sec 8: this is to avoid debasement of electoral process and also to assure candidates equal opportunity since
candidates must now depend on their individual merits, and not the support of political parties. This provision does not
create discrimination towards any particular party/group, it applies to all organizations.
LAMBINO VS COMELEC 174153
Facts: Petitioners (Lambino group) commenced gathering signatures for an initiative petition to change the 1987
constitution, they filed a petition with the COMELEC to hold a plebiscite that will ratify their initiative petition under RA
6735 These proposed changes will shift the present Bicameral-Presidential system to a Unicameral-Parliamentary form
of government.
Issue: WON peoples initiative include revisions to the Constitution.
HELD : NO. Section 1 of Article XVII, referring to the first and second modes, applies to "[A]ny amendment to, or revision
of, this Constitution." The framers of the Constitution intended, and wrote, a clear distinction between "amendment" and
"revision" of the Constitution. The framers intended, and wrote, that only Congress or a constitutional convention may
propose revisions to the Constitution. The framers intended, and wrote, that a people's initiative may propose only
amendments to the Constitution. There can be no dispute that a people's initiative can only propose amendments to the
Constitution since the Constitution itself limits initiatives to amendments. There can be no deviation from the
constitutionally prescribed modes of revising the Constitution. A popular clamor, even one backed by 6.3 million
signatures, cannot justify a deviation from the specific modes prescribed in the Constitution itself . Only Congress or a
constitutional convention may propose revisions to the Constitution
SECTION 2 INITIATIVE
DEFENSOR SANTIAGO VS COMELEC 1997
Related Law : 6735 AN ACT PROVIDING FOR A SYSTEM OF INITIATIVE AND REFERENDUM AND
APPROPRIATING FUNDS THEREFOR
COMELEC Resolution No. 2300, adopted on 16 January 1991 to govern the conduct of initiative on the Constitution and
initiative and referendum on national and local laws
FACTS:
Private respondent filed with public respondent Commission on Elections (COMELEC) a Petition to Amend the
Constitution, to Lift Term Limits of Elective Officials, by Peoples Initiative (Delfin Petition) wherein Delfin asked the
COMELEC for an order (1) Fixing the time and dates for signature gathering all over the country; (2) Causing the
necessary publications of said Order and the attached Petition for Initiative on the 1987 Constitution, in newspapers of
general and local circulation; and (3) Instructing Municipal Election Registrars in all Regions of the Philippines, to assist
Petitioners and volunteers, in establishing signing stations at the time and on the dates designated for the purpose. Delfin
asserted that R.A. No. 6735 governs the conduct of initiative to amend the Constitution and COMELEC Resolution No.
2300 is a valid exercise of delegated powers. Petitioners contend that R.A. No. 6375 failed to be an enabling law because
of its deficiency and inadequacy, and COMELEC Resolution No. 2300 is void.
ISSUE: Whether or not R.A. No. 6735 is adequate to cover the system of initiative on amendment to the
Constitution, and (3) COMELEC Resolution No. 2300 is valid.

HELD:
NO. R.A. 6735 is inadequate to cover the system of initiative on amendments to the Constitution.
Under the said law, initiative on the Constitution is confined only to proposals to AMEND. The people are not accorded the
power to "directly propose, enact, approve, or reject, in whole or in part, the Constitution" through the system of initiative.
They can only do so with respect to "laws, ordinances, or resolutions." The use of the clause "proposed laws sought to be
enacted, approved or rejected, amended or repealed" denotes that R.A. No. 6735 excludes initiative on amendments to
the Constitution.
Also, while the law provides subtitles for National Initiative and Referendum and for Local Initiative and Referendum, no
subtitle is provided for initiative on the Constitution. This means that the main thrust of the law is initiative and referendum
on national and local laws. If R.A. No. 6735 were intended to fully provide for the implementation of the initiative on
amendments to the Constitution, it could have provided for a subtitle therefor, considering that in the order of things, the
primacy of interest, or hierarchy of values, the right of the people to directly propose amendments to the Constitution is far
more important than the initiative on national and local laws.
While R.A. No. 6735 specially detailed the process in implementing initiative and referendum on national and local laws, it
intentionally did not do so on the system of initiative on amendments to the Constitution.
COMELEC Resolution No. 2300 is hereby declared void and orders the respondent to forthwith dismiss the Delfin Petition
. TRO issued on 18 December 1996 is made permanent.
COMELEC RESOLUTION NO. 2300 insofar as it prescribes rules and regulations on the conduct of initiative on
amendments to the Constitution is void. It logically follows that the COMELEC cannot validly promulgate rules and
regulations to implement the exercise of the right of the people to directly propose amendments to the Constitution
through the system of initiative. It does not have that power under R.A. No. 6735.
WHEREFORE, petition is GRANTED.
LAMBINO VS COMELEC
Facts: Petitioners (Lambino group) commenced gathering signatures for an initiative petition to change the 1987
constitution, they filed a petition with the COMELEC to hold a plebiscite that will ratify their initiative petition under RA
6735. Lambino group alleged that the petition had the support of 6M individuals fulfilling what was provided by art 17 of
the constitution. Their petition changes the 1987 constitution by modifying sections 1-7 of Art 6 and sections 1-4 of Art 7
and by adding Art 18. the proposed changes will shift the present bicameral- presidential form of government to
unicameral- parliamentary. COMELEC denied the petition due to lack of enabling law governing initiative petitions and
invoked the Santiago Vs. Comelec ruling that RA 6735 is inadequate to implement the initiative petitions.
Issue:
Whether or Not the Lambino Groups initiative petition complies with Section 2, Article XVII of the Constitution on
amendments to the Constitution through a peoples initiative.
Whether or Not this Court should revisit its ruling in Santiago declaring RA 6735 incomplete, inadequate or wanting in
essential terms and conditions to implement the initiative clause on proposals to amend the Constitution.
Held: According to the SC the Lambino group failed to comply with the basic requirements for conducting a peoples
initiative. The Court held that the COMELEC did not grave abuse of discretion on dismissing the Lambino petition.
1. The Initiative Petition Does Not Comply with Section 2, Article XVII of the Constitution on Direct Proposal by the People
The petitioners failed to show the court that the initiative signer must be informed at the time of the signing of the nature
and effect, failure to do so is deceptive and misleading which renders the initiative void.
2. The Initiative Violates Section 2, Article XVII of the Constitution Disallowing Revision through Initiatives
The framers of the constitution intended a clear distinction between amendment and revision, it is intended that the third
mode of stated in sec 2 art 17 of the constitution may propose only amendments to the constitution. Merging of the
legislative and the executive is a radical change, therefore a constitutes a revision.

SECTION 4 RATIFICATION
GONZALES VS COMELEC G.R. No. L-28196

1967

FACTS: This case is composed of consolidated cases filed separately by Petitioner Gonzalez and PHILCONSA assailing
for the declaration of nullity of RA. No. 4913 and R.B.H. No. 1 and 3. On March 16, 1967, the Senate and the House of
Representatives passed the following resolutions (Resolution of Both Houses/R.B.H resolutions are null and void
because:
1. The Members of Congress, which approved the proposed amendments, as well as the resolution calling a convention
to propose amendments, are, at best, de facto Congressmen;
2. Congress may adopt either one of two alternatives propose amendments or call a convention therefore but may not
avail of both that is to say, propose amendment and call a convention at the same time;
3. The election, in which proposals for amendment to the Constitution shall be submitted for ratification, must be
aspecial election, not a general election, in which officers of the national and local governments such as the elections
scheduled to be held on November 14, 1967 will be chosen; and
4. The spirit of the Constitution demands that the election, in which proposals for amendment shall be submitted to the
people for ratification, must be held under such conditions which, allegedly, do not exist as to give the people a
reasonable opportunity to have a fair grasp of the nature and implications of said amendments.
Subsequently, Congress passed a bill, which became RA No. 4913, providing that the amendments to the Constitution
proposed in the aforementioned Resolutions No. 1 and 3 be submitted, for approval by the people at the general elections
on November 14, 1967. This act fixes the date and manner of election for the proposed amendments to be voted upon by
the people, and appropriates funds for said election.
ISSUE : Whether or not RA No. 4913 in unconstitutional..
Held : NO .
1.) Pursuant to Article XV of the 1935 Constitution, SC held that there is nothing in this provision that states that the
election referred to is special, different from the general election. The Congress deemed it best to submit the
amendments for ratification in accordance with the provisions of the Constitution. It does not negate its authority
to submit proposed amendments for ratification in general elections
2.) Referring particularly to the contested proposals for amendment, the sufficiency or insufficiency, from a
constitutional angle, of the submission thereof for ratification to the people on November 14, 1967, depends in
the view of those who concur in this opinion, and who, insofar as this phase of the case, constitute the minority
upon whether the provisions of Republic Act No. 4913 are such as to fairly apprise the people of the gist, the main
idea or the substance of said proposals, which is under R. B. H. No. 1 the increase of the maximum number
of seats in the House of Representatives, from 120 to 180, and under R. B. H. No. 3 the authority given to
the members of Congress to run for delegates to the Constitutional Convention and, if elected thereto, to
discharge the duties of such delegates, without forfeiting their seats in Congress. We believe that Republic Act
No. 4913 satisfies such requirement and that said Act is, accordingly, constitutional.
3.) ."In the cases at bar, it is conceded that the R. B. H. Nos. 1 and 3 have been approved by a vote of three-fourths
of all the members of the Senate and of the House of Representatives voting separately.
We do not believe it has been satisfactorily shown that Congress has exceeded the limits thereof in enacting Republic
Act No. 4913. Presumably, it could have done something better to enlighten the people on the subject-matter thereof. But,
then, no law is perfect
Petition is therefore DENIED.
TOLENTINO VS COMELEC 1971
The 1971 Constitutional Convention came into being by virtue of two resolutions of the Congress approved in its capacity
as a constituent assembly convened for the purpose of calling a convention to propose amendments to the Constitution.
After election of delegates held on November 10, 1970, the Convention held its inaugural session on June 1, 1971.

In the morning of September 28, 1970, the Convention approved Organic Resolution No. 1 which is entitled as, "A
RESOLUTION AMENDING SECTION 1 OF ARTICLE V OF THE CONSTITUTION SO AS TO LOWER THE VOTING AGE
TO 18." On September 30, 1971, the COMELEC "resolved" to follow the mandate of the Convention, that it will hold the
said plebiscite together with the senatorial elections on November 8, 1971 .
Petitioner, Arturo Tolentino, filed a petition for prohibition, its main thrust being that Organic Resolution No. 1 and the
necessary implementing resolutions subsequently approved have no force and effect as laws in so far as they provide for
the holding of a plebiscite co-incident with the senatorial elections, on the ground that the calling and holding of such a
plebiscite is, by the Constitution, a power lodged exclusively in Congress as a legislative body and may not be exercised
by the Convention, and that, under Article XV Section 1 of the 1935 Constitution, the proposed amendment in question
cannot be presented to the people for ratification separately from each and all other amendments to be drafted and
proposed by the Constitution.
ISSUE:
Whether or not the Organic Resolution No. 1 of the 1971 Constitutional Convention violative to the Constitution.
HELD:
NO. All the amendments to be proposed by the same Convention must be submitted to the people in a single "election" or
plebiscite. In order that a plebiscite for the ratification of a Constitutional amendment may be validly held, it must provide
the voter not only sufficient time but ample basis for an intelligent appraisal of the nature of the amendment per se but as
well as its relation to the other parts of the Constitution with which it has to form a harmonious whole. In the present
context,
where the Convention has hardly started considering the merits, if not thousands, of proposals to amend the existing
Constitution, to present to the people any single proposal or a few of them cannot comply with this requirement.
Article XVIII Transitory provision
Sec. 8
MMDA v. Bel-Air Village Association, GR 135962, March 27,2000
FACTS: Metropolitan Manila Development Authority (MMDA), petitioner herein, is a Government Agency tasked with the
delivery of basic services in Metro Manila. Bel-Air Village Association (BAVA), respondent herein, received a letter of
request from the petitioner to open Neptune Street of Bel-Air Village for the use of the public. The said opening of Neptune
Street will be for the safe and convenient movement of persons and to regulate the flow of traffic in Makati City. This was
pursuant to MMDA law or Republic Act No. 7924. On the same day, the respondent was appraised that the perimeter wall
separating the subdivision and Kalayaan Avenue would be demolished.
The respondent, to stop the opening of the said street and demolition of the wall, filed a preliminary injunction and a
temporary restraining order. Respondent claimed that the MMDA had no authority to do so and the lower court decided in
favor of the Respondent. Petitioner appealed the decision of the lower courts and claimed that it has the authority to open
Neptune Street to public traffic because it is an agent of the State that can practice police power in the delivery of basic
services in Metro Manila.

ISSUE: Whether or not the MMDA has the mandate to open Neptune Street to public traffic pursuant to its regulatory and
police powers.
HELD: The Court held that the MMDA does not have the capacity to exercise police power. Police power is primarily
lodged in the National Legislature. However, police power may be delegated to government units. Petitioner herein is a
development authority and not a political government unit. Therefore, the MMDA cannot exercise police power because it
cannot be delegated to them. It is not a legislative unit of the government. Republic Act No. 7924 does not empower the
MMDA to enact ordinances, approve resolutions and appropriate funds for the general welfare of the inhabitants of
Manila. There is no syllable in the said act that grants MMDA police power.
It is an agency created for the purpose of laying down policies and coordinating with various national government
agencies, peoples organizations, non-governmental organizations and the private sector for the efficient and expeditious
delivery of basic services in the vast metropolitan area.

Sec 16
Dario v. Mison, 176 SCRA 84 (1989)

FACTS: The petitioners questioned their separation from the Bureau of Customs as a result of the reorganization
undertaken by the respondent Commissioner of Customs pursuant to EO 127 4
ISSUE: Whether or not the reorganization was valid
HELD:Reorganizations in this jurisdiction have been regarded as valid provided they are pursued in good faith. As a
general rule, reorganization is carried out in "good faith" if it is for the purpose of economy or to make bureaucracy more
efficient. In that event, no dismissal (in case of a dismissal) or separation actually occurs because the position itself
ceases to exist. And in that case, security of tenure would not be a Chinese wall. Be that as it may, if the "abolition," which
is nothing else but a separation or removal, is done for political reasons or purposely to defeat sty of tenure, or otherwise
not in good faith, no valid "abolition' takes place and whatever "abolition' is done, is void ab initio. There is an invalid
"abolition" as where there is merely a change of nomenclature of positions, 82 or where claims of economy are belied by
the existence of ample funds.
Mendoza v. Quisumbing, 186 SCRA 108 (1990)
FACTS: Immediately after the President Cory was sworn into office, she issued Proclamation No. 1 declaring as policy the
reorganization of the government. Thereafter, the President issued Executive Orders directing the reorganization of
various different departments of the government which affected their employees, among those affected was Francisco
Mendoza.
Petitioner Mendoza was the Schools Division Superintendent of Surigao City who was reappointed by respondent
Quisumbing as such with a "permanent" status. He has served the Department of Education for forty-two (42) years,
moving up the ranks in the public schools system. Then, the petitioner received the letter-order informing him that he
would be considered separated from the service without prejudice to availment of benefits. The letter particularly stated
that consistent with the mandate of reorganization to achieve greater efficiency and effectiveness, all incumbent
officials/personnel are on 'holdover' status unless advised otherwise. In the meantime, the petitioner, in a letter wrote
Secretary Quisumbing requesting reconsideration of the letter-order. The letter was forwarded to the Reorganization
Appeals Board (RAB). The motion for reconsideration remained unactedupon, hence the petitioner filed the instant petition
for certiorari, prohibition and mandamus with preliminary injunction.

ISSUES: (1) Whether or not the various reorganization programs in different agencies and/or departments of the
government implementing the orders issued pursuant to the Presidents proclamations were valid, and (2) Whether or not
the letter-order of the then Secretary Quisumbing which terminated his employment as Schools Division Superintendent
of Surigao City was valid.

HELD: There is no dispute over the authority to carry out a valid reorganization in any branch or agency of Government.
Pursuant to the Provisional Constitution and the various Executive Orders issued by the President when she was the sole
law-making authority, the different Departments of Government were authorized to carry on reorganization programs. The
grant of authority, however, was accompanied by guidelines and limitations. It was never intended that department and
agency heads would be vested with untrammelled and automatic authority to dismissthe millions of government workers
on the stroke of a pen and with the same sweeping powerdetermine under their sole discretion who would be appointed or
reappointed to the vacant positions. Reorganization was mandated by the People to remove "all iniquitous vestiges of the
previous regime." Under this mandate, the mass of lowly employees in the bottom rungs of the governmental hierarchy,
ordinarily constant and apolitical, were not intended to be summarily dismissed unless basic reasons outweighed or
overcame the rights to their jobs built up so laboriously over the years. The promotion of simplicity, economy and
efficiency is the usual standards which enables a delegation of powers in reorganization statutes to pass the test of
validity. Because the heads of departments and agencies concerned have chosen to rely on their own concepts of
unlimited discretion and progressive ideas on reorganization instead of showing that they have faithfully complied with the
clear letter and spirit of the two Constitutions and the statutes governing reorganization, the said reorganizations (in these
consolidated petitions) were set aside. The letter-order was also set aside, and the Secretary of DECS was ordered to
restore petitioner Mendoza to his position as Schools Division Superintendent of Surigao City without loss; of seniority
rights and with back salaries reckoned from the date of his termination.

Ontiveros v. CA, GR 145401 May 7, 2001


FACTS: Petitioner was Security Officer I in the Investigation and Security Division of the Ministry of Tourism. He was
dismissed from the service for inefficiency, incompetence, and unauthorized absences.Petitioner argues that his case

does not fall under the Review Committee's jurisdiction because his separation from the service was not in consequence
of the re-organization of the government, as provided in the Provisional (also known as the Freedom Constitution), but
was for cause; hence, appeal lies with the CSC.
ISSUE: Whether or not petitioners case falls under the Review Committee's jurisdiction or CSC
HELD:The Court of Appeals ruled that the CSC had no jurisdiction over petitioner's appeal, the proper appellate body
being the Review Committee established under E.O. No. 17. In addition, the appeals court held that review of petitioner's
dismissal was barred by laches. Petitioner was not dismissed by virtue of a proclamation or executive order of the
President of the Philippines nor by reason of the designation or appointment and qualification of his successor.
It is clear that petitioner's dismissal came within the coverage of E.O. No. 17, Sec3(3) and (5).Pursuant to Sec2 of the
order, the memorandum of dismissal of Minister Gonzales is considered the act of the then President.Nor can it be argued
that this case is not covered by Art. III, Sec2 of the Provisional Constitution since the grounds for petitioner's dismissal are
grounds under civil service laws and regulations. E.O. No. 17 provided the grounds for the separation of employees from
the service not to bring their cases under ordinary civil service laws and regulations but to provide limits on what otherwise
would be absolute discretion and thus prevent an abuse of power.

Sec 25
Bayan v. Zamora, GR 138570 October 10, 2000
FACTS: RP and US forged a military bases agreement which formalized, among others, the use of installations in the
Philippine territory by the US military personnel. To further strengthen their defense and security relationship, the
Philippines and the US entered into a Mutual Defense Treaty. Under the treaty, the parties agreed to respond to any
external armed attack on their territory, armed forces, public vessels and aircraft. RP-US Military Bases Agreement
expired. The United States panel met with the Philippine panel to discuss, among others, the possible elements of the
Visiting Forces Agreement (VFA). President Fidel Ramos approved the VFA.
President Estrada ratified the VFA; respectively signed by Secretary Siazon and US Ambassador Thomas Hubbard.
Senate approved it by (2/3) votes.
ISSUE:
Whether or not the VFA governed by the provisions of Section 21, Art VII or of Section 25, Article XVIII of the Constitution
HELD: The 1987 Philippine Constitution contains two provisions requiring the concurrence of the Senate on treaties or
international agreements:
Sec. 21 Art. VII
No treaty or international agreement shall be valid and effective unless concurred in by at least 2/3 of all the Members of
the Senate.
Applies to any form of treaty and international agreement in general with a wide variety of subject matter. All treaties and
international agreements entered into by the Philippines, regardless of subject matter, coverage or particular designation
requires the concurrence of the Senate to be valid and effective.
Sec. 25 Art. XVIII
After the expiration in 1991 of the Agreement between the RP and the US concerning Military Bases, foreign military
bases, troops or facilities shall not be allowed in the Philippines except under a treaty duly concurred in and when the
Congress so requires, ratified by a majority of votes cast by the people in a national referendum held for that purpose, and
recognized as a treaty by the Senate by the other contracting state.
Applies to treaties which involve the presence of foreign military bases, troops and facilities in the Philippines.The fact that
the President referred the VFA to the Senate under Sec. 21 Art. VII, and that Senate extended its concurrence under the
same provision is immaterial because both provisions share a common ground.
Undoubtedly, Sec. 25 Art. XVIII which specifically deals with treaties involving foreign military bases and troops should
apply in the instant case. Hence, for VFA to be constitutional it must sufficiently meet the following requisites:
a) It must be under a treaty.
b) The treaty must be duly concurred in by the Senate, and when so required by Congress, ratified by a majority of votes
cast by the people in a national referendum.
c) Recognized as a treaty by the other contracting State.

It is inconsequential whether the US treats the VFA only as an executive agreement because, under international law, an
executive agreement is as binding as a treaty. They are equally binding obligations upon nations.
Sec 26
Joya v. PCGG, 225 SCRA 568 (1993)
FACTS:The Republic of the Philippines through the PCGG entered into a Consignment Agreement with Christies of New
York, selling 82 Old Masters Paintings and antique silverware seized from Malacanang and the Metropolitan Museum of
Manila alleged to be part of the ill-gotten wealth of the late Pres. Marcos, his relatives and cronies. Prior to the auction
sale, COA questioned the Consignment Agreement, there was already opposition to the auction sale. Nevertheless, it
proceeded as scheduled and the proceeds of $13,302,604.86 were turned over to the Bureau of Treasury.
ISSUE: Whether or not PCGG has jurisdiction and authority to enter into an agreement with Christies of New York for the
sale of the artworks
HELD:On jurisdiction of the Court to exercise judicial review
The rule is settled that no question involving the constitutionality or validity of a law or governmental act may be heard and
decided by the court unless there is compliance with the legal requisites for judicial inquiry, namely: that the question must
be raised by the proper party; that there must be an actual case or controversy; that the question must be raised at the
earliest possible opportunity; and, that the decision on the constitutional or legal question must be necessary to the
determination of the case itself. But the most important are the first two (2) requisites.

Republic v. Sandiganbayan, 221 SCRA 189 (1993) (powers of PCGG)


FACTS:The four (4) herein consolidated petitions have as their common prayer the nullification of the already approved
and partially implemented compromise agreement dated November 3, 1990 executed between Roberto S. Benedicto and
the Presidential Commission on Good Government (PCGG) represented by its then Chairman, David M. Castro.
HELD: The court ruled that the authority of the PCGG to validly enter into compromise agreement for the purpose of
avoiding litigation or putting an end to one already commenced was indisputable.PCGG has been "primarily charged" with
the responsibility of recovering illegally acquired or misappropriated assets.It is advocated by the PCGG that respondent
Benedicto retaining a portion of the assets is anathema to, and incongruous with, the zero-retention policy of the
government in the pursuit for recovery of all ill-gotten wealth pursuant to Section 2(a) of Executive Order No. 1. While full
recovery is ideal, the PCGG is not precluded from entering into a compromise agreement which entails
reciprocal concessions if only to expedite recovery so that the remaining "funds, assets and other properties may be used
to hasten national economic recovery" (3rd WHEREAS clause, Executive Order No. 14-A).

Cojuangco v. Roxas, 195 SCRA 797 (1991) (vote of sequestered shares)


FACTS:On April 18, 1989, the annual meeting of shareholders of SMC was held. Among the matters taken up was the
election of fifteen (15) members of the board of directors for the ensuing year. Because of PCGGs claim that the shares
of stock were under sequestration, PCGG was allowed to represent and vote the shares of stock.
Representatives of the corporate shares present at the meeting claimed that the shares are not under sequestration; or
that if they are under sequestration, the PCGG had no right to vote the same.
ISSUE:whether or not the Presidential Commission on Good Government (PCGG) may vote the sequestered shares of
stock of San Miguel Corporation (SMC) and elect its members of the board of directors.
HELD:The Court finds and so holds that the PCGG has no right to vote the sequestered shares of petitioners including
the sequestered corporate shares. Only their owners, duly authorized representatives or proxies may vote the said
shares. Consequently, the election of private respondents Adolfo Azcuna, Edison Coseteng and Patricio Pineda as
members of the board of directors of SMC for 1990-1991 should be set aside. The right to vote sequestered shares of
stock is vested in the actual shareholders not in the PCGG.

Araneta v. Sandiganbayan, 242 SCRA 482 (1995) (investigate/prosecutory powers)


A careful reading of Sections 2(a) and 3, of Executive Order No. 1, in relation with Sections 1, 2 and 3 of Executive Order
No. 14, show that what the authority of the respondent PCGG to investigate and prosecute covers are:

(a)
The investigation and prosecution of the civil action for the recovery of ill-gotten wealth under Republic Act No.
1379, accumulated by former President Ferdinand E. Marcos, his immediate family, relatives, subordinates and close
associates, whether located in the Philippines or abroad, including the takeover or sequestration of all business
enterprises and entities owned or controlled by them, during his administration, directly or through his nominees, by taking
undue advantage of their public office and/or using their powers, authority, influence, connections or relationship; and
(b)
The investigation and prosecution of such offenses committed in the acquisition of said ill-gotten wealth as
contemplated under Section 2(a) of Executive Order No. 1.
HELD: It is clear that Araneta used his power, influence, connections or relationship as son-in-law of the late President
Marcos and that by reason of the manner by which the acquisition was effected, the wealth or items (a) and (c) are illgotten.
There was no grave abuse of discretion on the part of the Sandiganbayan in refusing to dismiss the case. While the
information may have used the exact terminologies of R.A. No. 3019, viz., "with manifest partiality and evident bad faith;"
"with manifestly and grossly disadvantageous terms and conditions;" "unwarranted benefits, advantages and
preferences;" and "causing undue injury to the damage and prejudice of the Government," it is, nevertheless, clear that
the charge against petitioner (for violation of R.A. No. 3019) is in relation to E.O. No. 1, No. 2, and No. 14-A, on the
ultimate recovery of ill-gotten wealth.

Romualdez v. Sandiganbayan, 244 SCRA 152 (1995) (authority over ill-gotten wealth)
The PCGG has authority to conduct preliminary investigations in ill-gotten wealth cases, including the "non-filing of
statements of assets and liabilities (as) a means of concealing one's assets and frustrating the efforts of the Government
to determine the actual value or extent of a public officer's wealth.
It does not seem, however, that this power extends to other acts or omissions not involving "ill-gotten wealth" penalized by
the Anti-Graft and Corrupt Practices Act. However, other violations of the Anti-Graft and Corrupt Practices Act not
otherwise falling under the Sections 2(a) and 3, of Executive Order No. 1, require a previous authority of the President for
the respondent PCGG to investigate and prosecute in accordance with Section 2(b) of Executive Order No. 1. Otherwise,
jurisdiction over such cases is vested in the Ombudsman and other duly authorized investigating agencies.
Thus for a penal violation to fall under the jurisdiction of the respondent PCGG under Section 2(a) of Executive Order No.
1, the following elements must concur:
(1).
It must relate to alleged ill-gotten wealth;(2).
Of the late President Marcos, his immediate family, relatives,
subordinates and close associates;(3). Who took undue advantage of their public office and/or used their power,
authority, influence, connections or relationship.
HELD: The crimes ascribed to Romualdez (failure to file his annual statements of assets and liabilities) do not "relate to
alleged ill-gotten wealth" amassed by him. No such relation may be perceived in the indictments themselves, which in fact
merely state that there was no justifiable cause for Romualdez's refusal or failure to file his annual statements. Moreover,
the Sandiganbayan itself made the finding that the cases against Romualdez did not refer to acquisition of wealth under a
crony status, but "solely (to) his bare physical non-compliance with his mechanical duty to file his statement of assets and
liabilities over a period of twenty-four (24) years . . ;" and that the omissions have no bearing on Civil Case No. 0035
against Romualdez involving transactions in which he "allegedly took advantage of his relationship with the spouses
Ferdinand and Imelda Marcos."
The Court therefore declares invalid the preliminary investigation conducted by the PCGG over the 24 offenses ascribed
to Romualdez (of failure to file annual statements of assets and liabilities), for lack of jurisdiction of said offenses.

Republic v. Sandiganbayan, 240 SCRA 376 (1995) (judicial action)


MAIN POINT:A sequestration or freeze order shall be issued only upon showing of a prima facie case. The order and the
list of the sequestered or frozen properties shall forthwith be registered with the proper court. For orders issued before the
ratification of this Constitution, the corresponding judicial action or proceeding shall be filed within six months from its
ratification. For those issued after such ratification, the judicial action or proceeding shall be commenced within six months
from the issuance thereof.The sequestration or freeze order is deemed automatically lifted if no judicial action or
proceeding is commenced as herein provided.
Thus, the rationale for the limitations placed upon the power of sequestration, etc. by the Constitution, these being the
following:

1.
The authority to issue such orders was made "operative for not more than eighteen months after ratification of . . .
(the) Constitution;" i.e., not beyond 18 months form February 2, 1987, unless extended by the Congress "on the national
interest, as certified by the President;" 2.
Said orders could issue only upon showing of a prima facie case;3.
The order and the list of sequestered or frozen properties had to be registered forthwith with the proper court; the
Sandiganbayan, according to law; 4.
For orders issued before ratification of the 1987 Constitution, the corresponding
judicial action or proceeding should be filed within six months therefrom (i.e., six months from February 2, 1987); and for
those issued thereafter, within six months from issuance of the order of the sequestration etc.
HELD:The issue in all the cases at bar chiefly concerns the fourth limitation pursuant to which the PCGG had to file "the
corresponding judicial action or proceeding" within a fixed period of six months. The evident purpose was to preclude the
possibility that the PCGG indefinitely maintain its orders of sequestration, etc. and to compel it, within a reasonable time,
to bring them into the realm of judicial oversight, evaluation and control, to the end that excesses of the officials and
agents enforcing and implementing said orders might be prevented and avoided and private rights duly protected and
vindicated, while the main business of determining the character of the property as "ill-gotten wealth" or not was being
attended to in court.

Sec 27
De Leon v. Esguerra
Facts: On May 17, 1982, petitioner Alfredo M. De Leon was elected Barangay Captain together with the other petitioners
as Barangay Councilmen of Barangay Dolores, Muncipality of Taytay, Province of Rizal in a Barangay election held under
Batas Pambansa Blg. 222, otherwise known as Barangay Election Act of 1982
On February 9, 1987, petitioner De Leon received a Memorandum antedated December 1, 1986 but signed by
respondent OIC Governor Benjamin Esguerra on February 8, 1987 designating respondent Florentino G. Magno as
Barangay Captain of Barangay Dolores and the other respondents as members of Barangay Council of the same
Barangay and Municipality.
Petitoners prayed to the Supreme Court that the subject Memoranda of February 8, 1987 be declared null and void and
that respondents be prohibited by taking over their positions of Barangay Captain and Barangay Councilmen.
Petitioners maintain that pursuant to Section 3 of the Barangay Election Act of 1982 (BP Blg. 222), their terms of office
shall be six years which shall commence on June 7, 1988 and shall continue until their successors shall have elected and
shall have qualified. It was also their position that with the ratification of the 1987 Philippine Constitution, respondent OIC
Governor no longer has the authority to replace them and to designate their successors.
On the other hand, respondents contend that the terms of office of elective and appointive officials were abolished and
that petitioners continued in office by virtue of Sec. 2, Art. 3 of the Provisional Constitution and not because their term of
six years had not yet expired; and that the provision in the Barangay Election Act fixing the term of office of Barangay
officials to six years must be deemed to have been repealed for being inconsistent with Sec. 2, Art. 3 of the Provisional
Constitution.
Issue: Whether or not the designation of respondents to replace petitioners was validly made during the one-year period
which ended on Feb 25, 1987.
Ruling: Supreme Court declared that the Memoranda issued by respondent OIC Gov on Feb 8, 1987 designating
respondents as Barangay Captain and Barangay Councilmen of Barangay Dolores, Taytay, Rizal has no legal force and
effect.
The 1987 Constitution was ratified in a plebiscite on Feb 2, 1987, therefore, the Provisional Constitution must be deemed
to have superseded. Having become inoperative, respondent OIC Gov could no longer rely on Sec 2, Art 3, thereof to
designate respondents to the elective positions occupied by petitioners. Relevantly, Sec 8, Art 1 of the 1987 Constitution
further provides in part:
"Sec. 8. The term of office of elective local officials, except barangay officials, which shall be determined by law, shall be
three years x x x."
Until the term of office of barangay officials has been determined by aw, therefore, the term of office of 6 years provided
for in the Barangay Election Act of 1982 should still govern.
Art 26
Cojuanco v Roxas

The issue squarely presented by the petitioners is whether or not the Presidential Commission on Good Government
(PCGG) may vote the sequestered shares of stock of San Miguel Corporation (SMC) and elect its members of the board
of directors. Sandiganbayan held that the PCGG can vote.
SC held that The PCGG cannot perform acts of strict ownership of sequestered property. It is a mere conservator. It may
not vote the shares in a corporation and elect the members of the board of directors.. PCGG has no right to vote the
sequestered shares of petitioners including the sequestered corporate shares. Only their owners, duly authorized
representatives or proxies may vote the said shares
It is through the right to vote that the stockholder participates in the management of the corporation. The right to vote,
unlike the rights to receive dividends and liquidating distributions, is not a passive thing because management or
administration is, under the Corporation Code, vested in the board of directors, with certain reserved powers residing in
the stockholders directly. The board of directors and executive committee (or management committee) and the corporate
officers selected by the board may make it very difficult if not impossible for the PCGG to carry out its duties as
conservator if the Board or officers do not cooperate, are hostile or antagonistic to the conservator's objectives.
Thus, it is necessary to achieve a balancing of or reconciliation between the stockholder's right to vote and the
conservator's statutory duty to recover and in the process thereof, to conserve assets, thought to be ill-gotten wealth, until
final judicial determination of the character of such assets or until a final compromise agreement between the parties is
reached

You might also like